You are on page 1of 274

abstract algebra

RASHID ALI
B.S Mathematics (Gold Medalist)
Govt. College of Science
Wahdat road Lahore
Page |2

Dedicated to My Parents and Beloved Teachers


Dr Naveed Akhtar
(A.P Math Govt. Dayal Singh College Lahore)
Javed Ali
(A.P Math Govt. College of Science Wahdat Road Lahore)
Page |3

Preface:
This text material is an introduction to learn algebra specially Group Theory, Ring Theory and
Vector Spaces for Bachelor and Masters level. Moreover this will cover MCQs and Interview
questions for Lecturer Mathematics written test of PPSC , FPSC , AJKPSC, SPSC, KPPSC,
BPSC alongwith admission tests of M Phil and PhD of different universities of Pakistan. There
are already many books available in the market for this purpose but I have tried to solve much
more examples so that everyone can understand the subject in a comprehensive way. The idea of
this text appears in my mind when we started preparation of PPSC Lecturer Mathematics in 2020
on our YouTube channel MathLogic.
My interest towards Algebra is due to my BS Instructor Mr. Javed Ali who taught me the course
Theory of Modules in final semester. I realized that I always enjoy studying abstract algebra after
this course. I would say thanks to all of my friends who really helped me in discussions on the
course and writing, formatting of this material. I would say special thanks to very dear Sir Akhtar
Abbas for his valuable suggestions and all contributions. For this collection of Material I have
consulted many of the books specially My favorite book “ Contemporary Abstract Algebra by
Joseph A Gallian ”. Recommended text books will be listed in references.

Errors if any are however all due to my fault. Friends and well wishers are requested to point out
the errors which will be highly acknowledged. Suggestions in the content and formatting will be
appreciated.

This material will be published soon with some changes and adding Interview questions from
Abstract Algebra.

A big thanks to my dear student Ms. Hadiqa Mumtaz by Whom I was able to share this
material to all of you.

Rashid Ali
Lecturer in Mathematics
Govt. Associate College (B) Haveli Lakha, Okara
Whatsapp# 92303-7728892
www.youtube.com/c/mathlogicpk
Page |4

<(1 2)(3 4) , (1 3)(2 4)>

<(1 2 3)> <(2 3 4)>

<(1 2)(3 4)> <(1 3)(2 4)> <(1 4)(2 3)>

“Algebra is generous, She often gives more than is asked of her”.

Jean Le Rond S. Alembert


Page |5

list

CONTENTS

Section:01

Group Theory 05 --- 184

Section:02

Ring Theory 185--- 225

Section:03

Vector Spaces 226 --- 274


Page |6

Groups

Binary Operation
Let and be any function defined from to . Then is called
Binary Operation on .
We write the image of under as instead of

If we can define such a mapping on a non-empty set say then we say is a binary operation
on Or equivalently is closed under .

Example:

usual addition

is closed under usual addition or ordinary addition.


Example:
For any set non-empty set and intersection and union are binary
operations on
i.e is closed under '' '' and '' ''

Q:1 Which of the following is not a binary operation on ?

(a) (b) (c) (d)

Explanation:
 Addition of two rational numbers is a rational number.
 Subtraction is a binary operation on
 Multiplication is a binary operation on
 Division is not binary operation on .Because 0  and we cannot divide any number by
zero. Option (d) is correct.
Page |7

Q:2 Which of the following is a binary operation on ?

(a) (b) (c) (d) √

Explanation:
 √ ( √ )
 √ √
 √ √
Option (d) is correct.

Q:3 Which of the following set is closed under binary operation ?

(a) (b) (c) (d) All of these

Explanation:
Since maximum of any two numbers in is again in . Similarly for and .
Option (d) is correct.

Q:4 Point out the set which is not closed under subtraction ?

(a) (b) (c) (d) None of these

Explanation:
 For any two integers and their difference –
 For any two rational numbers and their difference –
 For any two same irrational numbers –
Option (c) is correct.

Q:5 Which of the following function define binary operation on ?


[ ] [ ]
(a) [ ]
(b) (c) (d)

Note. ( ) stands for greatest common divisor and [ ] stands for least common multiple of
and
Page |8

Explanation:
 Since for [ ]
, so (a) is not binary operation on .
 Since for any two numbers [ ] divides both and
[ ]
both divide which implies that divides . So, and (b) is binary
operation on
 Since for . so (c) is not binary operation on
 Since for
[ ]
. so (d) is not binary operation on .
Option (b) is correct.

Q:6 Which of the following set is closed under binary operation .

(a) (b) (c) (d) Set of Odd Integers

Explanation:
 Since for any two rational numbers, . so (a) is binary operation on
 Since for any two irrational numbers, . so (b) is not binary operation on
 Since for any two natural numbers, . so (c) is not binary operation on
 Also for any two odd integers which is not an odd integer.
Option (a) is correct.

Q:7 Which of the following set is closed under matrix addition as well as matrix
multiplication?

(a) Set of all symmetric matrices (b) Set of all skew-symmetric matrices
(c) Set of all scalar matrices (d) All of these

Explanation:
 Since for * +* + * + which is not symmetric but sum of two
symmetric matrices is always symmetric because for two symmetric matrices and their
sum is
 Since for * +* + * + which is not skew symmetric but sum of two
skew-symmetric matrices is always skew-symmetric. Because

 Since sum as well as product of two scalar matrices is again scalar matrix.
Option (c) is correct.
Page |9

Important Points:
1. A binary operation is commutative if and only if .
e.g ( ) are all closed with
commutative binary operations.
( )
2. No of different binary operations on a set of elements are

Groupoid:
Let be a binary operation on then we say that the set or the pair is a groupoid.

Examples:

} while and set of odd integers under addition are not closed.

Semi group:
A groupoid is called a semi group if  is associative.
i.e.
Note that there is no restriction for distinct elements.

Examples:
While are not semi-groups.

Monoid
A semi group is called a monoid if G has identity under 
i.e. such that

Examples:

is identity of
is identity of
is identity of is identity of

Group:
A monoid is called a group if every element of has inverse in under
i.e. for every
such that

Examples:
} } }

While are not groups as inverses do not exist.


P a g e | 10

Abelian Group:
A group having commutative property is called abelian group.
a b  b  a  a, bG
Examples:
} } } While set of non-singular
matrices of order is a non-abelian group.

Before discussing more examples we will define the order of group.

Order of a Group:
For a finite group the number of elements in a group is called order of group

The order of an infinite group is infinity.

Symmetry:
By symmetry of an object, we mean if an object remains invariant under some transformation
like translation, rotation, reflection.
In this section, we will discuss symmetries of a rectangle, triangle and square.

Symmetries of Rectangle:
A rectangle has symmetry when it is rotated through (clockwise or anti-clockwise) through
centroid and flipped along horizontal and vertical lines which passes through centroid.

2 1

3 4
Rotations

If we rotate this rectangle through then It will not come back to its original position.
Therefore we will rotate through . Here we rotate clockwise and get,

4 3
a„


1 2
P a g e | 11

We denote this rotation by „ ‟. Again if we rotate through it will come back to its original
position. So a  e and there is only one rotation in rectangle through
2

Reflection

Here is two reflections is along and is along


We denote the reflection along by and reflection along by
From below symmetries we can clearly see that and have order
By mean of order if we get reflection of and along and respectively,
we get the identity
Reflection along y-axis
4 3 3 4

Reflection along y-axis

1
1 2 2 1

Reflection along x-axis


4 3 1 2

1 2 4 3

If we rotate „ ‟ through 180 we get „ ‟. We can write =


If we take reflection of „ ‟ along then we get =
This group is called Klein‟s four group.
V4   a, b | a 2  (b) 2  (ab) 2  e 
V4 is an abelian group of order 4.
(German Mathematician Felix Klein)
P a g e | 12

Group of Symmetries of n-sided regular Polygon:


This group is denoted by or whereas represents the number of vertices of sided
regular Polygon.

(Our convention will be )

Dn   x, y | x n  ( y ) 2  ( xy ) 2  e 
} so, there are total distinct elements.
2
Where is rotation through rad and is reflection.
n
Result to remember
for all

Dn is non-abelian  n  3
Group of Symmetries of an Equilateral triangle:
In case of equilateral triangle the number of vertices are and each rotation will be made

through .

1 1

2 3 2 3

1 3

2 3 1 2
P a g e | 13

1 2

Rotation through an angle of 240o

2 3 3 1

1 1

Reflection along vertical median

2 3 3 2

1 3

Reflection along right median

2 3 2 1

1 2

Reflection along left median

2 3 1 3
P a g e | 14

So, from symmetries of triangle we have and are


the distinct elements. Also and are not equal as so the given group is non
abelian. Here, the operation is Composition, I will denote it here by “.”

So, group of symmetries of an equilateral triangle is a non-abelian group of order 6 denoted by


Also it is the smallest non-abelian group.

Group of Symmetries of a Square:


In case of square the number of vertices are and each rotation will be made through .
So square has symmetry when
(i) Rotated through in anticlockwise or clockwise direction about centroid.
(ii) Reflected along , axis and along Main and secondary diagonals.

2 1 2 1

Rotation through

3 4 3 4

2 1 1 4

Rotation through

3 4 2 3
P a g e | 15

2 1 4 3

Rotation through 4
3

3 4 1 2

2 1 3 2

Rotation through 270o

3 4 4 1

2 1 3 4

Reflection along

3 4 2 1
P a g e | 16

2 1 1 2

Reflection along

3 4 4 3

2 1 2 3

Reflection along main diagonal

3 4 1 4

2 1 4 1

Reflection along 2nd diagonal

3 4 3 2

So, from symmetries of square we have, and


are the distinct elements. Also and are not equal as
so the given group is non- abelian.

Here the operation is Composition, I will denote it here by “.”


P a g e | 17

Group of residue classes under addition:


Consider a positive integer and let
̅ ̅ ̅ ̅ ̅̅̅̅̅̅̅} which is the set of remainders when every integer is divided by

Define addition in as follows : for ̅ , ̅ we put ̅ + ̅ ̅ where is the remainder


obtained after dividing ordinary sum of and by . is an abelian group of order

(Infact this is the mostly used family of abelian groups)

Example:
Take then we have, ̅ ̅ ̅ ̅ ̅ ̅ } is an abelian group of order 6.The Cayle‟s table
is as follows:

̅ ̅ ̅ ̅ ̅ ̅
̅ ̅ ̅ ̅ ̅ ̅ ̅
̅̅̅ ̅ ̅ ̅ ̅ ̅ ̅
̅ ̅ ̅ ̅ ̅ ̅ ̅
̅ ̅ ̅ ̅ ̅ ̅ ̅
̅ ̅ ̅ ̅ ̅ ̅ ̅
̅ ̅ ̅ ̅ ̅ ̅ ̅

Group of residue classes under multiplication:


Consider a positive integer and let

{̅ ̅ } named as unitary groups. (Group of units of ring


P a g e | 18

Where the multiplication is defined as for ̅ ̅ we put ̅ ̅ ̅ where is the remainder


obtained after dividing ordinary product of and by .

is an abelian group of order .

where denotes the Euler‟s Phi function.

Example:
Take then } is an abelian group of order 4.

Point to remember:
is isomorphic to .

Two groups are isomorphic (same) iff both have same order structure.

Its Cayle‟s table is given below

Group of Matrices:
All matrices of order forms an abelian group under addition.

General Linear Group:


[ ] | | } is a non abelian group under matrix
multiplication. The order of depends upon cardinality of .

when then | | | |

If is an infinite field then | | is also infinite.

 If is of finite cardinalty with | | then


| |=(

Now, we discuss some examples of general linear group of finite order.


P a g e | 19

Examples:
Take = , and then | |= | |

,* + * + * + * + * + * +-

,* + * + * + * + * + * + * + * + * + * +

* + * + * + * + * + * + * + * + * + * + * + * + * +

* + * + * + * + * + * + * + * + * + * + * + * + * +

* + * + * + * + * + * + * + * + * + * + * + * +-

Interestingly is isomorphic to .

Special Linear Group:


[ ] | | } is a non abelian group under matrix
multiplication. The order of depends upon cardinality of . When then

| | irrespective of cardinality of , [ ]}

If is an infinite field then | | is also infinite.

 If is of finite cardinalty with | | then


| |=

Now, we discuss some examples of Special Linear Group of finite order.

,* + * + * + * + * + * +-

,* + * + * + * + * + * + * + * + * + * +

* + * + * + * + * + * + * + * + * + * + * +

* + * + * +-
P a g e | 20

Group of Quaternions:
W.R Hamilton discovered the real Hamilton Quaternions

where are real numbers} where is an abelian group under


component wise addition.

Also Hamilton made a collection of 8 elements from the set which is


} where the symbols satisfying the relations
and . is a non-abelian group of order

Generalized Quaternions Group:


For any positive integer , there is a group of order with following presentation.

So for ,

Group of roots of Unity:


For all integer the set of complex th roots of unity
+ | }

(complex zeros of is a group under multiplication of complex numbers.

Example:
For the complex roots of are .We have }
P a g e | 21

Symmetric Group :
Let } then set of all permutations on is called the symmetric group of degree
denoted by . Elements of are of the form

( ) where is permutation on set A.

The order of is

It is a non-abelian group for all For it is an abelian group of order and

} }

Examples:
1. Let us take } then all permutations on set A are

( ), =( ) , =( ) , ( )

( ), = ( ) then the group } is non-abelian as

.The relation is used to compute other products.

(Convention for permutation multiplication is left-to-right).

2. Now, take } then all permutations on set are

Alternating Group
Let } then the set of all even permutations on is called Alternating Group of
degree denoted by
P a g e | 22

The order of Alternating Group of degree is .

It is a non-abelian group for all For it is abelian group. }

Example:

{ }

Example:
The set | } is a group under component wise addition
[ ]
P a g e | 23

Q:8 Space under cross product forms a


(a) Monoid (b) Semi-group (c) Group (d) Groupoid

Explanation:
 is closed under cross product.
Take

Associative law does not hold therefore it is not a semi-group.


Option (d) is correct.

Q:9 The inverse of * + in is

(a) * + (b) * + (c) * + (d) * +

Explanation:
[ ]
We know that, inverse of * + is * +
| |

Clearly , * + * + * + * + * +

Option (a) is correct.

Q:10 The set } under multiplication is

(a) Group (b) Abelian group (c) Semi-group (d) None of These

Explanation:
. 1 2 3
1 1 2 3
2 2 0 2
3 3 2 1

Which is not closed under multiplication modulo


Option (d) is correct.
P a g e | 24

Q:11 Which of the following is true?

(a) Set of odd integers forms a group under addition.


(b) Set of all m  n matrices forms a group under matrix multiplication.
(c) The set is a group under addition.
(d) } is a group under modulo multiplication of 20.

Explanation:

 Addition of two odd numbers gives an even number like


 Matrix multiplication is possible for those matrices whose order is same.
 is not a group. Zero is identity and does not contain identity.

4 8 12 16
4 16 12 8 4
8 12 4 16 8
12 8 16 4 12
16 4 8 12 16

16 is identity here.
Inverse of 4 is 4. Inverse of 8 is 12.
Inverse of 12 is 8. Inverse of 16 is 16.
So, option (d) is correct.

Q:12 What is the identity in where ?

(a) (b) (c) (d) does not exist

Explanation:
 If is the identity element then

Option (d) is correct.


P a g e | 25

Q:13 Set of Complex Numbers is ______ under | |

(a) Group (b) Groupoid (c) Monoid (d) Semi group

Explanation:
 Clearly given set is semi group under the given binary operation
 If e is the identity element then | |

So option (c) is not correct

Hence is semi group under given Binary Operation.

Q:14 Which of the following set is group under where

(a) (b) (c) (d)

Explanation:
are not closed under given operation so we have only option (d). Cleary is group
under whereas identity element is .

Option (d) is correct.

Q:15 Which of the following is identity of the group under where on the
set of real numbers?

(a) (b) (c) (d)

Explanation:
Let be identity of group under given binary operation then,

i.e.
Option (c) is correct.
P a g e | 26

Q:16 The inverse of in is ________

(a) (b) (c) (d)

Explanation:
Let be inverse of in then

Or , is inverse of in

Option (c) is correct.

Q:17 Let } be a group under the operation defined by then


solution of in is ______

(a) (b) (c) (d)

Explanation:
First of all, we apply given binary operation on then on .

Option (a) is correct.

Q:18 If then, ____

(a) (b) (c) (d)

Explanation:
We know that, dihedral group is defined as:
⟨ ⟩

Using this notation we find,


Option (b) is correct.
P a g e | 27

Q:19 Let } be a set and then choose the best option.

(a) is not closed under the given binary operation.


(b) Given set is a non-abelian group under given binary operation.
(c) Inverses of all elements does not exist.
(d) Given set is an abelian group under given binary operation.

Explanation:
We check all axioms of group. Clearly, for any two
is closed under given binary operation.
Associative property also holds. for any
We have

Now, we check identity.

Let identity exists and it be Then,

because,

Thus, identity of given set is under given binary operation.


Now, we check inverses. Let be inverse of
Then,
Thus, inverses of elements also exist.
Now, we check either given group is abelian or not.
Let then,
Thus, given set is an abelian group under given binary operation.
Option (d) is correct.

Q:20 Solution of in is _______

(a) (b) (c) (d)

Explanation:

Option (c) is correct.


P a g e | 28

Answers:

Q1 d Q2 d Q3 d Q4 c Q5 b

Q6 a Q7 c Q8 d Q9 a Q10 d

Q11 d Q12 d Q13 d Q14 d Q15 c

Q16 c Q17 a Q18 b Q19 d Q20 c


P a g e | 29

Properties of Groups
Remarks:
Let be a group then following identities hold:

1. is unique in G.
2. Inverse of each element is unique.
3. =
4. (Reversal law of inverse or Socks-Shoes Property)
5. (Left Cancellation law)
6. (Right Cancellation law)

Idempotent element:
An element is called idempotent element if

Results:
1. A non-empty set together with a binary operation is a group if
, and have solutions in
2. Cancellation law may not hold in a semi group like . Here but
3. A finite semi group in which cancellation law holds is a group.
4. An infinite semi group in which cancellation holds but not a group like .
5. A finite semi group in which cross cancelation law holds is an abelian group.
6. If is an abelian group then .
7. If is a group in which for three consecutive integers and then
is abelian.
8. If then is an abelian group.
9. Any group has only one idempotent element which is the identity element.
 Let be any group and then following hold





P a g e | 30

Order of an element:
Let be a group , if there exist a least positive integer “ for which then we say
is the order of . We denote by | | . If there is no such exists then the
element is of infinite order.

The order of identity element is and it is the unique element of order in any group

Example: 1
Let then group is of infinite order and there are two elements of finite order in One
is the identity element and other is whose order is 2.

Example: 2
Let then group is of infinite order and only element of finite order is identity element.

As there is no such “ exists for 1 for which so is of infinite order, in a similar


fashion, we can check all other elements.

Example: 3
Let } then group is of order and each non-identity elements is of order 2 as

Example: 4
} then group is of order and and

Example: 5
In Dihedral group order of group is

(i) Number of elements of order ={


(ii) If divides “ ” and then number of elements of order are where
denotes the Euler Phi function.

Example: 6
̅ ̅ ̅ ̅ ̅ ̅ ̅ } then order of group is and order of each non-identity element is
P a g e | 31

Example: 7
} then order of group is and so

so thus in each non-identity element


is of order

 if | | is finite then order of each element is finite.


 If order of each element is finite then there are two possibilities
1. is a finite group like
2. is an infinite group like the Quotient group discussion will be in section of
Quotient group).

Torsion Element:
If order of an element in a group is finite then, is called torsion element.

Involution:
An element of order in a group is called involution.

Periodic Group or Torsion Group:


If each element of is of finite order, then is a periodic group.

All finite groups are periodic also there exists infinite group in which order of each element is
finite the group is periodic.

Torsion free Group or periodic groups:


A group in which every non-identity element is of infinite order is called Locally infinite/

- periodic or Torsion free group.

are torsion free groups.

Mixed Group:
An infinite group in which non-identity elements of finite order are present as well as elements of
infinite order is called a mixed group.

are mixed group as in a non-identity element is of order .

Similarly has only two elements of order .


P a g e | 32

Results:
Let be a group and then

1. If | | and then divides


2. Order of an element and its inverse are always equal. | | | |
3. For two elements, | | | |
4. For two elements, | | | |
5. ,
6. If | | then if and only if
7. Let | | | | and then | | | | | |
8. If there are two elements of finite order then need not to be of finite order.
9. If are of finite order and commute then, | | divides the | || |
10. If | | then, | |
11. If | | then can be written in a unique way as product of two
commuting elements of order and
12. If are of finite order and commute. Let | || | and | || |
then | |
13. If an element is of unique order, then either | |
14. A group of even order contains odd number of involutions.
15. If all non-identity elements of a group are involutions then group is abelian.
16. If then
17. If where is an odd number then
18. If is an odd number then is an odd number.

Example:

* +, * + then | | | | but * + is of infinite order.


P a g e | 33

Q:1 Let be a finite group and then if and only if

(a) (b)

(c) (d) None of these

Explanation:
By a theorem if | | then if and only if is a multiple of
i.e.
So, option (b) is true.

Q:2 Which of the following is not a Periodic Group?

(a) (b) (c) (d)

Explanation:
Every finite group is periodic so (b) and (c) are not true options also option (a) is an
infinite periodic group. Hence (d) is true option as every non-identity element is of
infinite order

Q:3 Which of the following group is infinite non-abelian group?

(a) (b) (c) (d)

Explanation:
We know that is an infinite abelian group under addition.
is finite non-abelian group of order is a finite non-abelian group of
order is infinite non-abelian group as is
infinite. To see non-commutative property, we give an example.
Take * + * + * +* + * +
But * +* + * +
Option (b) is correct.
P a g e | 34

Q:4 The number of elements of finite order in are

(a) (b) all elements

(c) (d) all elements except identity are of infinite order

Explanation:
In given group, the non-identity element -1 is of order 2 so there are exactly two elements
of finite order hence option (c) is true.

Q:5 Which of the following is a mixed group?

(a) (b) (c) (d)

Explanation:
An infinite group is mixed if it has non-identity elements of finite order as well
elements of infinite order.

Every finite group is periodic and an infinite periodic group so only true option is (d)

As the group has a non-identity element of finite order so it is a mixed group.

Q:6 The inverse of in is _____

(a) (b) (c) (d)

Explanation:
(123) is a 3-length cycle and is of order 3 so its inverse must be of order 3 and also a cycle
of length 3, whereas the 3-length cycles in are (123) and (132) so inverse of (123) is
(132).

Hence option (b) is correct.


P a g e | 35

Q:7 Number of elements of order in are ______

(a) (b) (c) (d)

Explanation:
Number of elements of order in are where denotes the Euler-phi
function.
So, here we will calculate which is As if is prime.
So option (d) is true. Also every non-identity element in is of order

Q:8 A group in which each element has finite order is known as ______

(a) locally infinite group (b) Periodic group

(c) Mixed group (d) Torsion free group

Explanation:
A group in which every element is of finite order is known as Periodic or Torsion group.
Option (b) is true.

Q:9 Every group has _________ idempotent elements

(a) only (b) exactly (c) more than (d) None of these

Explanation:
The only idempotent element in a group is the identity element.
So, option (a) is true.

Q:10 The inverse of in is _______

(a) (b) (c) (d)

Explanation:
} and inverse of is as
Hence option (b) is true.
P a g e | 36

Q:11 Let | | and such that then | | ________

(a) (b) (c) (d)

Explanation:
Consider

Similarly, and

Since, is prime so, either | | or | | as is non-identity element.


So | |
Hence option (a) is true

Q:12 Let then | | _______

(a) (b) (c) (d)

Explanation:
| |
As we have a result | |
| |
here, and | |

So, substituting values we get, | |

Option (c) is correct.

Q:13 What is the order of in ?

(a) (b) (c) (d)

Explanation:
As so, is of order

Hence, option (b) is true.


P a g e | 37

Q:14 The number of elements of order in are ______

(a) (b) (c) (d)

Explanation:
The number of elements of odd prime order in any symmetric group
number of distinct cycles of length = here so,
Hence, option (a) is true.

Q:15 Maximum order of any element in is ______

(a) (b) (c) (d)

Explanation:
To find maximum order of any element in

We write positive integer partitions of , as


Choose such partition for which the L.C.M is maximum.
Here and l.c.m
So, maximum order of any element in is Hence, option (d) is true.

Q:16 ______

(a) (b) (c) (d)

Explanation:
The elements of are } clearly it is
So option (a) is correct.
P a g e | 38

Q:17 Let be the order of an element of a group Then which of the following elements
of has order different from ?

(a) (c)
(b) where (d) where

Explanation:
By properties of order of an element | | | | where is any element of , an
| |
element and have same order, Also | |
| |
as | | so | | .

Thus option (d) is true.

Q:18 For two commutative elements | || | and | || |


then | | _____

d m
(a) (b) (c) (d) None of these
m d
Explanation:
| | | | | | | | | | | |
| | | |
= similarly, | | | |
= as | | | | so,

| | | || | =
So, option (b) is true.

Q:19 If there is only one element of some specific order, then ______

(a) either a  e or a  2 (c) either a  e or a  2


(b) either a  e or a  2 (d) neither a  e nor a  2

Explanation:
Here, we have to find a unique element of some specific order in a group .
As we know identity element is of order and no other element of a group is
of order .
So, identity element is unique element of order also | | | | and we have to tell
that this element is unique of that specific order so only possibility from here is | | .

So, option (a) is true.


P a g e | 39

Q:20 Number of involutions in is/are ______

(a) (b) (c) (d)

Explanation:
We know that, an element of order in a group is called involution.
}

since, order of is and order of is


So in there is only one involution.
Option (b) is correct.

Q:21 If is only element of order in a group and is any other element of then

(a) (b) (c) (d)

Explanation:
Given that is an element of order and is any other element of
as | | | |
But is only element of order So,
Option (a) is correct.

Q:22 Number of involutions in is/are ______

(a) (b) (c) (d)

Explanation:
Number of elements of order in if d divides , For
Option (b) is correct.

Q:23 Order of a reflection in is ______

(a) (b) (c) (d)


P a g e | 40

Explanation:
Order of any reflection in dihedral group is
Option (d) is correct.

Q:24 Let be elements of a group | | then | | _______

(a) (b) (c) (d)

Explanation:
Given that, and

Thus,
Option (c) is correct.

Q:25 Let be an element of a group of order For which smallest positive integer
?

(a) (b) (c) (d)

Explanation:
We know that if and then divides
Since, for does not divide
For does not divide For divides
Thus, is the smallest positive integer for which
Option (d) is correct.

Q:26 If where then, which of the following is also of


order
?
(a) (b) (c) (d)
P a g e | 41

Explanation:
We know that, if then
Using this result, we find order of each element in given options.

Option (b) is correct.

Q:27 Let be a group and for then there exists

(a) (b) (c) (d)

Explanation:
In a group of odd order, every element can be written as square of some element of
group.
Since,

Here, we suppose which is clearly an element of group.


Option (b) is correct.

Q:28 Suppose are non-identity elements of a group such that then


____

(a) (b) (c) (d)

Explanation:
Given that
then

Thus,
Option (b) is correct.
P a g e | 42

Q:29 If and then, _______


(a) (b) (c) (d)

Explanation:
Given that
Then
Option (c) is correct.

Q:30 If then _____

(a) (b) (c) (d)

Explanation:
We know that, for
,
Using these properties of group elements,
We have
Because,
Thus,
Option (d) is correct.

Answers:

Q1 b Q2 d Q3 b Q4 c Q5 d
Q6 b Q7 d Q8 b Q9 a Q10 b
Q11 a Q12 c Q13 b Q14 a Q15 d
Q16 a Q17 d Q18 b Q19 a Q20 b
Q21 a Q22 b Q23 d Q24 c Q25 d
Q26 b Q27 b Q28 b Q29 c Q30 d
P a g e | 43

Subgroups
Definition:
Let be a group, then is subgroup of if is group. If is a
subgroup of we denote it mathematically as

Criteria:
Let then is subgroup of if and only if where may be
of any order. finite or infinite but in case if is finite it is enough only to check closure
property in .

For Addition:
if is group

Trivial Subgroup:
For a group } is called trivial subgroup of

Non-Trivial Subgroup:
Expect } remaining all subgroups are called non-trivial subgroups.

Improper Subgroup:
A group is subgroup of itself, known as improper subgroup of

Proper Subgroup:
For a group all subgroups of are its proper subgroups except

Examples:
1. is a proper subgroup of
2. is a proper subgroup of
3. is a proper subgroup of
4. is a proper subgroup of
5. .
P a g e | 44

Before finding subgroups of finite groups , we need to know some results.


(1) For a prime there is a unique group of order which is upto isomorphism
(2) If is prime then there are two groups of order upto isomorphism and the groups are
and .
(3) There are two groups of order and upto isomorphism.
(4) The order of subgroups divides the order of group. Also its contrapositive , if does not
divide | | then does not have a subgroup of order .
6. } then has order while are of order .
Since are the only positive divisors of so subgroups of these orders can exist.
The subgroup of order will always be unique as element of order is the only identity
element. The subgroup of order will have an element of order alongwith identity since
is of order so } is subgroup of order . The subgroup of order is the group itself. So
all of its subgroups are } }

7. } while each non-identity element is of order . Also subgroup of order


can exist. Since any subgroup of order will have an element of order alongwith identity so
there are three subgroups of order as are of order .
Hence all subgroups are } } } } }

8. } are of order while are of order . Since there are


elements of order so there are three subgroups of order . For a subgroup of order we
need identity and two elements of order as there are two elements of order and }
satisfies closure law so it is a unique subgroup of order .
Hence its all subgroups are } } } } }

9. } has a elements of order so there


are five subgroups of order . Its subgroups of order will be of two kinds ,
(i) has two elements of order so choose an element of order from such that
the resulting set satisfies closure law.
If we take } then similarly taking or the
resulting set is not closed. So take } which is closed. Hence it is a
subgroup of order which is isomorphic to
(ii) Take three elements of order alongwith identity and check the closure law.
} } these are subgroups isomorphic to .
So all subgroups are
} } } } } } } } }
In general, must be subgroups of

10. } has a unique element of order while elements of order


So a unique subgroup of order exists. i:e }
For a subgroup of order we need to take two elements of order such that both are inverses
of each other along with an element of order and identity, since there are elements of
order and there are three pairs so three subgroups exists of order which are
} } }
its all subgroups are } } } } }
P a g e | 45

11. } | | }
12. { √ }
13. ,* + -
14. } it‟s all subgroups are } } } }

15. } its all subgroups are


} } } } } }
} } }
}
}
16. The set of all even permutation is a group called alternating group which is proper
subgroup of symmetric group In example is proper subgroup of For we
have a group

} it‟s all non-trivial subgroups are


Subgroups of order , (Isomorphic to
} } } } } } }
} }
Subgroups of order (Isomorphic to
} } } }
Subgroups of order
} (Isomorphic to Klien-4)
While other subgroups of order 4 are isomorphic to
}
} }
Subgroup of order (Isomorphic to
} }
} }
Subgroups of order (Isomorphic to
}
}
}

Subgroup of order (

17. In general, we can write it as


where is any field.
18. ,* + * + * + * + * + * +- is improper subgroup of
P a g e | 46

19. ,* + * + * + * + * + * + * + * + * +

* + * + * + * + * + * + * + * + * + * + * +,

* + * + * + * + is a subgroup of

20. ,* + -

Important Points:

1. If and are subgroups of then need not to be a subgroup of


Example:
} and } }
}
} it is not closed under multiplication of elements
2. is a subgroup of iff either is contained in or is contained in .
3. is always a subgroup of , moreover the intersection of any collection of subgroups
of a group is a subgroup of
4. An infinite group has infinite subgroups.
5. If and then
6. For each ⟨ ⟩
Example:
Take then }
7. If | | | | then } } | || |
Example:
Take } } } then } but their orders
are not relatively prime.
8. is a subgroup of iff then
9. If is a finite group of order then has only one subgroup of order .
Example:
| |
Order of is product of two distinct primes.
}. has only one subgroup of order which is {e, a, a2}
10. If is a finite group of order . Then has exactly one subgroup of order .

Example:
 In S3 , | | . has exactly one subgroup of order

| | | | D5  {e, a, a 2 , a 3 , a 4 , b, ab, a 2b, a 3b, a 4b} and H  {e, a, a2 , a3 , a 4}
only subgroup of order 5.
11. If are subgroups of a finite group , such that
√ , √ then
Example:
}
P a g e | 47

| | √
}
}
So
12. cannot have a subgroup of order where | |
If then for there is an obvious subgroup of order one identity subgroup.
13. Let be an abelian group, then | | } is always a subgroup of
( Periodic or Torsion subgroup of
14. Let A be any group then for
} is the cyclic subgroup of generated by .
15. If then
16. {x  Dn | x  e} is not a subgroup of
2

17. Number of subgroups of


18. Number of subgroups of

∏ , ∏

where
P a g e | 48

Q:1 Let be an Abelian group and be a set of all elements of finite order in . Then
is _______

(a) Only a subset of (b) Abelian subgroup of

(c) Subgroup of (d) None of these

Explanation:
Let be an abelian group then | | } is always a subgroup of .
Here group is abelian therefore every subgroup is abelian.
So option is correct.

Q:2 } is a subgroup of _______

(a) (b) (c) (d)

Explanation:
It is not subgroup of .
} has no element of order
But given group has two elements of order
} is a subset of .
But binary operation is not same.
} is a group under multiplication.
} is not subgroup of
} is itself . Option (a) is correct.

Q:3 Choose the option for subgroups.

(a) (b) (c) (d)

Explanation:
is neither a subset of nor of
not possible.
}
}
Subgroups of are of the type . etc.
Option (c) is correct
P a g e | 49

Q:4 Total number of subgroups of .

(a)  (n)   (n) (b)  (n)   (n)


(c)  (n)   (n) (d)  (n) (n)   (n)

Explanation:
 (n)  Number of positive divisors of n
 (n)  Sum of all positive divisors of n
Option (a) is correct
Example:
D5  10 , n  5
Positive divisors of 5 = 1, 5
 ( n)  2
 ( n)  1  5  6
 ( n )   ( n)  2  6  8 .

Q:5 Let be two distinct subgroups, each of order 2, of a finite group . Let be
the smallest subgroup containing and then | | ________

(a) Always 2 (b) Always 4


(c) Always 8 (d) None of these

Explanation:
Take
Both H1 and H 2 are distinct and of order 2.
Option (d) is correct.

Q:6 Let and be finite subgroups of a group . Then

O ( H ).O ( K )
(a) O( H )  O( K ) (c)
O( H  K )
O ( H ).O ( K )
(b) O( H ).O( K ) (d)
O( H  K )
Explanation:
For two finite subgroups of a group
Order of .
Option (c) is correct.
P a g e | 50

Q:7 If and are subgroups of a group with | | and | | , then which of


the following may be the order of ?

(a) 3 (b) 12 (c) 4 (d) 16

Explanation:
H  K  H and H  K  K
Order of always divide the order of and .
Because is subgroup of both and .
H  K  12,16
of 12,16 is 4.
H  K =1,2,4
Option (c) is correct.

Q:8 No. of subgroups of is __________

(a) 8 (b) 9 (c) 10 (d) 11

Explanation:
Given group is D4
Positive divisors of 4 are 1,2,4
 ( n)  3 ,  ( n )  1  2  4  7
 (n)   (n)  3  7  10 .
Option (c) is correct

Q:9 Let be group elements, if | | and | | , then | | ?

(a) 1 (b) 2 (c) 1 or 2 (d) 4

Explanation:
Since | |=24 so the subgroup generated by is of order 24.
Also | | 10 so subgroup generated by is of order 10.
|⟨ ⟩ ⟨ ⟩| divide 24.
of 24,10 is 2
divisors of 2 is 1,2.
Option (c) is correct
P a g e | 51

Q:10 Let be a group, which of the following is not a subgroup of ?

(a) } where is fixed element of .


(b) } where is a subgroup of .
(c) } where is identity of .
(d) } where are subgroups of

Explanation:
In option (d) it is the intersection of two subgroups which is again subgroup
In option (c) it is given that all elements whose square is identity form a subgroup
But it is not always true. For justification we give a counter example.
Counter Example:
Consider
}
It is not subgroup of because it is not closed.
For option (a) and (b) , we can check using subgroup test.
It is not necessary for to be subgroup. It may be any subset of group.
Option (c) is correct.

Q:11 Order of sub group generated by 7 in _______

(a) 4 (b) 3 (c) 5 (d) 1

Explanation:
}
We will find order of element , as
So the subgroup generated by is of order 4 .
The subgroups is }
Option (a) is correct.

Q:12 Which of the following is subgroup corresponding to given group?

(a) ,( ) -

(b) ,( ) -
(c) }
(d) All of these
P a g e | 52

Explanation:
We use subgroup criteria to check whether given set is subgroup or not.
For option (a), take ( ) ( )

( ),

( ) ( )

( )

( )

For option (b), take ( ) ( ) ( )

( )( ) ( )
For option (c), take then

Option (d) is correct.

Q:13 Choose the best option.

(a) Every non-trivial subgroup of non-abelian group is non-abelian.


(b) There may exist group of infinite order in which every non-trivial subgroup is infinite.
(c) Every proper subgroup of an infinite group can never be finite.
(d) None of these

Explanation:
Every non-trivial subgroup of non-abelian group need not to be non-abelian.
For justification, we consider counter example of non-abelian group of order
} it‟s all non-trivial subgroups are abelian.
There may exist an infinite group whose every non-trivial subgroup is infinite.
For example, group of integers. It‟s all non-trivial subgroups are infinite which are of the
form
There may exist an infinite group whose all proper subgroups are finite. For example
Option (b) is correct.
P a g e | 53

Q:14 Number of subgroups of ________

(a) (b) (c) (d)

Explanation:
We know that number of subgroups of

Positive divisors of .
So, number of subgroups of
Option (b) is correct.

Q:15 Let then _______

(a) (b) (c) (d)

Explanation:
We know that for two subgroups, if then }
Here, }
Option (c) is correct.

Q:16 The “subgroup property’’ is __________

(a) Reflexive (b) Symmetric (c) Transitive (d) Both a & c

Explanation:
Let be a group. be its subgroups. Since, a group is always its subgroup.
So, property of “being subgroup‟‟ is reflexive.
Property of “being subgroup‟‟ is not symmetric.
Take } } } but
Property of “being subgroup‟‟ is transitive. If then
Option (d) is correct.

Q:17 Number of subgroups of order in a group is/are ________

(a) (b) (c) (d)


P a g e | 54

Explanation:
We know that }.
Now subgroups of order are
} } }
Option (b) is correct.

Q:18 Number of subgroups isomorphic to Klein group in are _______

(a) (b) (c) (d)

Explanation:
Take }
Now subgroups of order , isomorphic to Klein group are
} }
Option (b) is correct.

Q:19 Which of the following is not a subgroup of ?

(a) (b) (c) (d)

Explanation:
is not closed under addition as but

While remaining are subgroups of as

Q:20 The number of subgroups order in

(a) (b) (c) (d)


P a g e | 55

Explanation:
Since has no element of order so it will not have an abelian subgroup of order .
And number of subgroups in will be . Here so number of
subgroups will be

Answers:

Q1 b Q2 a Q3 c Q4 a Q5 d

Q6 c Q7 c Q8 c Q9 c Q10 c

Q11 a Q12 d Q13 b Q14 b Q15 c

Q16 d Q17 b Q18 b Q19 d Q20 b


P a g e | 56

Cyclic Groups
Definition:
A group is said to be cyclic if it can be generated by a single element there is some

element such that

} Under usual multiplication

} Under usual addition

In both cases we simply can write it as ⟨ ⟩ and read as is generated by or is

generator of

In other words, every element of is a power of

 Be careful about the phrase “A group generated by a single element‟‟ it does not mean
that group has only one generator. All elements of group must be generated by at least
one element. A cyclic group can have more than one generators.
Some famous cyclic groups are given below.

Examples:
1. Simplest example of infinite cyclic group is set of integers under addition with two
generators
2. Set of all nth roots of unity } is finite cyclic group
under multiplication.
3. Additive group is a finite cyclic group generated by residue class
4. Groups of order are all cyclic.
5. Group of units is cyclic group for all prime numbers.
6. Group of units is cyclic for all odd prime numbers.
2
7. Cube roots of unity } is cyclic group of order under usual multiplication.
8. Fourth roots of unity } is a cyclic group of order under usual
multiplication.
P a g e | 57

Results:
1. For each positive integer there is a unique cyclic group of order
2. An infinite cyclic group has exactly two distinct generators and .
3. A finite cyclic group of order has generators, where is Euler-phi function
and defined as: ( )( )( ) ( )

where

4. Number of generators of a cyclic group of order are always even.

5. Number of generators of ( ) if is cyclic.

6. Any infinite cyclic group is isomorphic to


7. Any finite cyclic group of order is isomorphic to
8. Every cyclic group is abelian. Converse need not to be ture.
9. If is generator of a cyclic group of order then is also generator of if and only
if
10. Specifically, is generator of
11. If | | then is cyclic iff there exist some such that | |
12. Every subgroup of cyclic group is cyclic.
13. For a group of order if is divisor of then has a unique subgroup of order
14. Every group of prime order is cyclic. Converse need not to true.
15. If is a group of order | | where does not divide also
are distinct primes then is cyclic group of order No other group exists of order if
exists it will be isomorphic to this cyclic group. Group of order is cyclic.
16. Number of subgroups of a cyclic group where is totient function

where

17. Number of cyclic subgroups of order , where is divisor of


order of group.
18. Number of elements of order of a finite cyclic group of order
19. Number of cyclic subgroups of
20. For an odd prime ( ) is cyclic group.
P a g e | 58

21. If is prime then all proper subgroups of are cyclic.

22. There are two groups of order (upto-isomorphsim) which are and , is prime.

23. Every group of order is abelian.


24. If is cyclic group of order generated by and is its subgroup of order then
| |

25. For is non-cyclic.


26. For a cyclic group }, generator of subgroup ⟨ ⟩ ⟨ ⟩ is
⟨ ⟩ [ ] ⟨ ⟩ ⟨ ⟩ ⟨ ⟩

27. No of cyclic subgroups of order in {

28. A cyclic group of even order has unique element of order


29. A cyclic group of order has exactly three subgroups.
30. For a prime number a group having more than elements of order can not be
cyclic.
31. A finite cyclic group cannot be written as a union of its proper subgroups.
32. Every element of cyclic group of order is a prime number can be written as

How to apply above results?


1. To find number of generators of we evaluate . For this we do prime
factorization of
( )( )
Number of generators of
2. Take , we find all generators of group. For this first we find those numbers that
are co-prime to Numbers co-prime to are
⟨ ⟩
3. Let }, we find cyclic subgroups of .
⟨ ⟩ } ⟨ ⟩ } ⟨ ⟩ } ⟨ ⟩ }
⟨ ⟩ } ⟨ ⟩ }
Number of cyclic subgroups of are
4. A group of order is only cyclic group as does not divide
5. For a prime ( ) .
6.
P a g e | 59

7. For a cyclic group of order ⟨ ⟩ we find all of its subgroups


corresponding to each divisor of
⟨ ⟩ }
⟨ ⟩ }
⟨ ⟩ }
⟨ ⟩ } }
⟨ ⟩ }
⟨ ⟩ }
⟨ ⟩ }
⟨ ⟩ }
⟨ ⟩ }
⟨ ⟩ }
⟨ ⟩ }
⟨ ⟩ }
8. For a cyclic group of order we find order of some elements
| | | | | |
 For a cyclic group we find all of its subgroups corresponding to each of its divisor
subgroups generated by these divisors are:
⟨ ⟩ }
⟨ ⟩ }
⟨ ⟩ }
⟨ ⟩ }
⟨ ⟩ }
⟨ ⟩ }
 Elements of the cyclic group of order can be written as is
} }

 Consider a cyclic group of order We see that it cannot be written as a union of its
proper subgroups. } } }

Some useful facts:


1. If all proper subgroups of a group are abelian then group itself need not to be cyclic.
Similarly true for all abelian groups all proper subgroups of an abelian group does not
guarantee that group is abelian.
Klien-4 group of order has all cyclic subgroups but itself is non-cyclic because
} cannot be generated by a single element but all of its proper subgroups
are } } } } are cyclic.
Other reason is that there does not exist an element of order
P a g e | 60

Further counter examples are . All of these groups are non-abelian but all of
their proper subgroups are cyclic obviously abelian
} ⟨ ⟩,
its proper subgroups are cyclic as:
⟨ ⟩ } ⟨ ⟩ }
} } } } }
2. is not cyclic . For } is not cyclic but
direct product of cyclic groups.
3. are not cyclic groups.
4. To check whether the given group of order is cyclic or not, we can check order of
elements if there exists an element of order then group must be cyclic.
Contrapositively, if there is no element of order then group will be non-cyclic.
5. A subgroup of an abelian group need not to be cyclic.
is not cyclic subgroup of the abelian group
P a g e | 61

Q:1 Which of the following is non-cyclic group?

(a) (b) (c) (d)

Explanation:
Using result for group of units

which is not cyclic but the direct product of cyclic groups.


By using result, is
cyclic if and only if
which is cyclic.
( ) , using this result , which is cyclic
For each positive integer there is a cyclic group .
Option is correct.

Q:2 If | | is cyclic group then number of subgroups of are _____

(a) (b) (c) (d)

Explanation:
Number of subgroups of a cyclic group of order
For
Option is correct.

Q:3 The generating set of is _______

(a) } (b) }
(c) } (d) }

Explanation:
We know that if is generator of a group of order then also generates the group if
and only if and are relatively co-prime to each other.
Numbers co-prime to are .
So the generating set of the given
group is }.
Option is correct.
P a g e | 62

Q:4 Point out the number corresponding to which each group is cyclic?

(a) (b) (c) (d)

Explanation:
Using the fact that “a group of order where are distinct prime numbers and

is only cyclic if does not divide


But if divides then an additional group of the given order also exists which
is non-abelian.
Also every group of order is abelian, one is cyclic and other is non-cyclic
abelian,
where is a prime number.
since divides so every group of order is not cyclic.
one is cyclic and other is non-cyclic group corresponding to this number.
again corresponding to this number each group is not cyclic.
since does not divide
So So only cyclic group corresponding to
Option is correct.

Q:5 The order of generators of a group of order is are ______

(a) (b) (c) (d)

Explanation:
First of all we check how many types of groups are of order then we find the order
of generators of group.
Since does not divide .
So only cyclic group exists of order and we know that order of generator of a cyclic
group is equal to order of the group.
So |⟨ ⟩|
Option is correct.

Q:6 Which of the following is are cyclic subgroups of ?

(a) (b) (c) neither nor (d) both


P a g e | 63

Explanation:
Since is a cyclic group for each positive integer
So corresponding to each divisor of
there must exist a cyclic subgroup of }
Divisors of are
So the subgroups corresponding to these numbers are as:
} , } , }
}=
}
} ,
Option is correct.

Q:7 Choose the correct statement.

(a) Every abelian group of finite order is cyclic.


(b) Every group of order is abelian.
(c) There must exists a non-cyclic group of order
(d) is not a subgroup of a cyclic group of order

Explanation:
Every finite abelian group need not to be cyclic. is a finite abelian group but not
cyclic as there does not exist an element of order
A group of prime order is always cyclic hence abelian.
So a group of order is cyclic hence abelian.
does not divide Only cyclic group exists of order
A cyclic group of order must have a subgroup corresponding to its divisor which
will be isomorphic to }
Option is correct.

Q:8 Number of cyclic subgroups of a dihedral group of order are _____

(a) (b) (c) (d)

Explanation:
We know that number of cyclic subgroups of are
,

Option is correct.
P a g e | 64

Q:9 Which of the following is/are generators of }?

(a) (b) (c) (d)

Explanation:
The given group is infinite cyclic.
So it must be isomorphic to group of integers having precisely two generators
} , - ⟨ ⟩
Option is correct.

Q:10 Number of generators of are ________

(a) (b) (c) (d)


Explanation:
We know that if is cyclic then number of generators of are ( )
Also we know that is cyclic odd prime numbers.
is cyclic
its number of generators are ( ) ( )
Option is correct.

Q:11 If is a group of order then point out the condition for which must be cyclic?
(a) must contain at least one element of order
(b) must contain elements of order
(c) must have all proper non trivial subgroups of order .
(d) does not contain any element of order

Explanation:
There exists a group of order that contains at least one element of order
but itself non-cyclic }
We know that every group contains a unique element of order
If all remaining elements are of order equal to order of group then group must be of
prime order and hence cyclic.
If a group of order has all proper abelian subgroups of order then group need not to
be cyclic.
has all proper abelian subgroups of order but itself is not
cyclic.
If a group of order contains no element of order then group can never be cyclic.
Option is correct.
P a g e | 65

Q:12 If then |⟨ ⟩| _______

(a) (b) (c) (d)

Explanation:
We know that every element generates cyclic group of order
Since also generates .
So |⟨ ⟩|
Option is correct.

Q:13 For which value } must be a cyclic group?

(a) (b) (c) (d)

Explanation:
We can solve it into two methods.
By using closure property of group
Since group is cyclic so order of group must be equal to order of generator
| | ⟨ ⟩ }

Option is correct.

Q:14 Number of elements of order of a subgroup generated by in are ______

(a) (b) (c) (d)

Explanation:
First of all we find the subgroup generated in then we will find number of
elements of order if exists.

⟨ ⟩ }
A subgroup of order can never has an element of order
Option is correct.
P a g e | 66

Q:15 If is an element of order then ⟨ ⟩ ______

(a) } (b) }
(c) } (d) }

Explanation:
We can find the subgroup generated by as:
⟨ ⟩ }
}
Option is correct.

Q:16 If is a cyclic group having exactly three subgroups, first is identity second is group
itself third one is of order 19. What possible order group can have?

(a) 171 (b) (c) (d) not possible

Explanation:
We know that a cyclic group of order has exactly three subgroups identity, group
itself and a group of order as possible divisors of are
Using this result order of group must be
Option is correct.

Q:17 Number of cyclic subgroups of order in are _______

(a) (b) (c) (d)

Explanation:
We know that }
Since, all reflections are of order
There does not exist an element of order
Also does not divide | |
So no cyclic subgroup of order
Option is correct.

Q:18 Let G be a cyclic group such that G has an element of infinite order then number of
elements of finite order in G is/are

(a) 0 (b) 1 (c) Infinite (d) None of these


P a g e | 67

Explanation:
Since is a cyclic group with an element of infinite order than must be infinite and
will be isomorphic to .
Thus has only one element of finite order (identity element).
Option (b) is correct.

Q:19 Number of elements of order in a cyclic group of order is/are

(a) 41 (b) 44 (c) 40 (d) 1

Explanation:
Number of elements of order in a cyclic group of order is divisor of
Thus, number of elements of order in a cyclic group of order

Option (c) is correct.

Q:20 The order of the subgroup generated by ⟨ ⟩ ⟨ ⟩ in is?

(a) (b) (c) (d)

Explanation:
Actually we have to find the order of generator of ⟨ ⟩ ⟨ ⟩
Or |⟨ ⟩ ⟨ ⟩|
There are two ways to find this order.
We know that ⟨ ⟩ ⟨ ⟩ ⟨ ⟩ ⟨ ⟩ [ ]

⟨ ⟩ ⟨ ⟩ }
|⟨ ⟩ ⟨ ⟩| |⟨ ⟩|
Or | | |⟨ ⟩|
Or first we find ⟨ ⟩ ⟨ ⟩ then ⟨ ⟩ ⟨ ⟩ and finally |⟨ ⟩ ⟨ ⟩|
⟨ ⟩ } }
⟨ ⟩ } }
⟨ ⟩ ⟨ ⟩ }
Option is correct.
P a g e | 68

Q:21 All proper subgroups of ______ are cyclic.

(a) (b) (c) (d)

Explanation:
There exists non-cyclic abelian subgroup of order in
i.e. }
There exists similar non-cyclic subgroup of order in as above mentioned.
There exists non-abelian subgroup of order in }
All proper subgroups of are cyclic.
Option (a) is correct.

Q:22 Point out pair of groups which have same number of cyclic subgroups.

(a) (b) (c) (d)

Explanation:
Cyclic subgroups of are of order
Number of elements of order are so there are cyclic subgroups of order
Number of elements of order are so there are cyclic subgroups of order
Number of elements of order are so there are cyclic subgroups of order
So total cyclic subgroups are
Number of cyclic subgroups of are
Number of Cyclic subgroups of are Cyclic subgroups has subgroups.
Cyclic subgroups of are of order . has elements of order so there are
cyclic subgroups of order While cyclic subgroups of order so there are total
cyclic subgroups.
Cyclic subgroups of are ,* +-

,* + * + * +- ,* + * +- ,* + * +- ,* + * +-
which are in numbers. So, both and have same number of cyclic
subgroups.
Cyclic subgroups of are in numbers.
Cyclic subgroups of are in numbers.
Option (d) is correct.
P a g e | 69

Q:23 If are generators of a cyclic group of order then must be a/an

(a) Odd number (b) Even number (c) Prime number (d) Composite number

Explanation:
If both are generators of a cyclic group of order then
| |
| |
which is possible only if is an odd number.
Option (a) is correct.

Q:24 Which of the following is correct statement?

(a) All proper subgroups of a non-abelian group may be cyclic.


(b) is generated by }
(c) has only one generator.
(d) All proper subgroups of a non-abelian group must be non-abelian.

Explanation:
All proper subgroups of a non-abelian group need not to be non-abelian.
For example, all proper subgroups of are cyclic.
Number of generators of are .
Generators of are } }
Option (a) is correct.

Q:25 Maximum order of an element in ______

(a) (b) (c) (d)

Explanation:
We know that ( ) is cyclic for all odd primes. So,
Since, maximum order of an element in a cyclic group is the order of group.
So, maximum order of an element in is
Option (d) is correct.
P a g e | 70

Answers:

Q1 a Q2 b Q3 c Q4 d Q5 c
Q6 b Q7 b Q8 a Q9 d Q10 b
Q11 b Q12 c Q13 b Q14 a Q15 c
Q16 b Q17 a Q18 b Q19 c Q20 d
Q21 a Q22 a Q23 d Q24 a Q25 d
P a g e | 71

Cosets and Lagrange’s Theorem

Cosets:
Let be a group and be any subgroup of then the set } denoted by is
called left coset of in containing Similarly, the set } denoted by is right
coset of in . The element is called Coset representative of . The number of
distinct left or right cosets of in is called index of in and denoted by [ ]

Examples:
Consider } and }
} } } } }
} } } } }
Then there are exactly distinct cosets.
} and } then there are exactly two distinct cosets of in
which are and
} and } then there are exactly two
distinct cosets of in which are

Under Addition:
} and } are the left and right cosets of in

Consider and then there exactly four distinct cosets of in which are

If and then there are exactly two distinct cosets of in which are
is an odd integer.
If } } then there are exactly three distinct cosets of in
which are } }

Index of a Subgroup:
Let be a group and a subgroup of The index of in is the number of distinct
left(right) cosets of in It is denoted by [ ]
| |
For finite group, index is defined as: [ ]
| |
P a g e | 72

Results:
1. Any Two left or right cosets are either identical or disjoint.
2. Any Two left or right cosets have the same cardinality. | | | | | |
3. If is a finite group then number of distinct cosets of any subgroup in is .
4. Cosets form the partition of a group. Union of all cosets is and they are disjoint.
} and For example, cosets of in are
Clearly,
5.
6. where
7.
8. .
9. Let then for is coset of in
10. If then is only coset of which is subgroup of
11. If is abelian then
12. If and for some then
13. There is one-one onto map between set of left cosets and set of right cosets of in
where
14. Let [ ] . If then
15. If [ ] [ ] [ ]
[ ][ ].
16. Let and [ ][ ] then [ ] [ ][ ]
17. Let then [ ] [ ]
18. A subgroup of finite index in an infinite group has non-trivial intersection with every
infinite subgroup of group.
19. An infinite group may have a subgroup with finite index and every subgroup of a finite
group has finite index. For example, there are many infinite subgroups of group of
integers which have finite index.
20. If | | where are distinct primes and | | | |
then [ ]
21. Let be an abelian group. | | | | then there exists a
| |
subgroup of of index
22. Number of distinct left(right) cosets of in
23. Number of distinct left(right) cosets of in
24. Number of distinct left(right) cosets of in
25. Number of distinct left(right) cosets of in ( )
P a g e | 73

Lagrange’s Theorem:(Finite groups)


If is subgroup of a finite group then order of divides order of Moreover, number of
| |
distinct left or right cosets of in is | |
| |
Order of an element an of divides the order of . Moreover,

Converse:
If a positive integer (say) divides the order of then must have a subgroup of order .
But converse of Lagrange‟s theorem does not hold in general, as order of is and divides
but does not have a subgroup of order . (Explanation will be discussed later).
Convers holds in following families of groups:

26. Finite Abelian and Cyclic groups


27. Dihedral groups.

Why does not have a subgroup of order ?


Consider an alternating group of order

List of all subgroups of is already discussed in Subgroup section, so here we will only
discuss about nonexistence of a subgroup of order .

As divides the order of so if a subgroup of order (if exists) will be isomorphic to


either or . (As there are exactly two non-isomorphic groups of order ( is prime), one
is and the other is ). Since there is an element of order in but does not have
an element of order so it is impossible for to have an element of order in
Now, we will discuss whether exists and isomorphic to
} the two length cycles are of order and three
length cycles are of order So, must have elements of order and elements of order
along with the identity element.
Now consider are in and about remaining elements of
order we must choose .
But is not closed under any combination of these elements.
So, does not exist.
P a g e | 74

Q:1 Index of ⟨ ⟩ in is __________

(a) (b) (c) (d)

Explanation:
We know that } and subgroup generated by is
⟨ ⟩
}
}
Since, given group is finite.
| |
So, index of ⟨ ⟩ in is |⟨ ⟩|
Option (d) is correct.

Q:2 Number of right cosets of in are ______

(a) (b) (c) (d)

Explanation:
| |
For a finite group number of left or right cosets of in | |
Since, is a finite group of order and is its subgroup of order
So, number of right cosets of in are
Option (b) is correct.

Q:3 Under what condition, every left coset is right coset?

(a) If is an abelian group.


(b) If is subgroup of index in a finite group
(c) If is a group of order
(d) All of these

Explanation:
Since, in an abelian group all elements of group commute with each other.
So, every element of group must commute with any subgroup of

If is a finite group and is its subgroup of index then every left coset of is
also right coset of in A group of order is only cyclic group.
So, all of its right cosets are left cosets.
Option (d) is correct.
P a g e | 75

Q:4 Number of left cosets of ⟨ ⟩ in _______

(a) (b) (c) (d)

Explanation:
We know that } ⟨ ⟩ }
Number of left cosets of ⟨ ⟩ in are
Option (b) is correct.

Q:5 Which of the following subgroup has infinite index corresponding to given group?

(a) (b)
(c) (d) All of these

Explanation:
Since,
In fact for each rational number, there is a distinct coset.
Since rational numbers are infinite. So, number of distinct cosets are also infinite.
In short index of in is infinite. Similarly, index of in
and in is infinite as:
√ √

Option (d) is correct.

Q:6 In which of the following group, converse of Lagrange’s theorem hold?

(a) (b) (c) (d) All of these

Explanation:

}
Corresponding to each divisor of | | there exists a subgroup
(see in subgroups section)
Also Corresponding to each divisor of | | | | there must exist a subgroup.
Option (d) is correct.
P a g e | 76

Q:7 Let | | then index of ⟨ ⟩ in ⟨ ⟩ is _______

(a) (b) (c) (d)

Explanation:
Since, order of is So, order of group generated by is also
Order of subgroup generated by ⟨ ⟩
Because ⟨ ⟩ } Index of ⟨ ⟩ in ⟨ ⟩ is
Option (b) is correct.

Q:8 Which of the following statement is accurate?

(a) Every subgroup of an infinite group has infinite index.


(b) If order of a subgroup of a group is infinite then its index in is also infinite.
(c) Two subgroups of a group having same index are always isomorphic.
(d) None of these

Explanation:
A subgroup of infinite order of an infinite group need not to have infinite index.
For example if we take then its cosets are
So, index of in is If we take } then index will be infinite.
Two subgroups of a group having same index, need not to be isomorphic.
For example, in there are two subgroups of index but they are non-isomorphic.
One is cyclic subgroup of order } and other is Klein subgroup of
order }
Option (d) is correct.

Q:9 If are subgroups of a group [ ] and [ ] then


[ ] _________

(a) (b) (c) (d) None of these

Explanation:
We know that if are subgroups of If [ ][ ] are finite then
[ ] [ ][ ]
Using above formula, we have [ ] [ ]
Option (a) is correct.
P a g e | 77

Q:10 Which of the following subgroup of has infinite index?

(a) ⟨ ⟩ (b) ⟨ ⟩ (c) ⟨ ⟩ (d)

Explanation:
We know that }, ⟨ ⟩ }
Its cosets in are
Similarly, coset of ⟨ ⟩ in are ⟨ ⟩ ⟨ ⟩
Cosets of in is only one
Cosets of ⟨ ⟩ in are infinite. ⟨ ⟩ ⟨ ⟩ ⟨ ⟩ ⟨ ⟩
Option (b) is correct.

Q:11 Number of left cosets of in are _____

(a) (b) (c) (d)

Explanation:
We know that number of distinct left or right cosets of
in ( ) .
Using this formula, we have distinct left cosets of in
Option (b) is correct.

Q:12 There must exist an element of order in _____

(a) (b) (c) (d)

Explanation:
The order structure of is as there are elements of order will
is of order has an element of order does not have an element of order
(already discussed on page # ).
is non-abelian group of order
So, it has no element of order
Option (c) is correct.
P a g e | 78

Q:13 A subgroup of finite index in an infinite group has _______ with every infinite
subgroup of
(a) Trivial intersection (b) Non-trivial intersection
(c) Finite intersection (d) None of these

Explanation:
Here, we are talking about an infinite group.
So, for better understanding we take group of integers as any infinite group is
isomorphic to
Take which is an infinite group with finite index( ).
Clearly,
Similarly, intersection with any infinite subgroup must be non-trivial in fact infinite.
Option (b) is correct.

Q:14 There does not exist an element of order in ________

(a) (b) (c) (d) All of these

Explanation:
We know that in there must exists an element of order
So, in there is an element of order
In there is an element of order by Lagrange‟s theorem.
In there exists non-abelian subgroup of order but there does not exist an element
of order
Option (a) is correct.

Q:15 Let and [ ] [ ] then [ ] ______


(a) (b) (c) (d)

Explanation:
We know that for two subgroups of a group if [ ][ ] then
[ ] [ ][ ]
Using above result, we have [ ]
Option (d) is correct.
P a g e | 79

Answers:

Q1 d Q2 b Q3 d Q4 b Q5 d
Q6 d Q7 b Q8 d Q9 a Q10 b
Q11 b Q12 c Q13 b Q14 a Q15 d
P a g e | 80

Center, Centralizer and Normalizer

Center of a Group:
The set of those elements of a group which commute with every element of is called center
of group. Mathematically, it can be written as:

Remember:
1. Center of a group is a subgroup of
2. if and only if is abelian.
3. where are any groups.
4. Let be a non-abelian group of order is a prime number then
( )
5. Center of finite groups is non-trivial. | | ( )
6. ( ( ( )))

7. ( ( ( ))) {

Examples:
1. Take } Since, given group is abelian. So, its center coincides with
the group.
2. Take } We find those elements of
which commute with every element of Clearly, identity commute with every element
of We check other elements.
Thus, }
3. Take } Thus,
}
4. Take }

Thus, }
P a g e | 81

Center of some families of Group:


1.
2. ( )
3. }
4. }
5.
6. }
}
7. {
, -
8. ( )
9. ( ( ))

Centralizer of a Subgroup:
Let be a subgroup of The set of those elements of which commute with every element
of is called centralizer of in Mathematically, it is written as:

Note: Centralizer can also be defined for any subset of a group

Results:
1. Centralizer of a subgroup of a group is always subgroup of
2. In an abelian group for any subgroup of
3. In any group
4. For any ⟨ ⟩
5. A subgroup of a group is maximal abelian if
6. For any subgroup of
7. Let then } }
8. For any group ( )
9. For ( )
10. For any ( ( ))
11. If then
12. Let be a cycle in then ⟨ ⟩ }
13. Let be a cycle in then | |
14. Let be a cycle in then
15.
P a g e | 82

Examples:
1. Take } }
Clearly, identity element commutes with every element of So, Since, for

which is not equal to


which is not equal to
Thus, } Since, is maximal abelian subgroup of
2. Take } }
Clearly, Since, group is abelian. So, every element of commute with
every element of Thus,
3. Take } }
Clearly, commute with every element of Thus,
Now, which is
equal to
which is equal to
which is equal to
Thus, }

Centralizer of some special subgroups:

1. Take ⟨ ⟩ , -

2. Take

3. Let be a non-abelian group of order is a prime and .


( )

Complex in a Group:
Any subset of a group is called complex of a group. For example, } is complex of
Two complexes are said to be permutable or if there exists in and
in such that

Normalizer of a Subgroup:
Let The set of those elements of which permute with is called normalizer of in
Mathematically, it can be written as:

}
P a g e | 83

Results:
1. Normalizer of a subgroup of a group is always a subgroup of
2. In an abelian group subgroup of
3. For any subgroup of
4. Let | | then
5. Let be a finite group then
6. Let | | if divides [ ] then
7. For any ⋂
8. For any group ⟨ ⟩
9. Let then ( )
10. If then
11. For any group ( )

Self-Normalizing:
A subgroup of a group is self-normalizing if normalizer of coincides with it.

Examples:
4. Take } }
Since, group is abelian. So,
5. Take } }
Clearly,

} }
} }
} }
Thus,
P a g e | 84

Q:1 Which of the following is not true for any abelian group ?

(a) (b)
(c) (d)

Explaination:
In an abelian group
Also in an abelian group cancellation law holds.
Option (b) need not to be true. For justification, we consider a counter example.
Take } Then,
Option (b) is correct.

Q:2 ______

(a) (b) (c) (d) }

Explanation:
is non-abelian group with trivial center.
Option (d) is correct.

Q:3 If then ( ) _____

(a) (b) (c) (d)

Explanation:
Since, and every group of order is abelian.
So,
Option (b) is correct.

Q:4 If then ________ where is non-abelian group.

(a) (b) (c) (d)


P a g e | 85

Explanation:
We know that center of a - group is non-trivial.
Also, if is a non-abelian group of order then order of center of group is .
| |
Option (d) is correct.

Q:5 ( ( ( ))) _______

(a) (b) (c) } (d) None of these

Explanation:
is the set of all those elements of which commutes with all elements of
So, is an abelian subgroup of any group and ( ) because is
abelian. Thus, we can write as ( ( ( )))
Option (b) is correct.

Q:6 __________

(a) } (b) } (c) } (d) }

Explanation:
We know that
}
{
, -
Here, }
Option (c) is correct.

Q:7 Point out the group for which there exists two proper non-trivial subgroups
such that } }

(a) (b) (c) (d)


P a g e | 86

Explanation:
In there exists two proper non-trivial subgroups
Here, } }
In all other options, given groups do not meet the criteria of given condition.
Option (c) is correct.

Q:8 For | | , Choose the correct result?

(a) | | (b) | |
(c) | | (d) | |

Explanation:
Given group is group.
So, here are two possibilities.
If is an abelian group then | | | |
If is non-abelian group then | | because center of a group is non-
trivial.
Option (d) is correct.

Q:9 If and } then ________

(a) } (b) (c) } (d) }

Explanation:
Given that } }
We know that } Also, normalizer of center of group is the whole
group.
( )
Option (b) is correct.
P a g e | 87

Q:10 Which of the following is correct statement?

(a) A group itself, is always self-normalized.


(b) The center of non-abelian group of order is
(c) Identity subgroup of any group is always self-normalized.
(d) For any subgroup of . ( ( ( )))/

Explanation:
A subgroup of is said to be self-normalized if
Since, for any group
So, the group itself is always self-normalized.
The center of a non-abelian group of order is
Thus, center of non-abelian group of order is
For any, group } }
Thus, } which shows identity group is not self-normalized.
For any subgroup of . ( ( ( )))/
Option (a) is correct.

Q:11 For which of the following subgroup of ?

(a) } (b) }
(c) } (d) All of these
Explanation:
We know that, a subgroup of is maximal abelian if and only if
Since,
}
} } are maximal abelian subgroups of
So, for each of these subgroups,
Option (d) is correct.

Q:12 Take } } then which of the following is


best option?

(a) (b)
(c) (d)
P a g e | 88

Explanation:
We know that So, ( )
Since, is maximal abelian subgroup of
So,
because is a subgroup of abelian group
( )
because is any subgroup of abelian group
( )
because is any subgroup of abelian group
( )
Option (b) is correct.

Q:13 For any subgroup of a group choose the false statement.

(a) (b)
(c) (d) None of these

Explanation:
For any subgroup of a group
But need to be contained in
For justification, we have a counter example.
Take then is not contained in
Option (c) is correct.

Q:14 Take } then _______

(a) } (b) (c) (d)

Explanation:
Clearly, } }
} }
}
} }
} }

Option (b) is correct.


P a g e | 89

Q:15 There exists a subgroup of order ______ in which is self-normalized.

(a) (b) (c) (d) None of these

Explanation:
Take } } } }
Since, because
Option (d) is correct.

Answers:

Q1 b Q2 d Q3 b Q4 d Q5 b
Q6 c Q7 c Q8 d Q9 b Q10 a
Q11 d Q12 b Q13 c Q14 b Q15 d
P a g e | 90

Conjugacy Classes and Class Equation

Conjugate Element:
is conjugate to if b  gag 1 for some g  G
Notation:

is conjugate to if
g 1bg  a
( g 1 )b( g 1 ) 1  a

Some Results:
1. Conjugacy relation between elements of is an equivalence relation.
2. Two elements are conjugate to each other if there exists such that

3. Conjugate elements have same order. | | | |


4. If then every element of commutes with all its conjugates.

Conjugacy Class:
Ca  {x  G , x  gag 1 for some g  G }
The set of all elements of that are conjugate to element is called conjugacy class of
element

Size of Conjugacy Class:

Ca  G : NG (a) , The number of elements in a conjugacy class of an element is equal to index


of normalizer of that element in group.
 If is a finite group, a  G then Ca divides G .

Conjugacy Classes of Some Groups:

(i) Conjugacy Classes of Abelian and cyclic Groups:

In an abelian group every element is self conjugate so | | .


If is of order then there are conjugacy classes.
Same case is for cyclic groups.
P a g e | 91

(ii) Symmetric Group :


The number of conjugacy classes of is the number of positive integer partition of .

Or the number of cycle type in . The elements of of same structure are conjugate.

For size of each conjugacy class we state a result,

Suppose that has a cycle pattern which corresponds to the partition then
number of elements which are conjugate with in is given by

Using this theorem we can also find number of elements of some specific order in

Example:
S3  {e, a, a 2 , b, ab, a 2b}
} then positive integer partition of are

So, there are conjugacy classes of stated as:


Corresponding to the partition , so number of conjugates elements
corresponding to the partition are so only one element in this
conjugacy class.

Corresponding to the partition , so number of conjugates elements


corresponding to the partition are so three elements or
in this conjugacy class.

Corresponding to the partition , so number of conjugates elements corresponding to


the partition are so two elements or in this conjugacy class.

Example:
| | and partitions of are so there are
total conjugacy classes.
Corresponding to the partition , so number of conjugates elements
corresponding to the partition are so only element in this
conjugacy class.
Corresponding to the partition , so number of conjugates elements
corresponding to the partition are so six elements
} in this conjugacy class.
P a g e | 92

Corresponding to the partition , so number of conjugates elements corresponding


to the partition are so three elements } in this
conjugacy class.

Corresponding to the partition , so number of conjugates elements


corresponding to the partition are so eight elements
} in this conjugacy class.

Corresponding to the partition , so number of conjugates elements corresponding to


the partition are so six elements
} in this conjugacy class.
By using above information we can state that there are elements of order elements of
order elements of order in

Example:

| | there are conjugacy classes of . Which are },


}, , } , }

(iii) Dihedral Group :

n3
 2 n is odd
No of conjugacy classes of = 
 n  6 n is even
 2
D5  10
53 8
Here, n  5   4
2 2
No. of conjugacy classes in
Dn  2n , Dn   r , s | r n  (s)2  (rs)2  e 

Case: I n  2k  1 kN
 {e},{r },{r },...,{r  k },{sr b | b  1,..., n}
1 2

O( Dn )  2n  1  n  2  2  ...  2
n 1
times
2

Example: D5  {e, r , r 2 , r 3 , r 4 , s, rs, r 2 s, r 3 s, r 4 s}


D5  10
n  5  2k  1  2(2)  1
P a g e | 93

n3 8
No. of conjugacy classes in D5   4
2 2
O( D5 )  10  1  5  2  2  class equation
 {e},{r 1},{r 2}
 {e},{r , r 1},{r 2 , (r 2 ) 1}
 {e},{r , r 4 },{r 2 , r 3}
{sr b | b  1,..., n}
 {sr1 , sr 2 , sr 3 , sr 4 , sr 5 }
 {r 4 s, r 3 s, r 2 s, rs, s}

Case: II n  2k , k  N
 {e},{r k },{r 1},{r 2},...,{r  ( k 1) }
,{sr 2b , b  1,..., k} , {sr 2b1 , b  1,..., k}
n n
O( Dn )  2n  1  1    2  2  ...  2
2 2 n2
times
2

Example: D4  {e, r , r 2 , r 3 , s, rs, r 2 s, r 3 s}


D4  8
n  4  2k  2(2)
n  6 10
No. of conjugacy classes in D4   5
2 2
4 4 42
O( D4 )  8  1  1   
2 2 2
O( D4 )  8  1  1  2  2  2  class equation
 {e},{r 2},{r 1}
 {e},{r 2},{r, r 1}
 {e},{r 2 },{r , r 3}
{sr 2b | b  1,..., k}
 {sr 2 , sr 4 }
 {r 2 s, s}
{sr 2b 1 , b  1,..., k}
{sr 2b 1 , b  1, 2}
 {sr1 , sr 3}
 {r 3 s, rs}
P a g e | 94

Class Equation:

5. G  n  C1  C2  ...  Ci
 All abelian groups have only one element in each conjugacy class.
 Z n  1  1  1  ...  1 same for Cn and U n
n times

 S3  6  1  3  2
 A3  3  1  1  1
 S4  24  1  6  3  8  6
 A4  12  1  3  4  4
When is odd
 Dn  2n  1  n  2  2  ...  2
n 1
times
2

 D5  10  1  5  2  2
When is even
n n
 Dn  2n  1  1    2  2  ...  2
2 2 n2
times
2

 D4  8  1  1  2  2  2

Class Equation of Non-Abelian Groups of Order :

 Z (G)  p
  a Z (G) , NG (a)  p 2
 No. of conjugate classes of cardinality are
 No. of conjugate classes of cardinality '' p '' are p 2  1 .
 So total conjugacy classes  p2  p  1
O(G)  p 3  1  1  1  ...  1  p  p  p  ...  p
p times
( p 2 1) times

Example: Q8  {1,1, i, i,  j, j, k , k}


Q8  8  23
p2
No. of conjugacy classes of size are p  2
No. of conjugacy classes of size are p 2  1 = 3
So total conjugacy classes  p 2  p  1  4  2  1  5
P a g e | 95

Q8  p3  1  1  1  ...  1  p  p  p  ...  p
p times
( p 2 1) times

Q8  23  1  1  2  2  2
    
{1},{1} ,{i} , { j} ,{ k}

Remember:
n(n  1)(n  2)...(n  (m  1))
 No. of conjugates of  ( cycles) in Sn 
m
 If is odd, then set of all cycles consists of two conjugacy classes of equal size in An .

Some more results to remember:

1. If a finite group has only two conjugacy classes then, order of group is
2. If then
3. If a prime and then
4. If and if } then is abelian.
5. A dihedral group is odd has only one conjugacy class of involutions.
6. If are two conjugacy classes of a group then
7. A conjugacy class of a subgroup of index either is a conjugacy class of the group, or it
contains half of its elements.
8. For }
9. Let be a normal subgroup of a group then is union of all conjugacy classes in of
all elements of
10. If are from same conjugacy class of a group then | | | |
11. A finite group of order greater than does not have exactly three conjugacy classes.
12. Elements of same conjugacy class have conjugate centralizers.
13. All elements of a conjugacy class of an element of a group are of same order.
14. For any | |
15. Two non-isomorphic group can have same class equation.
16. All elements of a center of group are self-conjugate.
17. Any two conjugate subgroups are of same order and isomorphic.
P a g e | 96

Q:1 The number of conjugacy classes of are

(a) (b) (c) (d)

Explanation:
If is a non-abelian group of order then number of conjugacy classes of are

| | then there are total conjugacy classes of

Option (b) is correct.

Q:2 is class equation of

(a) A4 (b) (c) D12 (d) S4

Explanation:
Here, the sum and this cannot be the class equation of an
abelian group, so we will check only .
There are 5 conjugacy classes of and each conjugacy class is of size and 9
conjugacy classes of
Option (d) is correct.

Q:3 Number of conjugacy classes of are

(a) (b) (c) (d)

Explanation:
The number of conjugacy classes of symmetric group is the number of positive
integer partition of Here
Option (a) is correct.
P a g e | 97

Q:4 then

(a) {e, b} (b) {e, a, a2} (c) {b, ab, a2b} (d) {e}

Explanation:
Given group is . The number conjugacy classes are and the classes are
} } } So, are conjugates to
Option (c) is correct.

Q:5 For a non-abelian group of order which is not true?

(a) Center will have one element only.


(b) Total number of conjugate classes will be
(c) Group must have elements of order
(d) Lagrange‟s converse does not hold in group.

Explanation:
There are exactly two non-isomorphic groups of order one is and other is .
Center of is trivial.Conjugacy classes of are Since, is prime.
So, have elements of order
Also converse of Lagrange‟s theorem holds in .
Option (d) is correct.

Q:6 Number of elements in conjugacy class of in is/are

(a) (b) (c) (d)

Explanation:
There are conjugacy classes of which are } } } } }
Option (b) is correct.

Q:7 Which of the following result is not appropriate?

(a)
(b) CG  CG  CG ( A)    CG ( A) where A  G
(c) CG ( A)  NG ( A)
(d) The center of non-abelian group of order is
P a g e | 98

Explanation:
For any,
For option (b) is the set of all those elements of which commutes with , while
is set of all those elements of which commutes with all elements of
which is so .
For option (c) clearly from definition that .
Since, center of a non-abelian group of order is .
So center of non-abelian group of order will be
Option (a) is correct.

Q:8 Which of the following could be class equation?

(a) 9  3  3  3 (c) 27  1  1  3  3  3  7
(b) (d) 18  1  3  6  8

Explanation:
Option (c) and (d) are not true as does not divide and does not divide
Moreover, every group of order is abelian.
So, each conjugacy is of size
Thus, option (a) cannot be true. For (b) we have two groups of order one is abelian
and other is , the conjugacy classes of are 4, and of size
Option (b) is true.

Q:9 A subgroup of a group is Maximal Abelian if

(a) H  NG ( H ) (b) H  CG ( H ) (c) Both (d) None

Explanation:
A subgroup of a group is called Maximal Abelian subgroup if
Option (b) is correct.

Q:10 The conjugacy classes of are

(a) (b) (c) (d)


P a g e | 99

Explanation:
There are conjugacy classes of of size and classes are },
} , }
}
Option (a) is correct.

Q:11 ⋂

(a) CG (a) (b) NG (a) (c) d)

Explanation:
denotes the centralizer of any element which is the set of all those elements
of which commutes with }. Thus, .
Option (c) is true.

Q:12 Point out the pair which is not conjugate to each other in the respective group.

(a) * + * + in (b) * + * + in

(c) * + * + in (d) None of these

Explanation:
We know that two elements of a group are conjugate to each other if there exists
such that
Since, there exists * + * + such that

* + * + * + and * +* + * +
So, in option (a) elements are conjugate to each other.
Since, so, in option (b) elements are also conjugate to each
other.
In the similar way, there exists * + * + such that

* +* + * + * +* + * +
Option (d) is correct.
P a g e | 100

Q:13 Which of the following group has exactly three conjugacy classes?

(a) (b) (c) (d)

Explanation:
We know that in an abelian group, number of conjugacy classes is equal to order of
group.
So, number of conjugacy classes of are
In number of conjugacy classes is equal to number of positive integer partitions of
Since, partitions of are
Partitions of are
Number of conjugacy classes of a non-abelian group of order are
Option (b) is correct.

Q:14 Choose the correct statement.

(a) All elements of same order of a group must belong to same conjugacy class.
(b) The order of each element of a conjugacy class need not to be same.
(c) All elements of a conjugacy class have same order.
(d) Conjugacy classes of any two elements of a group must be disjoint.

Explanation:
All elements of same order of a group need not to be in a conjugacy class.
For example, elements of order in } are
and that are not in same conjugacy class because they are in
numbers.
All elements of a conjugacy class are of same order.
Conjugacy classes are equivalence classes.
So, any two conjugacy classes are either disjoint or identical.
They need not be always disjoint.
Option (c) is correct.

Q:15 Which of the following pair of elements must be conjugate to each other?

(a) in a symmetric group of degree


(b) in a dihedral group of order
(c) in a cyclic group of order
(d) in a symmetric group of degree
P a g e | 101

Explanation:
We know that all cycles of same structure in a symmetric group of degree are
conjugate to each other.
Also order of conjugate elements must be same.
In an abelian group, each element is self- conjugate.
Option (a) is correct.

Answers:

Q1 b Q2 d Q3 a Q4 c Q5 d
Q6 b Q7 a Q8 b Q9 b Q10 a
Q11 c Q12 d Q13 b Q14 c Q15 a
P a g e | 102

Normal Subgroups

Definition:
Let be a subgroup of a group then is called normal subgroup of if for all
We denote it as .

A subgroup of a group is normal subgroup of if and only if

A subgroup of a group is normal subgroup of if and only if

Examples:
1. For any group the trivial group } are always normal subgroup of
2. Take } } } }
} } } } } }
} } } } } } Thus, is normal
subgroup of Similarly, all its remaining subgroups are also normal in it.
3. Take } } }
} } } } } Since, left coset
of is not equal to right coset of in So, is not normal subgroup of
} } } } }
} } } }
} } } Since, all left cosets of in are equal
to right cosets of in So, is normal subgroup of
4. Take } } }
} } } } } }
} } } } } } So, is
normal subgroup of
} } } }
} } } } So, is also
normal subgroup of Similarly, every subgroup of is normal in it.
5. Take } } }
}
} } } } } Since, so is
not normal subgroup of } } } }
} } } } } }
} } } } } }
P a g e | 103

} } } } }
} } } } }
Thus, is normal subgroup of Similarly, is also normal subgroup of

Normal Subgroups of some famous families of Groups:


1. All subgroups of an abelian or cyclic group are its normal subgroups. Converse needs
not to be true. For example, group of quaternions of order
2. The center of a group is always its normal subgroup.
3. The alternating group of even permutations is normal subgroup of
4. Every subgroup of consisting of rotations is normal subgroup of
5. Special linear group is normal subgroup of general linear group
6. Normal subgroups of are of the form
7. Number of normal subgroups of
8. Number of normal subgroups of a cyclic group ( )

9. Number of normal subgroups of {

10. Number of normal subgroup of {

11. Number of normal subgroups of {

Results:
1. Intersection of any collection of normal subgroups of a group is normal subgroup.
2. Product of two normal subgroups is normal.
3. If a group has unique subgroup of some finite order then is normal subgroup of

4. } is a normal subgroup of
5. A subgroup of a group of index is normal in it.
6. If is normal subgroup of then is normal subgroup of
7. If a group has a normal subgroup of order then intersection of all subgroups of
order is normal subgroup of
8. If is a normal subgroup of order in a group then must be contained in the
center of group.
9. Let be a group and a prime number then | } is a normal subgroup of
10. Let be a subgroup of then is normal subgroup of if and only if for all

11. If are normal abelian subgroups of a group } then is also


abelian.
P a g e | 104

12. If is a normal subgroup of a group ,| | odd number and then

13. Let be a finite group and be any prime such that | | then every subgroup of
order (if exists) is normal in
14. ,* + - is normal subgroup of ,* + -
15. Property of being normal subgroups need not to be transitive.
16. If is any subgroup of a group and a normal subgroup of then is
normal subgroup of
17. Any subgroup of center of group is normal subgroup of
18. For any two subgroups of if then
19. Let then
20. If is the smallest prime divisor of order of a finite group then any subgroup of index
is normal in
21. For any subgroup of
22. A subgroup of a group is normal in it if and only if
23. If a group has a subgroup of finite index then it must have a normal subgroup of finite
index.
24. A group which has no normal subgroups except group itself and identity group is called
simple group.
25. All cyclic groups of prime order are simple.
26. For any subgroup of is normal subgroup of
27. Let be an element of order in then ⟨ ⟩ is normal subgroup of if and
only if
28. Product of subgroup and a normal subgroup of a group is its subgroup.
P a g e | 105

Q:1 Pick up the group whose all subgroups are normal?

(a) (b) (c) (d)

Explanation:
There exists a subgroup } of which is not normal in it that we have already
discussed in detail.
There exists a subgroup } which not normal in it.
Because
} } }
}
All subgroups of are not normal in it. There exists a subgroup
} which is not normal because, }
} } }
All subgroups of are normal in it that we have discussed in detail.
Option (c) is correct.

Q:2 Which of the following is a simple group?

(a) (b) (c) (d)

Explanation:
A simple group is a group whose only normal subgroups are }
We know that all subgroups of are normal.
There are four normal subgroups of }
Normal subgroups of are }
Only normal subgroups of are }
Option (a) is correct.

Q:3 Finite simple abelian groups are of _____ order.

(a) prime power (b) composite (c) prime (d) even

Explanation:
The only finite simple abelian groups are of prime order that are cyclic groups of prime
order.
Option (c) is correct.
P a g e | 106

Q:4 If is a normal subgroup of a group then which is true for ?

(a) (b)
(c) Every right coset of is equal to left coset (d) All of these

Explanation:
All of these statements are equivalent for a normal subgroup of a group
Option (d) is correct.

Q:5 If and then choose the best option.

(a) (b) (c) (d)

Explanation:
If we take } } }
}
Now, if we take } } }
Now, if we take }
Consider an arbitrary element
Then
Option (c) is correct.

Q:6 Every subgroup of index _______ is normal in a group

(a) (b) (c) (d) None of these

Explanation:
We have a useful result “Every subgroup of index is normal in a group
For remaining options, we find counter examples.
Consider } which is not normal and of index
Consider } which is not normal and of index
Option (a) is correct.

Q:7 Number of normal subgroups of are

(a) (b) (c) (d)


P a g e | 107

Explanation:
Normal subgroups of are }
Option (c) is correct.

Q:8 If then ______

(a) (b) (c) (d) }

Explanation:
We know that center of a group is always its normal subgroup and a subgroup of a
group is normal in it if and only if
Using above results, we have
Option (c) is correct.

Q:9 Let then choose the correct one.

(a) (b) (c) (d) None of these

Explanation:
We know that normalizer of a normal subgroup is the entire group.
Here,
Option (b) is correct.

Q:10 Take then which of the following subgroup of self-normalizes?

(a) } (b) (c) (d) None of these

Explanation:
We know that a subgroup of a group is self-normalized if
Also normalizer of a normal subgroup is the whole group.
Since, all above subgroups are normal in So, }
Option (d) is correct.
P a g e | 108

Q:11 In unique normal subgroup is

(a) Subgroup of rotations (b) Subgroup of reflections (c) Both (d) None

Explanation:
We know that subgroup of reflection is not normal subgroup of as set of all
reflections does not form a group.
Only unique normal subgroup is the subgroup of rotations.
Option (a) is correct.

Q:12 Pick up the group whose proper subgroup can never be self-normalized.

(a) Abelian group (b) Cyclic group (c) Non-abelian group (d) None

Explanation:
We know that, all subgroups of a cyclic(abelian) group are normal in it.
So, their normalizer must coincide with entire group.
There exists a subgroup of non-abelian group which is self-normalized.
} is self-normalized subgroup in
Option (b) is best option.

Q:13 For any group which of the following subgroup of can never be self-
normalized?
(a) Abelian subgroup (b) Cyclic subgroup
(c) Normal subgroup (d) None of these

Explanation:
There exists an abelian or cyclic subgroup } of which is self-normalized.
Normalizer of a normal subgroup is the entire group which can never be self-normalized
in case of proper normal subgroup.
Option (c) is correct.

Q:14 Number of normal subgroups of are

(a) (b) (c) (d)


P a g e | 109

Explanation:
We know that number of normal subgroups of

Number of normal subgroups of


Option (a) is correct.

Q:15 Number of non-isomorphic normal subgroups of order in are

(a) (b) (c) (d)

Explanation:
Every subgroup of order is normal in being of index
Subgroups of order in are } } } one
cyclic of order and remaining two are Klein four subgroups.
Option (b) is correct.

Answers:

Q1 c Q2 a Q3 c Q4 d Q5 c

Q6 a Q7 c Q8 c Q9 b Q10 d

Q11 a Q12 b Q13 c Q14 a Q15 b


P a g e | 110

Quotient Groups and Homomorphism


Quotient Group:
Let be any group and be normal subgroup of then set of all left or right cosets of in is the
group called factor or quotient group of by denoted by .

Whereas, the operation of factor group is .

The identity element is . For better understanding we discuss some examples.

Example
Consider and

Then } } is the factor group of


order . The group on is actually the group of residue classes modulo under addition
or

Example

Consider the group,

, -

Identity

4  4  4 
       
3  3  3 
4  8  4 
       
3  3  3 
4   12 
       4  
3   3 

p q  q
The above group is of infinite order while, order of each element is finite.
P a g e | 111

Examples for Non–Abelian Groups:

1. Consider symmetric group of degree

An is a normal subgroup of Sn

Sn n!
 2
An n !
2
Sn
 C2  2n  2
An
A3  {e , a , a 2 } , S3  {e , a , a 2 , b , ab , a 2b}
S3
 {gA3 , g  S3}
A3

bA3  b{e , a , a 2 }  {b , ab , a 2b}  ba  a 2b


abA3  ab{e , a , a 2 }  {ab , b , a 2b}  ba 2  ab
a 2bA3  a 2b {e , a , a 2 }  {a 2b , ab , b}  a(ba)  a.a 2b  b
  A3 ,{b , ab , a 2b}  a(ba 2 )  a.ab  a 2b

 A ,{b , ab , a b} has order 2. {b , ab , a b} is a self-inverse.


3
2 2

 e , f  , f 2  e

 
 0,1

Point to Remember:
 Quotient group of cyclic (abelian) group is cyclic (abelian) but converse is not true.

Results:
G
1. A group G is abelian if and only if factor group is cyclic.
Z (G )
2. the normal subgroups are } } } we construct factor group for
}
} is of order 4. Now, to determine whether it is cyclic on non-cyclic abelian
we need to find order of elements of factor group.
, }, },
P a g e | 112

} so, } since, each non-identity element is of


order so cannot be isomorphic to , Hence
3. Consider Dn Family

D4  e, r , r 2 , r 3 , s, rs, r 2 s, r 3 s
Z (G )  Z ( D4 )  1, r 2 
H  e, r , r 2 , r 3 
K  e , r 2 , s , r 3 s
D4 8
 4
Z ( D4 ) 2

There are two groups of order 4. V4 and Z 4 .


G
We cannot select Z 4 according to result G is abelian if and only if factor group is cyclic.
Z (G )

D4
Here, factor group is non-abelian.
Z ( D4 )

D4
So,  V4
Z ( D4 )

G
  gH ,  g  G
H
sH  s e, r , r 2 , r 3   s, r 3 s, r 2 s, rs
rsH  rs e, r , r 2 , r 3   rs, s, r 3 s, r 2 s
r 2 sH  r 2 s e, r , r 2 , r 3   r 2 s, rs, s, r 3 s
r 3 sH  r 3 s e, r , r 2 , r 3   r 3 s, r 2 s, rs, s

  H , {s, rs, r 2 s, r 3 s
G D4

H H
 {H , rsH }  C2  2
 rsH  .  rsH    rsrs  H
 rsH  .  rsH    r.r 3 s.s  H   r 4 s 2  H  eH  H

So order of  rsH  is 2.
Now, For
P a g e | 113

K  {e , r 2 , s , r 2 s}

  gK ,  g G   K , r 3 sK   C2 
G
2
K
rk  r e , r 2 , s , r 2 s  r , r 3 , rs , r 3 s
r 3 K  r 3 e , r 2 , s , r 2 s  r 3 , r , r 3 s , rs
rsK  rs e , r 2 , s , r 2 s  rs , r 3 s , r , r 3 
r 3 sK  r 3 s e , r 2 , s , r 2 s  r 3 s , rs , r 3 , r

For Order

 r sK  . r sK    r s  . r s  K
3 3 3 3

 r sK  . r sK   r  sr  s  K
3 3 3 3

 r sK  . r sK    r . rs . s  K  K
3 3 3

So order of  r sK  is 2
3

Similarly, in Alternating Groups

A4 12 A
  3  4  C3  3
V4 4 V4
S4 24
  6  We have to check for S3 , 6
V4 4
Since, there is no element of order in the factor group
So, this factor group will be isomorphic to non-abelian group of order .

Homomorphism:

If we have two groups  G,  ,  G ' ,  and  :G  G' be a function such that


  a  b     a    b  then function  :G  G' is homomorphism.
ker  {g G ,  ( g )  e'}
ker   G , ker  G

Types of Homomorphism:
 one – one homomorphism  Monomorphism
 onto homomorphism  Epimorphism
 (one – one + onto) homomorphism  Isomorphism
P a g e | 114

Automorphsim:
 :G  G
(homomorphism + bijective)  automorphism

Examples of Homomorphism:

 G  GL2 ( R)  R  G '
 ( A)  A
 ( A.B)  AB  A B
 ( A.B)   ( A). ( B)
ker   {A GL2 ( R) ,  ( A)  1} { SL2 ( R)}

  :R  R
 ( x)  x
 ( x. y)  x. y  x y
 ( x. y )   ( x). ( y )
  homomorphism
ker  {g R ,  ( g )  1} {1,1}
 R  x   R  x  ,  R  x  ,   all real polynomials
 :  p( x)   p ' ( x)
  p( x)  q( x)    p( x)  q( x)   p ' ( x)  q ' ( x)
'

  p( x)  q( x)     p( x)     q( x) 
ker   Constant polynomials
 G   a  infinite cyclic group
 :G 
 (a n )  n
 (a k . a l )   (a k l ) k  n, l  n
 (a k . a l )  k  l
 (a k . a l )   (a k )   (a l )
  homomorphism
ker  {g G , ( g )} {g G , (an )  0 , n  0}  {1}
P a g e | 115

Result:

 :G  G' is one – one iff ker   {0}


Since for each n , then there is an G such that  (a n )  n
  is isomorphism.

Properties of Isomorphism acting on elements:

  : G  G' is isomorphism
1.  (e)  e'
2.  ( g n )   ( g )  g  G  n
n

3. For any and commute iff and commute.


4. iff
5. | | | |
6. For a fixed integer a fixed group element in the equation has the same
number of solutions in as in .
7. If is finite, then and have exactly the same number of elements of every order.

Properties of Isomorphism acting on Groups:


Suppose is an isomorphism from a group onto a group Then

 is an isomorphism from onto .


 is cyclic (abelian) iff is cyclic (abelian).
 If is subgroup of then is a subgroup of
 If is subgroup of then } is a subgroup of
 ( )

Cayle’s Theorem:
Every group is isomorphic to a subgroup of group of permutations.

This result tells us that there is an isomorphic copy of each group in group of
permutations or every group has presentation in terms of permutations.

Examples:
9. Consider the group then this group is isomorphic to each of the following subgroups
} } } of .
10. } .
11. } .

12. }
P a g e | 116

13. } .
14. } .

Result:
Every group is isomorphic to some group of matrices.

Examples:

1. ,* + * +-

2. ,* + * + * +-

3. ,* + * + * + * +-

4. ,* + * + * + * +-
5.
6. ,* + * + * + * +-

7. ,* + * + * + * + * + * +-

8. {[ ] [ ] [ ] [ ] [ ] [ ]}

9. ,* + * + * + * + * + * + * + * +-

10. ,* + * + * + * + * + * + * + * +-
P a g e | 117

Q:1 The group of all nth roots of unity is isomorphic to.

(a) n (b) (c) (d)

Explanation:
As we know that, every finite cyclic group of order is isomorphic to and given
group is cyclic of order so the result follows.
Option (a) is correct.

Q:2 ______

(a) 4 (b) V4 (c) S3 (d) A3

Explanation:
By a well-known theorem is cyclic if and only if is abelian.
The center of Group of Quaternions is so is group of order 4 and isomorphic to .
Option (b) is correct.

Q:3 Order of ̅̅̅̅ in is ______

(a) (b) (c) (d)

Explanation:
As, ̅̅̅̅̅ and order of ̅̅̅̅ is
Option (d) is correct.

Q:4 If is homomorphism. Then Ker  is

(a) Subgroup of (c) Normal subgroup of


(b) Only a subset of (d) Normal subgroup of
P a g e | 118

Explanation:
For a group homomorphism
the where is the identity element of
By definition is a subset of , moreover is a normal subgroup of
Option (c) is correct.

Q:5 For _______

(a) D4 (b) V4 (c) S3 (d) 4

Explanation:
So, and is cyclic if and only if is abelian thus is non-cyclic
group of order hence isomorphic to
Option (b) is correct.

Q:6 is homomorphism then is injective if and only if


(a) } (b) }
(c) (d) None of these

Explanation:
A group homomorphism is injective if and only if consists of only
identity of
Option (a) is correct.

Q:7 Which of the following is not a transitive relation in groups?

(a) Subgroup relation (c) Relation of conjugacy


(b) Normality of subgroup (d) Isomorphism

Explanation:
Subgroup relation, conjugacy relation and being isomorphic is a transitive relation
among groups.
While normality of subgroups is not transitive as we have a counter example in
then } and } then is normal in and is normal in
but is not normal in .
Option (b) is correct.
P a g e | 119

Q:8 Which of the following is true?

(a) S3 is normal subgroup of S4 .


(b) A4 has a subgroup of order
(c) A homomorphism may have an empty kernel.
(d) An Sn and [ ] .

Explanation:
have four normal subgroups which are } and so, is not normal in
while has no subgroup of order 6.
Every group homomorphism has a non-empty kernel as there is atleast an identity
element in a group which maps to the identity.
Option (d) is correct.

Q:9 ______

(a) 2 (b) 3 (c)  2  (d) 10

Explanation:
As }
Hence, given Quotient group is isomorphic to
Option (a) is correct.

Q:10 For a non-abelian group | | | | then

(a) {e} (b) S3 (c) 6 (d) D6

Explanation:
Since for any group is cyclic if and only if is abelian, thus given quotient
group will be isomorphic to any non-cyclic group of order as is non-abelian.
So,
Option (b) is correct.
P a g e | 120

Answers:

Q1 a Q2 b Q3 d Q4 c Q5 b

Q6 a Q7 b Q8 d Q9 a Q10 b
P a g e | 121

Automorphism and Derived Subgroup

Fundamental Theorems of Homomorphism:

(i) If is a homomorphism then where is the set of


images of the elements of under group homomorphism
If is onto
Then

(ii) If then

(iii) If and then .

Endomorphism:
Any Homomorphism is called an endomorphism.

Automorphism:
If Endomorphism is bijective then it is called Automorphsim.

Group of Automorphism:
is an isomorphsim} is the group under compositions of functions, the
identity element is Identity function defined by which is also an
automorphsim.

Inner Automorphism:
Let be fixed element then defined by

If .

Collection of all Inner Automorphsims of a group is also a group under the composition of
functions where identity element is
P a g e | 122

Results:
(i)
(ii)

(iii) is isomorphic to a subgroup of where is any subgroup of .

(iv) Outer Automorphism group


(v) A group is abelian if and only if defined by is an
automorphism.
(vi) is a cyclic group if and only if is a group of order and
is an odd prime.
(vii) Isomorphic groups have isomorphic automorphism groups but converse need not to
be true.
(viii) In an abelian group of odd order, the mapping sending every element to its square is
an automorphism.
(ix) In general linear group defined as
is an automorphism for all
(x) If ( ) then
(xi) If is an abelian group of order and be a positive integer such that
then, the mapping is an automorphism.
(xii) Let be a finite cyclic group of order then the mapping is an
automorphism of if and only if
(xiii) If is finite abelian group such that ( ) then
(xiv) Let be an automorphism of a group then } is a
subgroup of
(xv) Let then defined by is an
automorphism.
(xvi) No group have its automorphism group, cyclic of odd order greater than

Examples:
(i) then [ ]

Whereas } then the automorphsims are

. Also } using result (ii).

The Outer Automorphism group for is also

(ii) then there are only two automorphsims that have which are

The Identity and The Inverse map, Hence


P a g e | 123

(iii)
(iv)

For following examples use result (ii).

(v)
(vi)
(vii)

Points to remember:
 If is infinite then need not to be infinite.
 is abelian then need not to abelian like .
 is cyclic but need not to be cyclic.
 of an abelian group may be non-abelian.

Some Automorphism Groups:


(i) ( ) ( )
(ii) for
(iii) ( ) for all odd prime numbers.
(iv) ( )
(v) }
(vi)

Order of some Automorphism groups:


| | where denotes Euler-Phi function.

| |

| | for | |

| |

Characteristic subgroup:
Let be a group and be any subgroup of if

then is called characteristic subgroup of


P a g e | 124

Examples:
1. Consider then } thus there are two automorphsims
and
take } then } }
Also } } thus is characteristic subgroup of .
2. For take and thus only automorphsims are
and then it is clear that Hence, is the
characteristic subgroup of

Results:
(i) Every characteristic subgroup is normal but converse may not be true,
as For defined by is an
automorphism but for ,
So, is not Characteristic subgroup of
(ii) Characteristic subgroup property is transitive.
(iii) If is unique subgroup of order in a finite group , then is characteristic
subgroup of
(iv) All subgroups of a cyclic group are characteristic subgroups.

Commutator/Derived subgroup:
For the Commutator of and is denoted by [ ] and defined as

[ ] . While the subgroup generated by all commutators of group is called


Commutator/ derived subgroup and denoted by We write as:

[ ] [ ] [ ] [ ]

Commutator Identities:
For any group and following identities hold.

 [ ] [ ]
 [ ] [ ] [ ]
 [ ] [ ][ ]
 [ ] [ ]
P a g e | 125

Some famous Derived Groups and their orders:

 2

 {

Results:
1. is abelian iff [ ]
2.
3. is the smallest subgroup of for which is abelian.

(above results will be used to find derived subgroup rather than finding commutators)

Examples:

1. then Normal subgroups are }, , then and is the smallest normal


subgroup for which factor group is abelian. Similarly on generalizing, we say that the derived
subgroup of is .
2. then Normal subgroups are } } } and Hence
}
}
3. } and normal subgroups of are
} } } } } The Quotient group so
}
}.
4. then normal subgroups are } and smallest normal subgroup is for which
.Hence .
P a g e | 126

Q:1
(a) (b) (c) (d)

Explanation:
As so, , because does not have an
element of order
Option (a) is correct.

Q:2 | |

(a) | | (b) | | (c) | | (d) Both (a) and (b)

Explanation:
Since, so, ( ) ( )
Option (d) is correct.

Q:3 A group is abelian iff the factor group is ______

(a) Abelian (b) cyclic (c) Abelian but not cyclic (d) Both (a) and (c)

Explanation:
Above statement is a well-known result.
A group is abelian if and only if is a cyclic group.
Option (b) is correct.

Q:4 Which of the following is an automorphism of an abelian group?

(a) (b) (c) (d)

Explanation:
For an abelian group, there are only two automorphsims which are identity and inverse
maps is not a bijection in while is not one-one .
Option (a) is correct.
P a g e | 127

Q:5 Derived sub group of which of the group is }

(a) (b) (c) (d)

Explanation:
A group is abelian iff its derived subgroup is }. Only abelian group is
Option (d) is correct.

Q:6 Which of the following is true?

(a) Every characteristic subgroup is normal.


(b) Every normal subgroup is characteristic.
(c) is not characteristic sub group
(d) is not metabelian group.

Explanation:
Every characteristic subgroup is normal but converse is not true.
is always a characteristic subgroup of any group .
A group is metabelian if its derived subgroup is abelian.
The derived subgroup of is abelian.
So, it is metabelian.
Option (a) is correct.

Q:7 Which of the following is true?

(a) is normal subgroup of


(b) A homomorphism may have an empty kernel.
(c) has a subgroup of order
(d) is a normal subgroup of .

Explanation:
The normal subgroups of are } .
Kernel of any group homomorphism have atleast identity element.
has not subgroup of order 6, while is a normal subgroup of
Option (d) is correct.
P a g e | 128

Q:8 Which is a false statement?

(a) An inner automorphism of an abelian group must be only identity map.


(b) Every subgroup of an abelian group is normal in
(c) is cyclic group of order
(d) has elements.

Explanation:
For any abelian group ,
So, } hence identity map is the only inner automorphism for an abelian
group.
Every subgroup of an abelian group is normal.
which is cyclic group of order .
While the group has elements.
Option (d) is correct.

Q:9 The derived sub group of is

(a) (b) (c) (d)

Explanation:
The derived subgroup of is the smallest normal subgroup for which is abelian.
In case of
Option (d) is correct.

Q:10 If is normal in such that is abelian then

(a) (b) (c) (d)

Explanation:
Since is the smallest normal subgroup for which is abelian.
So, for any other normal subgroup for which is abelian then
Option (a) is correct.
P a g e | 129

Q:11 is a group under the operation

(a) Multiplication (b) Addition (c) Subtraction (d) Composition

Explanation:
is the set of all automorphsims(functions) for any group .
So its binary operation is composition of functions.
Option (d) is correct.

Q:12 If is any infinite cyclic group then

(a) (b) (c) (d) }

Explanation:
Since, every infinite cyclic group is isomorphic to and .
Option (b) is correct.

Q:13 Let be a normal subgroup of a finite group . Then the order of quotient group
will be

o(G )
(a) o(G )  o( N ) (b) (c) o(G )  o( N ) (d) o(G )  o( N )
o( N )

Explanation:
| |
If is a normal subgroup of a finite group then order of factor group is group | |
.
Option (b) is correct.

Q:14 Which of the following is only unique cyclic subgroup of order in

(a) Characteristic subgroup of Dn (c) Subgroup of rotation


(b) Subgroup of reflection (d) Both (a) and (c)
P a g e | 130

Explanation:
The subgroup of rotation is the unique cyclic subgroup of order in
Set of all reflections does not form a subgroup of order
Being the unique subgroup of order subgroup of rotation is Characteristic subgroup.
Option (d) is correct.

Q:15 Point out false statement.

(a) In any group inversion  : a 1 is a bijection.


(b) Group may be non-abelian if inversion  : a 1 is an automorphism.
(c) (V4 )  GL(2, 2 )
(d) All above

Explanation:
In any group the function which maps each element to its inverse is always a bijection
due to uniqueness of inverses, Also .
For any non-abelian group , the inversion map is not a homomorphism.
Option (b) is correct.

Q:16 Take and then

(a) C2 (b) C3 (c) Aut (C3 ) (d) Aut (C2 )

Explanation:
Let be any subgroup of then is isomorphic to a subgroup of .
Here, so by using theorem, we can say
Option (a) is correct.

Q:17 Which of the following group is complete?

(a) D4 (b) C3 (c) A3 (d) S4


P a g e | 131

Explanation:
A group is complete if every automorphism of is inner and it is centerless.
The Symmetric groups are complete when .
Option (d) is correct.

Q:18 A set consisting of finite number of conjugates of a group is


(a) Normal in (b) Characteristic in (c) Subgroup of (d) None

Explanation:
For any group set of all conjugate elements of a group is characteristics subgroup of

Option (b) is best option.

Q:19 Let denotes the group of automorphism of a group Which one of the
following is not a cyclic group?

(a) (b) (c) (d)

Explanation:
Since so } which is cyclic.
} cyclic. } which is non-cyclic
} and is an element of order so it is cyclic.

Q:20 For a group

(a) {e} (b) Aut (S3 ) (c) I ( S3 ) (d) None of these

Explanation:
Since, and so }
Option (a) is correct.
P a g e | 132

G
Q:21 If and then 
H

(a) S4 (b) S3 (c) S2 (d) V4

Explanation:
As and then the factor group is of order 6 and
will be isomorphic to as the factor group has no element of order 6.
Option (b) is correct.

Q:22 Point out false statement.

(a) Identity is a Commutator.


(b) Inverse of a commutator is again commutator.
(c) Product of two commutators is again commutator.
(d) None of these

Explanation:
The commutator of two elements of a group is defined as [ ]
Since, for all we have [ ] thus identity is commutator.
Similarly, inverse of a commutator is again a commutator, but product of two
commutators need not to be commutator.
Option (c) is correct.

Q:23 For a group , let denote the group of automorphism of . Which of the
following statement is true?
(a) (b) is cyclic is cyclic
(c) is trivial then is trivial (d)

Explanation:
is of order as has only two automorphism.
So, . which is not cyclic for each .
Moreover, is trivial group but is not trivial.
Option (a) is correct.
P a g e | 133

Q:24 Point out the group, whose all subgroups are characteristics?

(a) (b) (c) (d) All of these

Explanation:
The group of integers has a unique non-identity automorphism and this takes to
It maps every subgroup to itself. Similar happens for any finite cyclic group because
then a subgroup is unique of its order and therefore mapped to itself.
Option (c) is correct.

Q:25 Pick up the false statement.

(a) Every automorphism of is an endomorphsim of


(b) Any two groups corresponding to isomorphic automorphism are isomorphic.
(c) All automorphsims of are inner automorphism.
(d) All of these

Explanation:
A bijective homomorphism from a group to a group is called automorphism.
A homomorphism from a group to a group is called an endomorphism.
Every automorphism is an endomorphism.
All automorphsims of a group are inner automorphism.
Two groups corresponding to isomorphic automorphism group need not to be
isomorphic.
For example,
But groups are non-isomorphic of different order.
Option (b) is correct.

Q:26 Pick up the pair of groups, whose automorphism groups are isomorphic.

(a) (b) (c) (d)

Explanation:
We know that

Option (d) is correct.


P a g e | 134

Q:27 Automorphism group of a/an ________ is not of order

(a) non-abelian group (b) abelian group


(c) cyclic group (d) Non-cyclic group

Explanation:
Automorphism group of a non-abelian group cannot be trivial.
So, automorphism group of a non-abelian group is not of order
Option (a) is correct.

Q:28 For [ ][ ][ ] _______

(a) (b) (c) (d)

Explanation:
By using definition of commutator, [ ]
[ ] [ ]

Thus, [ ][ ][ ]
Option (d) is correct.

Answers:

Q1 a Q2 d Q3 b Q4 a Q5 d
Q6 a Q7 d Q8 d Q9 d Q10 a
Q11 d Q12 b Q13 b Q14 d Q15 b
Q16 a Q17 d Q18 b Q19 c Q20 a
Q21 b Q22 c Q23 a Q24 c Q25 b
Q26 d Q27 a Q28 d
P a g e | 135

Direct Product

External Direct Product:


Let be two groups , then } is a group called External
Direct product of two groups and where and are called Direct factors of The
operation is component wise multiplication.

Alert:
Many authors use as notation for external direct product.

Before discussing examples, we will discuss some results, later on we will use these results for
explanation of examples.

Results:

1. if and only if so direct product of two cyclic groups need not to


be cyclic. Moreover, direct product of two abelian groups is abelian.
2.
3. For any groups
4. is abelian if and only if both are abelian groups.
5. If then
6. If then
7. | | [| | | |] where [| | | |] denotes Least Common Multiple of | | | |
8. if and only if
9.
10. for an odd prime .
11. An abelian group of order with all elements of order is direct product of copies of

12. Only normal subgroup of is either abelian subgroup or intersects with one of
subgroups } or } non-trivially.
13. Number of elements of order in
(i) Number of subgroups of order in
P a g e | 136

(ii) Number of subgroups of order in


(iii) Number of subgroups of order in
(iv) Number of subgroups of order in

Now, we will discuss examples with detailed discussion.

Examples:
1. Take then

} as is of order 6 so it will be isomorphic


to abelian group of order 6 which is using result (i)).

Also we can check order structure of both groups and .

Elements of order 3 in are , Element of order 2 in is .

Elements of order 6 in are , which is same as order structure of

( have one element of order 2 , two elements of order 6 and two elements of order 3).

(i) }

Which is an abelian(Non-cyclic) group of order 8 with each non-identity element is of order 2.

This group has 7 subgroups of order 2 while no cyclic subgroup of order 4, as there is no element
of order 4 exists in the group.

(ii) Take then

Now we will discuss order structure of whether it is isomorphic to .

Elements of order 2 are ;

Elements of order 3 are ;

Elements of order 6 are ;

As has two elements of order 6 so it cannot be isomorphic to to , Now if we see order


structure of which has 7 elements of order 2, 2 elements of order 6 , 2 elements of order 3
along with identity element so given group is isomorphic to .
P a g e | 137

(iii) Let be any group and } then is subgroup of called


Diagonal of . If where is additive abelian group then is plane
and is the line

Internal Direct Product:


Let be a group and be normal subgroups of such that

(i)
(ii) Any given then with in a unique way.

We then say, is internal direct product of

Example:
1. Take ⟨ ⟩ ⟨ ⟩ we can easily see that is an internal
direct product of because both are normal subgroup of and also we can list
unique factorization of elements of

Results:

1. If is an internal direct product of Then for } and if


then

2. A group is internal direct product of normal subgroups if and only if


and }

3. Let be an internal direct product of then


P a g e | 138

Q:1 If then number of elements of order in are

(a) (b) (c) (d)

Explanation:
For any | | we have to find such that | |
when | | | | in this case

when | | | |
So there are total 12 elements.
Option (a) is correct.

Q:2 If then number of subgroups of order in

(a) (b) (c) (d)

Explanation:
To find number of subgroups of order we have to find number of elements of order
The elements of order are , and Number of subgroups(cyclic) of order
are also
Option (a) is correct.

Q:3 If and is a subgroup(if exists) of order then

(a) (b) (c) Both are possible (d) None of these

Explanation:
Firstly, we have to check order of elements of group , if there exist an element of order
in then may be cyclic of order
But there is no element of order in
So, is non-cyclic and since, Converse of Lagrange‟s Theorem holds in finite abelian
groups so exists and is non-cyclic abelian group of order
Thus
Option (b) is correct.
P a g e | 139

Q:4 | |

(a) (b) (c) (d)

Explanation:
By a theorem
Thus | |
Option (c) is correct.

Q:5 Order of is

(a) (b) (c) (d)

Explanation:
Order of | | | |
Option (a) is correct.

Q:6 How many subgroups of order in ?

(a) (b) (c) (d)

Explanation:
There are two types of groups of order cyclic groups and elementary abelian groups.
Thus, if is generator for cyclic group of order in and is generator for
the subgroup of order cyclic subgroups of order are generated by elements of
order
Since, there is a unique element of the form in each such subgroup.
It follows that there are cyclic subgroups of order
The subgroup of elements of order dividing is generated by and and so has
elements.
This is therefore the unique elementary abelian subgroup of order
Thus, there are subgroups with elements.
Option (a) is correct.
P a g e | 140

Q:7 The order of in

(a) (b) (c) (d)

Explanation:
The order of is and order of in is
So, order of is being of and
Option (d) is correct.

Q:8 has _______ elements of finite order

(a) (b) (c) No element (d)

Explanation:
has only one element of finite order, while other two groups are finite groups.
So, only element of finite order are

Option (a) is correct.

Q:9 Which of the following is true?

(a) (b)
(c) (d)

Explanation:
Since, direct product of two abelian groups is abelian and only if

So, option (d) is true.

Q:10 How many subgroups of are isomorphic to

(a) All (b) only (c) (d) no subgroup


P a g e | 141

Explanation:
Since, is isomorphic to so, only one subgroup of is isomorphic to , which is
itself.
Option (b) is correct.

Q:11 _______

(a) (b) (c) (d)

Explanation:
Since, } and } and in both groups each non-identity
element is of order 2, so both are isomorphic to
Option (d) is correct.

Q:12 Let p be a prime, then has exactly ______ subgroups of order p.

(a) (b) (c) (d)

Explanation:
By a well-known result, we know that has exactly subgroups of order .
Option (c) is correct.

Q:13 Which of the following group is non-cyclic?

(a) (b) (c) (d)

Explanation:
is cyclic group of order 14, is also cyclic group of order 120 and
also is cyclic of order 403.
and hence
which is non-cyclic.
Option (b) is correct.
P a g e | 142

Q:14 Choose the most appropriate statement.


(a) Every abelian simple group is of prime order.
(b) is not simple.
(c) Direct product is commutative but not associative.
(d) In symmetric groups

Explanation:
As we know that, only simple abelian groups are of prime order.
Hence, (a) is most appropriate statement.
Direct product is both commutative and associative, also .
Moreover, is simple group as every group of prime order is simple.
Option (a) is correct.

Q:15 Let = and }. The quotient group is isomorphic to

(a) 8 (b) 4  2 (c) 2  2  2 (d) The dihedral group D4

Explanation:
is of order and the factor group is order 8.
Now to determine whether is cyclic or non-cyclic.
For this we will write the elements of and then finding orders of elements.
There are 8 distinct cosets:
, , , , , ,
, , as because
So, is cyclic group of order since, it has an element of order
Option (a) is correct.

Q:16 How many cyclic subgroups of order in ?

(a) (b) (c) (d)

Explanation:
The number of cyclic subgroups of order in are .
Option (d) is correct.
P a g e | 143

Q:17 Choose the false statement.

(a) Every cyclic group can be written as an internal direct product of its two proper
normal subgroups.
(b) Every subgroup of is of the form where
(c) Direct product is commutative.
(d) All of these

Explanation:
All cyclic groups cannot be written as an internal direct product of its two proper normal
subgroups as cyclic group of prime order have no two proper normal subgroups.
Direct product of any two groups is commutative.
Every subgroup of direct product of two groups is not of the form
where
For example has a subgroup } which is not direct product
of a subgroup of with a subgroup of
Option (b) is correct.

Q:18 Which of the following group is not internal direct product of any two of its proper
subgroups?

(a) (b) (c) (d)

Explanation:
Take two proper subgroups } } of
Then we can always write as an internal direct product of as both are normal
subgroup and every element of has unique expression that we have already discussed
above. There exist two proper normal subgroup }
} of such that } and
We know very well that is an internal direct product of two its normal proper
subgroups of order
Dihedral group of order cannot be written as an internal direct product of its two
proper normal subgroup.
If we talk about subgroups of order then it is impossible for subgroups to be
disjoint.
Option (c) is correct.
P a g e | 144

Q:19 Number of elements of order in are

(a) (b) (c) (d)

Explanation:
For any two groups ( ) ( ) where
We find elements of order in so that we can find elements of order in

Elements of order in } are


Elements of order in } are
Now, elements of order in are

Thus, there are total elements of order in


Option (a) is correct.

Q:20 For a group which of the following must be a subgroup of ?

(a) } (b) }
(c) } (d) All of these

Explanation:
We check each of the following subset using subgroup criteria.
Given } For any
So,
Take

So,
Take
So,
Option (d) is correct.
P a g e | 145

Answers:

Q1 a Q2 a Q3 b Q4 c Q5 a
Q6 d Q7 d Q8 a Q9 d Q10 b
Q11 d Q12 c Q13 b Q14 a Q15 a
Q16 d Q17 b Q18 c Q19 a Q20 d
P a g e | 146

Classification of Finite Abelian Groups

Suppose is a group of order | | and we are to find non-isomorphic abelian groups


of order
There are three abelian groups of order 8 that are non-isomorphic.
Except these three if we have an abelian group of order 8 in any structure, it will be isomorphic to
one of these three abelian groups.
Now, we discuss the technique to find the number of non-isomorphic abelian groups of any finite
order.

Technique:
n  p11 . p2 2 ... pk k
No. of non-isomorphic abelian groups of order
where = No. of positive integer partitions of
where are positive integers.
First of all, we find prime factorization of After this we find all partitions of the powers of
prime
numbers.
We count the number of positive integers partitions of powers of prime and then multiply these.
This is the way to find the Number of non-isomorphic abelian groups.

Positive Integer Partitions:

Non-Isomorphic Abelian Group Of Some Order:


1. For Now we will check number of partitions of
Non-isomorphic abelian group of order

P Partition of  Groups
P a g e | 147

2. For
Now, we will check number of partitions of
Non-isomorphic abelian group of order

P Partition of  Groups

3. For
Now, we will check number of partitions of and

Non-isomorphic abelian group of order

P Partition of  Groups

Now multiply

4. For
Now, we will check number of partitions of

Non-isomorphic abelian group of order

P Partition of  Groups

Now multiply
= ,
= ,
P a g e | 148

If we find these groups in another method, they may be seeming to be different but they are
isomorphic to one of these groups.
For example one abelian group of order may be but this is isomorphic to one of
these four groups.

5. For
Now, we will check number of partitions of

Non- isomorphic abelian group of order

P Partitions of  Groups

Now multiply

6. For
Now, we will check number of partitions of

Non- isomorphic abelian group of order

P Partitions of  Groups

Now multiply,

These are non-isomorphic abelian groups of order


P a g e | 149

7. For
Now, we will check number of partitions of

Non- isomorphic abelian group of order

P Partitions of  Groups

23 3,1+2,1+1+1

53 3,1+2,1+1+1

Now multiply

These are non-isomorphic abelian groups of order

Results:
1. The number of non-isomorphic groups of order is at most
2. Every finitely generated abelian group is isomorphic to direct product of its cyclic
subgroups. This is known as „‟Fundamental Theorem of Finitely Generated Abelian
Groups‟‟.
3. If divides the order of a finite abelian group then has a subgroup of order
4. Let be relatively prime positive integers. If there are non-isomorphic abelian
groups of order and of order then there are non-isomorphic abelian groups of
order
5. A finite abelian group is not cyclic if and only if it contains a subgroup isomorphic to
for some prime
6. Let be a finite abelian group of order where is a prime number that does not
divide Then where { } }
Moreover, | |
7. An abelian group of order must have odd number of elements of order
P a g e | 150

Q:1 Number of non-isomorphic abelian groups of order

(a) (b) (c) (d)

Explanation:
Here,
We will check number of partitions of

Non-isomorphic abelian groups of order


Since every group of order is abelian.
One is cyclic and other is abelian .
So, option (a) is right.

Q:2 Non-isomorphic abelian groups of order 360?

(a) (b) (c) (d)

Explanation:
Here,
Taking product of number of partitions of

Option (b) is correct.

Q:3 Non-isomorphic abelian groups of where, are distinct primes.

(a) (b) (c) (d)

Explanation:
are distinct primes.
Taking product of number of partitions of

Option (a) is correct.


P a g e | 151

Q:4 Non-isomorphic abelian groups of order are

(a) (b) (c) (d)

Explanation:
(10)5   2.5   (2)5 (5)5
5

Taking product of number of partitions of

Option (d) is correct.

Q:5 Non-isomorphic abelian groups of order are

(a) (b) (c) (d)

Explanation:
are distinct numbers.
Taking product of number of partitions of

Option (b) is correct.

Q:6 There exist exactly one abelian group of order?

(a) (b) (c) (d)

Explanation:
Since, is a prime number and a group of prime order is unique and being cyclic it is
abelian.
so, there are abelian groups.
so there are also more than one abelian groups.
so there are also more than one abelian groups.
So, option (a) is correct.
P a g e | 152

Q:7 How many abelian groups of order exist?

(a) Exactly one (b) More than


(c) More than 2 (d) Exactly two

Explanation:
is a product of two distinct primes.

Since, for two distinct primes “ ” and “ ” if | | where and


then is unique cyclic group of order .
So, option (a) is correct.

Q:8 For which of the following order, every group is abelian?

(a) (b) (c) (d) All of these

Explanation:
is non-abelian group of order
is non-abelian group of order
Since, and ⁄ (using result if order of group is for distinct

Primes if and and divides then there must be non-abelian group of


order .
So, option (a) is correct.

Q:9 Pick out the true statement.

(a) Every group of order is abelian.


(b) Every group of order is non abelian.
(c) A group in which every element has at most order is abelian.
(d) None of these.

Explanation:
is a non-abelian group of order
is an abelian group of order
A group having every element of at most order must contain one element of order one
identity and remaining element of order
A group having all element of order (obviously except identity) is abelian.
So, option (c) is correct.
P a g e | 153

Q:10 Number of non-isomorphic abelian groups of order is/are _____

(a) (b) (c) (d)

Explanation:
We know that Since, number of partition of is
So, number of non-isomorphic abelian group =
Option (d) is correct.

Q:11 The groups and are

(a) Isomorphic abelian groups (b) Non-isomorphic abelian groups


(c) Non-cyclic abelian groups (d) Non-isomorphic non-cyclic groups

Explanation:
For any positive integer is cyclic group.
For two cyclic groups if and only if
Option (b) is correct.

Q:12 Which of the following groups are non-isomorphic?

(a) (b) (c) (d) None

Explanation:
There are three non-abelian groups of order that are
There exists an element of order in but not in
We can easily check element structure of groups to show that which we
have already discussed. Also,
Option (b) is correct.
P a g e | 154

Answers:

Q1 a Q2 b Q3 a Q4 d

Q5 b Q6 a Q7 a Q8 a
Q9 c Q10 d Q11 b Q12 b
P a g e | 155

Sylow Theory

Cauchy Theorem:
Let be a finite group and a prime divides the order of , then has an element of order

Examples:
Consider | | then divides order of | | , so by Cauchy Theorem has an element of
order . Moreover has a cyclic subgroup of order 2.

Now we see above in all group of order

Abelian Groups of order are .

} } } , and of order
respectively.

Non abelian groups of order 8 are and , then } } are subgroups of order 2.

Sylow Subgroup:
A subgroup of a group is said to be a sylow subgroup (where is divisor of order of if
order of is where divides order of but does not divide order of

Examples:
1. Take }, subgroups of are } } } } is
prime divisor of subgroups of divides | | . Also divides | | but does
not divide order of group. So, sylow subgroup of is of order .
2. Take } subgroups of are
} } } } } }. are prime divisors of
order of subgroups of So, sylow subgroup of is }. Also sylow
subgroups of are } } }
3. Take } subgroups of are
} } } } } } } } }.
P a g e | 156

is prime divisor of order of subgroups of So, sylow subgroup of is of


order
4. Take

}
Subgroups of are
} } } }
} } } } are prime divisors
of subgroups of So, sylow subgroup is of order . Also sylow
subgroups of are
} } } }

Sylow 1st Theorem: (Existence Theorem)


Let be a finite group and be a prime , if divides | | and does not divide | | , then has
a subgroup of order called Sylow subgroup of .

Sylow 2nd Theorem:


Any two Sylow subgroups are conjugate to each other.

Sylow 3rd Theorem:


The number of Sylow subgroups “ ” are of the form and divides | |.

Examples:
1. Let | | then divides and does not divide so has a Sylow subgroup
of order . Moreover divides only when so , Sylow
subgroup of order is unique.
2. Let | | then divides and does not divide so has a Sylow subgroup of
order . Moreover divides when or so number of
Sylow subgroups In case ), . Also divides and
does not divide so has a Sylow subgroup of order The number of Sylow
subgroups so when divides , hence .

3. Let | | then divides and does not divide so has a Sylow subgroup
of order . Moreover divides only when so , Sylow
subgroup of order is unique. Since is a group so can generalize a result that when
is a group of order then has a unique Sylow subgroup of order . is the
sylow subgroup of itself.
P a g e | 157

4. Let | | then divide and does not divide so has a Sylow


subgroup of order . Then divides when
so or . Also divide and does not divide so has a Sylow subgroup
of order . Then divides when
so or

5. Let | | then has a Sylow subgroup of order Then


divides when so Also has a Sylow subgroup of order Then
divides when so .

6. Let | | then has a sylow subgroup of order is its sylow


subgroup hence unique.

7. Let | | then has sylow subgroup of order divides


so, or or Also has sylow subgroup of order
divides so, unique sylow subgroup of order

8. Let | | then has sylow subgroup of order divides


so, or Also has sylow subgroup of order
divides so, unique sylow subgroup of order

9. Let | | then has sylow subgroup of order divides


so, or Also has sylow subgroup of order
divides so, unique sylow subgroup of order

10. Let | | then divide and does not divide so has a Sylow
subgroup of order . Then divides when
so or . Also divide and does not divide so has a Sylow subgroup
of order . Then divides when
so or

11. Let | | then has sylow subgroup of order divides


so, or Also has sylow subgroup of order
divides so, sylow subgroup is unique.

12. Let | | then divide and does not divide so has a Sylow
subgroup of order . Then divides when
so or or . Since divide and does not divide so has a Sylow
subgroup of order . Then divides when
so .
P a g e | 158

13. Let | | then has a sylow subgroup of order


divides when so, or Also has sylow subgroup of order
divides when so, or
14. Let | | then has a Sylow subgroup of order . Then
divides when so or . Also has a Sylow subgroup of order
Then divides when so .

15. Let | | then divide and does not divide so has a Sylow
subgroup of order . Then divides when
so or or . Also divide and does not divide so has a Sylow
subgroup of order . Then divides when
so or or . Since divide and does not divide so has a Sylow
subgroup of order . Then divides when
so or .

16. Let | | then has a Sylow subgroup of order .Then


divides when so Also has a Sylow subgroup of order Then
divides when so .

17. Let | | then has a Sylow subgroup of order . Then


divide when so Also has a Sylow subgroup of order Then
divides when so .

18. Let | | then has sylow subgroup of order divides


so, or 7. Also has sylow subgroup of order
divides so, unique syow subgroup. Similarly, has unique
sylow subgroup of order

19. Let | | and has a Sylow subgroup of order . Then


divides when
so or or . Also has a Sylow subgroup of order . Then divides
when
so or has a Sylow subgroup of order . Then divides
when
so or .

20. Let | | then has sylow subgroup of order


divides so, unique sylow subgroup of order Also has
sylow subgroup of order divides so, unique
P a g e | 159

sylow subgroup of order Also has sylow subgroup of order


divides so, unique sylow subgroup.

21. Let | | then has sylow subgroup of order


divides so, or or or or Also has sylow
subgroup of order divides so, or or Also
has sylow subgroup of order divides so, or

22. Let | | then has sylow subgroup of order


divides so, or or Also has sylow subgroup of order
divides so, or

23. Let | | then has sylow subgroup of order


divides so, or or or or . Also has
sylow subgroup of order which is unique being of index

24. Let | | thenhas sylow subgroup of order


divides so, unique sylow subgroup of order Also has
unique sylow subgroup of order and unqiue sylow subgroup of order

25. Let | | then has sylow subgroup of order


divides so, unique sylow subgroup of order Also
has sylow subgroup of order divides so,
unique sylow subgroup of order

Results:
 Let be a Sylow subgroup of a group , then followings are equivalent.
 is unique Sylow subgroup of .
 is normal in .
 is characteristic in .
 Let be a Sylow subgroup of then is the only Sylow subgroup of
contained in .
 Let be a group of order where and are distinct primes and then the Sylow
subgroup is normal in .
 Let be an odd integer greater than then number of Sylow subgroups
of are , where | | .
 Let be a group of order , where and are distinct primes and if does not divide
and does not divide then is abelian.
P a g e | 160

 Let be a group of order , where are distinct primes. If are two distinct
sylow subgroups of then is normal subgroup of
 Suppose that is finite group and has a unique Sylow subgroup for each prime
then is the internal direct product of its non-trivial Sylow subgroups and if each
Sylow subgroup is cyclic then is cyclic.
 Let be prime and and be Sylow subgroups of then
 Let be a group of order , where are distinct primes and if does not divide
and does not divide then is abelian.
 Let be a normal subgroup of a group , then is the union of the conjugacy classes of
the elements of in
 Let be a non-abelian group of order where are distinct primes and then
has exactly non-trivial proper subgroups.
 Let be a group of order then
i. has normal sylow subgroup.
ii. has a normal subgroup of order
iii. has normal sylow subgroup if does not divide
 Let be the normal Sylow subgroup of and let be any subgroup of then
is the unique Sylow subgroup of
 If | | where is odd and has a cyclic Sylow subgroup then has a
normal subgroup of order
 Sylow subgroup of ( ) is of order
 If is a Sylow subgroup of a finite group and is any subgroup of then is
contained in
 If is a Sylow subgroup of a group and is any Normal subgroup of then

is a Sylow subgroup of .

is a Sylow subgroup of

 If is a normal subgroup of a group and is a Sylow subgroup of then


.
 If a group has a subgroup containing the normalizer of Sylow subgroup of then

 If is a finite group in which every Maximal subgroup is normal , then Sylow


subgroup of is normal in
 The number of Sylow subgroups in a group is where is any Sylow
subgroup of
 Let be a finite group in which for every where is a prime
dividing the order of group then
P a g e | 161

i.The sylow subgroup of is normal in and hence unique.


ii.If is a sylow subgroup of then there exists a normal subgroup of
with } and
 An abelian group is isomorphic to the direct product of its Sylow subgroups.

Results (Simple Groups):


1. Let be the non-prime positive integer and be prime divisor of
If is the only divisor of that is equal to modulo then there does not exist a simple
group of order .
2. Let be the odd integer greater than then a group of order is not simple.
3. If is a finite group and is a proper subgroup of such that | | does not divide
[ ] then contains a non-trivial normal subgroup of
Hence is not simple (Index Theorem)
4. There is no simple group of order where are distinct primes.
5. Let be a group of order where are primes and then
i. is not simple group as it has a normal subgroup of order
ii. is cyclic if does not divide
6. There is no simple group of order where are primes.
7. Any group of order is not simple, where is a prime.
8. There is no simple group of order for any integer
9. The only simple abelian groups are of prime order.
10. A finite group with precisely Sylow subgroups is not Simple or has a Sylow
subgroup of order .
11. If a finite non-abelian simple group has a subgroup of index , then is isomorphic to
a subgroup of (Embedding Theorem)
12. Any simple group of order less than is of prime order.
13. All cyclic groups of prime order are simple.
14. For two distinct primes a group of order is not simple.
15. If a group has a subgroup of index or then group is not simple.
16. Alternating group simple.
17. If is a finite non-abelian simple group and is a subgroup of [ ] .
18. If then has no non-abelian simple subgroups.
19. are not simple groups.
20. are simple groups except for
21. If are sylow subgroups of then
is sylow subgroup of . Similarly, if
is unique sylow subgroup of then is unique
sylow subgroup of .
P a g e | 162

22. If is normal sylow subgroups of then


is normal sylow subgroup of

Applications:
1. Groups of order
are Simple
since these are groups of prime order.
2. Groups of order are not simple as they are abelian groups and only abelian
simple groups are of prime order.
Note: Now we will discuss simplicity of non-abelian groups only.
3. The Non abelian group of order is is not simple, as has a unique Sylow
subgroup of order 3 which is normal, also by result .
4. Groups of order are not simple by result .
5. Groups of order
are not simple
by using result
6. For a group of order there are unique Sylow and Sylow subgroups so is
not simple group by result
7. Groups of order are not simple using result .
8. Group of order is not simple as it has a unique Sylow subgroup of order by
result
9. Group of order is not simple as it has or Sylow subgroup and or Sylow
subgroups. If has Sylow or Sylow subgroups then is not Simple and for
other the result follows.
10. Group of order is not Simple as it has a unique Sylow subgroup of order by
result
11. Group of order is not simple as it has a proper subgroup for which | | and
| | does not divide [ ] So is not Simple. Also follow by result
12. Group of order is not simple as it has a unique Sylow subgroup of order by
result
13. Group of order is not simple as it has a unique Sylow subgroup of order by
result
14. Group of order is not simple as it has a unique Sylow subgroup of order by
result
15. Group of order is not simple as it has a unique Sylow subgroup of order by
result
16. Group of order is not simple as it has a proper subgroup for which | | and
| | does not divide [ ] So is not Simple
P a g e | 163

17. Group of order is not simple as it has a unique Sylow subgroup of order by
result
18. Group of order is not simple as it has a unique Sylow subgroup of order by
result
19. Group of order is not simple as it has a unique Sylow subgroup of order by
result
20. Group of order is not simple as it has either or Sylow subgroups if has
Sylow subgroup then is not simple and if has Sylow subgroups then by
result is not Simple.
21. Group of order is not simple as it has a unique Sylow subgroup of order by
result
22. Group of order is not simple as it has a unique Sylow subgroup of order ,
Also by result
23. Group of order is not simple as it has a unique Sylow subgroup of order by
result
24. Group of order is not simple as it has a unique Sylow subgroup of order by
result
25. Group of order is not simple as it has a unique Sylow subgroup of order by
result
26. Group of order is not simple as it has either or Sylow subgroups of order .
If has Sylow subgroup then is not Simple and if has Sylow subgroups then
index of Normalizer of any Sylow subgroup in is which is not possible as does
not divide .
27. Group of order is not simple as it has a unique Sylow subgroup of order by
result
28. Group of order is not simple as it has a unique Sylow subgroup of order by
result
29. Group of order is not simple as it has a unique Sylow subgroup of order by
result
30. Group of order is not simple by result
31. Group of order is not simple as it has a unique Sylow subgroup of order clearly
by sylow third theorem also by result
32. Group of order is not simple as it has a unique Sylow subgroup of order by
result
33. Group of order is not simple as it has a unique Sylow subgroup of order by
result
34. Group of order is not simple as it has a unique Sylow subgroup of order by
result
35. Group of order is not simple as it has a unique Sylow subgroup of order by
result
P a g e | 164

36. Group of order is not simple as it has a unique Sylow subgroup of order by
result
37. Group of order is not simple as it has a unique Sylow subgroup of order .
38. Group of order is not simple as it has a unique Sylow subgroup of order by
result
39. Group of order is not simple as it has a proper subgroup for which | | and
| | does not divide [ ] So is not Simple.
40. Group of order is not simple as it has a unique Sylow subgroup of order as it has
a proper subgroup for which | | and | | does not divide [ ] .
41. Group of order is not simple as it has a unique Sylow subgroup of order by
result
42. Group of order is not simple as it has a unique Sylow subgroup of order as it
has a proper subgroup of order for which order of group does not divide [ ]
43. Group of order is simple as group of prime order is simple.
44. Group of order is not simple by result
45. Group of order is simple group.
46. Group of order is not simple by result
47. Group of order is simple group by result as | | .
48. Group of order is not simple by result
49. Group of order is not simple group by result
50. Group of order is not simple by result
51. Group of order is not simple by result
P a g e | 165

Q:1 Let | | and is subgroup of then which is true about


(a) is a Sylow subgroup of order
(b) is a Sylow subgroup of order
(c) is a Sylow subgroup of order
(d) is a Sylow subgroup of order

Explanation:
Since | | and has Sylow subgroup of order , Sylow subgroup of
order
So is a Sylow subgroup of order
Option is correct.

Q:2 Point out the order of group for which is not Simple

(a) (b) (c) (d)

Explanation:
Since then any group of order is not simple while is odd.
While are primes so these are order of simple groups.
Option is correct.

Q:3 Indicate the group which has unique Sylow subgroup

(a) (b) (c) (d)

Explanation:
The group has a unique Sylow subgroup which is , while does not have a
unique Sylow subgroup , similarly don‟t have and cannot have a subgroup of
order
Option is correct.

Q:4 The Sylow subgroup of is of order

(a) (b) (c) (d)


P a g e | 166

Explanation:
Since Sylow subgroup of ( ) is of order so here using
values we get order of Sylow subgroup which is
Option is correct.

Q:5 There is no simple group of order

(a) (b) (c) (d) All of these

Explanation:
As there is no simple group of order where ,
Here
So these cannot be orders of simple groups.
Option is correct.

Q:6 Which of the following group has no non abelian simple subgroup?

(a) (b) (c) (d) None of these

Explanation:
is simple in and smallest non abelian simple group is so does not
contain any non-abelian simple subgroup.
Option is correct.

Q:7 Let be a group of order 125 and be a subgroup of then which is accurate

(a) None of these is accurate

Explanation:
If is a -group and is any subgroup of then
Here is a group so the result follows.
Option is correct.
P a g e | 167

Q:8 Choose the correct statement

(a) has a non-abelian simple subgroup


(b) There is only one group of order 45
(c) Every group is its own Sylow subgroup
(d) There must exist a non-abelian simple group of order

Explanation:
The smallest non abelian group is of order .
There are two groups of order and .
The group of order is not simple as
Also every group is its own Sylow subgroup.

Q:9 There must exist a non-abelian simple group of order

(a) (b) (c) (d)

Explanation:
The group of order is not simple as its order is where is odd.
Similarly group of order is not simple.
The group of order is not simple as by Sylow rd Theorem has a unique Sylow
subgroup or has Sylow subgroups.
If has a unique Sylow subgroup then is not simple.
Now consider has Sylow subgroups.
Then is the index of Normalizer of Sylow subgroup.
But then by embedding theorem is isomorphic to a subgroup of of order 120 , but
has no subgroup of order .
Finally we have as order of a non abelian Simple group
Infact this group is ( )

Q:10 Which of the following two subgroups of are conjugate to each other

(a) =e ,
(c) =e
P a g e | 168

Explanation:
As the conjugate subgroups are always isomorphic so subgroups in option (a) , (b) , (c)
cannot be conjugate subgroups of as their order is not same.
The subgroups =e are conjugate in

Q:11 How many sylow-2 subgroups of ?


(a) (b) (c) (d)

Explanation:
The group a unique Sylow subgroup and Sylow subgroups of order

Q:12 Which of the following group has a unique Sylow subgroup

(a) (b) (c) S3 (d) Both (a) and (b)

Explanation:
Both Groups and are groups for and every group is the
Sylow subgroup of itself which is unique.

Q:13 Let and is a subgroup of then

(a) (b) (c) } (d)

Explanation:
Since is the unique Sylow subgroup of so it is normal then

Q:14 How many Sylow subgroups of order ______ exist in a group of order ?

(a) Unique, 127 (b) , 63 (c) , 127 (d) , 127

Explanation:
then has sylow subgroup of order as divides
But does not divide order of group. divides order of group
only if So, unique sylow subgroup of order 127.
P a g e | 169

Q:15 Let , be its Sylow subgroup of order and } be its


normal subgroup then be Sylow subgroup of

(a) (b) (c) (d) Both (b) and (c)

Explanation:
If is a Sylow subgroup of a group and is any Normal subgroup of then
is a Sylow subgroup of using this result for then

Q:16 Take and } be Sylow subgroup of and be


normal subgroup of then is Sylow subgroup of

(a) (b) (c) (d) None

Explanation:
If is a Sylow subgroup of a group and is any Normal subgroup of then

is a Sylow subgroup of . Here then is the

Sylow subgroup of

Q:17 Indicate the group for which there exists a sylow subgroup and a subgroup
such that is not a Sylow subgroup of .

(a) A4 (b) S4 (c) A5 (d) All of these

Explanation:
Take then there exists a sylow subgroup } and a
subgroup } but } is not sylow subgroup of
Take then there exists a sylow subgroup } and a
subgroup } but } is not sylow
subgroup of
Take then there exists a sylow subgroup
} and a subgroup

}
then } is not sylow subgroup of
All are correct options.
P a g e | 170

Q:18 A normal subgroup is contained in

(a) every subgroup (b) every sylow subgroup (c) normal subgroup (d) all of these

Explanation:
We have a result “A normal subgroup is contained in every sylow subgroup‟‟

Q:19 How many Sylow subgroup of ?

(a) (b) unique (c) (d)

Explanation:
} since is a group so it has a unique
Sylow subgroup.

Q:20 The Sylow subgroup of is isomorphic to

(a) (b) (c) (d)

Explanation:
Sylow subgroup of
Sylow subgroup of Sylow subgroup of

Q:21 Which of the following is true?

(a) Every subgroup of every finite group is a Sylow subgroup


(b) The normalizer in of a subgroup of is always a normal subgroup of
(c) A group of prime-power order p n has no Sylow subgroup
(d) Every Sylow subgroup of a finite group has order

Explanation:
Every subgroup of a finite group need not to be Sylow subgroup like in
groups the group itself is Sylow subgroup although it has many other
subgroups.
For normalizer take } then H is self-normalizing subgroup and it is
not normal in
Every group of order is the group so it is the Sylow subgroup of itself and for
any group order of Sylow subgroup is the by definition.
P a g e | 171

Q:22 The order of a normal subgroup of a group of order G  2500

(a) (b) (c) (d) None

Explanation:
Since then Sylow subgroup is unique and of order so being
unique it is normal.

Q:23 Which of the following is not possible order of unique sylow p-subgroup of

(a) 8 (b) 3 (c) 4 (d) All above

Explanation:
Since, | | then there exists sylow subgroup of order or 3.
These are }
}
}
Also there exists sylow subgroup of order 3. or These are
} } } }
So, sylow subgroup of order is not unique.
There does not exist sylow subgroup of order

Q:24 Which of the following statement is false?

(a) Every group whose order is the power of a prime has a non-trivial center.
(b) Every group of under p 2 is cyclic, where is a prime number
(c) Every non-abelian group of order is isomorphic to S3
(d) Every finite group of order is isomorphic to a subgroup of Sn

Explanation:
The center of finite groups is non-trivial.
There are two non-isomorphic classes of groups of order , Cyclic , Non cyclic
abelian , Also There are two non-isomorphic classes of groups of order ,
Cyclic , Non-abelian , so every non abelian group of order is isomorphic to
.
Last statement follows from Cayle‟s Theorem.
P a g e | 172

Q:25 Let be a group. Suppose | | where and are distinct primes.


Then which of the following statement is always true?

(a) has more than one sylow subgroup.


(b) has at least one normal sylow subgroup.
(c) The number of sylow subgroups of is divisible by .
(d) is simple group.

Explanation:
Let be a group of order where and are distinct primes.
Then has at least one normal sylow subgroup.
In other words is not simple.

Q:26 If and respectively denote the permutation group and alternating group, then
which is false?

(a) A3 is the sylow subgroup of S3


(b) Sylow subgroup of S3 is unique
(c) {I , (12)},{I , (12)}{I , (23)} are sylow subgroup of S3
(d) A3 is a normal subgroup of S3

Explanation:
has the unique Sylow subgroup so it is normal.
Meanwhile there are Sylow subgroup of .
Option is correct.

Q:27 Under what condition, a subgroup of a group self-normalizes?

(a) If is unique sylow subgroup of


(b) If is contained in some sylow subgroup of
(c) If contains normalizer of some sylow subgroup of .
(d) If is any sylow subgroup of

Explanation:
A subgroup of a group is said to be self-normalized if .
Take , , a unique sylow subgroup of but
Now, take , which is contained in sylow subgroup of
but .
Similarly, if we take sylow subgroup of then it does not self-normalize.
We have a result “A subgroup of a group is self-normalized if contains
normalizer of some sylow subgroup of ‟‟.
P a g e | 173

Q:28 If index of a subgroup in a group is then choose the correct.

(a) must be a simple group of order


(b) must be non-simple group of order
(c) must be a non-simple group of order
(d) must be a group non-simple group whose order | | | |

Explanation:
A group of order is not simple as it contains a subgroup of order
such that [ ] A subgroup of order is not simple as it contains a
subgroup of order whose index in is
We have a result “If a group contains a subgroup such that [ ] then must
be non-simple group‟‟.
Option (b), (c) are also correct but our best option is (d) which covers both of these.

Q:29 Which of the following statement is true about a group of order

(a) is a simple group.


(b) has unique sylow subgroup.
(c) Sylow subgroup of is not unique.
(d) Sylow subgroup is not unique.

Explanation:
Since, | | then has normal sylow subgroup which is
unique being normal. Since does not divide so, has normal sylow
subgroup which is also unique being normal. Since, has a proper normal
subgroup so, is not simple.
For a group of order , following results hold
i. has normal sylow subgroup.
ii. has normal subgroup of order
iii. has normal sylow subgroup if does not divide

Q:30 Choose the correct statement.

(a) For any fixed prime , all sylow subgroups are isomorphic.
(b) There must exists a non-abelian group of order .
(c) There must exists non-isomorphic sylow subgroup of
(d) None of these
P a g e | 174

Explanation:
For a fixed prime all sylow subgroups are isomorphic.
All sylow subgroups of order are isomorphic that are in numbers.
} } } }
“If | | , where are distinct primes. If does not divide and does
not divide then must be abelian‟‟.
Since, | | that satisfies above condition so, any group of order
must be abelian.

Q:31 Number of sylow subgroup in are ________

(a) (b) (c) (d)

Explanation:
Sylow subgroups of are
} } } } }

} } } } }
There are total sylow subgroups.

Q:32 Which of the following subgroup of is self-normalized?

(a) Sylow subgroup (b) Subgroup of order


(c) Sylow subgroup (d) Not possible

Explanation:
Since, | | , divides only if so,

unique sylow subgroup.


Being unique it is normal and normalizer of a normal subgroup is entire group.
So, sylow subgroup is not self-normalized.
In a dihedral group, subgroup of rotation is always normal.
So, a subgroup of order is not self-normalized subgroup of .
Now, we have a result “In a dihedral group where is odd integer, any sylow
subgroup is self-normalized that is .
P a g e | 175

Q:33 If is a dihedral group of order and denotes sylow subgroup then choose
the correct statement.
(a) (b)
(c) (d)

Explanation:
We have a result “If is a group and is smallest prime divisor of order of group.
Also if denotes the sylow subgroup then .
Since, in is odd number all sylow subgroups are self-normalized.
So,we can write as .

Q:34 Let be a non-abelian group of order and denotes the sylow subgroup
then | | | | _____

(a) (b) (c) (d) None of these

Explanation:
There exists only two groups of order one is cyclic and other is non-
abelian which is dihedral group .
Here which is odd prime so, all sylow subgroups of are self-
normalized. which implies | | .
Also since is smallest prime divisor of | |
So, . Thus, | | .
| | | |
| | | | | | | |

Q:35 Let be any group of order and is a subgroup of order then

(a) is simple group (b) is unique sylow subgroup.


(c) } (d) All of these
P a g e | 176

Explanation:
The subgroup is normal being of index
Being normal it is unique sylow subgroup.
Also, is order of largest proper normal subgroup of
So, is simple. Since is normal so, }.
If is group and is a proper normal subgroup of largest order then is simple.

Q:36 Let and be its subgroup then choose correct about sylow
subgroup and be subgroup of order 7
bw and sylow subgroup.
(a) | | (b) must be cyclic subgroup of
(c) must be normal subgroup of (d)

Explanation:
| | and | | which implies | | .
Since, order of is and only group of order is cyclic subgroup of .
Now, number of sylow subgroup or or or or .
We are not sure that it must be unique.
So, it may not be normal. is normal subgroup of . So, .

Q:37 Let and where | | | | then choose the correct

(a) is normal subgroup of but is not.


(b) is normal subgroup of but is not.
(c) Neither nor is normal in
(d) Both and are normal in

Explanation:
First of all, we check sylow subgroup.
Since sylow subgroup is unique in so, sylow subgroup is normal in
In , number of sylow subgroups are So, is not normal in
Now, we check sylow subgroup. Since, sylow subgroup is unique in .
So, sylow subgroup is normal in
Also, sylow subgroup is normal in . So, is normal subgroup of
P a g e | 177

Q:38 Take then choose the best option.

(a) order of sylow subgroup of is


(b) Sylow subgroup of is non-abelian.
(c) Sylow subgroup of is .
(d) All of these

Explanation:
We know that if is sylow subgroups of then
is sylow subgroup of .
Sylow subgroup of Sylow subgroup of Sylow subgroup of
Sylow subgroup of Sylow subgroup of which
is non-abelian.
Order of sylow subgroup of

Q:39 Which of the following must be unique sylow subgroup in corresponding group?

(a) Sylow subgroup in where | |


(b) Sylow subgroup in where | |
(c) Sylow subgroup in where | |
(d) Sylow subgroup in where | |

Explanation:
By sylow third theorem, number of sylow subgroup must divide which
is possible only when .
Thus, sylow subgroup in unique in a group of order
Now, number of sylow subgroup must divide which is possible only
when . Thus possible number of sylow subgroup is in a group of
order
Number of sylow subgroup in non-abelian group are
Number of sylow subgroup divide which is possible only for
Thus, possible number of sylow subgroup is .

Q:40 Which of the following group must have cyclic sylow subgroup of order ?

(a) (b) (c) (d)


P a g e | 178

Explanation:
Maximum order of element in is .
So, a cyclic sylow subgroup of order does not exist in
is sylow subgroup of itself which is non-abelian.
Sylow subgroup of which is abelian but non-cyclic. is itself
cyclic sylow subgroup of order

Answers:

Q1 D Q2 c Q3 a Q4 b Q5 d
Q6 A Q7 c Q8 c Q9 d Q10 d
Q11 C Q12 d Q13 b Q14 a Q15 d
Q16 C Q17 d Q18 b Q19 b Q20 b
Q21 D Q22 b Q23 d Q24 b Q25 b
Q26 B Q27 c Q28 d Q29 b Q30 a
Q31 B Q32 c Q33 d Q34 b Q35 d
Q36 B Q37 a Q38 d Q39 a Q40 d
P a g e | 179

Counting Of Group Homomorphisms

Method
Suppose we have to find all group homomorphisms from to . Write all normal subgroups
of then make factor groups

The number of group homo are | ( )| where represents number of type


subgroups in .

Example:1
Find all group homomorphisms from to .

Solution
Let‟s us find all normal subgroups of which are }

Now make factor groups }


. Which is not a subgroup of so there is no homomorphism
whose kernel is trivial.

For factor group } which is the unique subgroup in . So a homomorphism exists


whose kernel is . Which is trivial homomorphism so there is only trivial homomorphism exists
between and which is not one-one and onto.

Example:2
Find all group homomorphisms from to

Solution
The normal subgroups of are } and .

The factor group }


which is a unique subgroup of . So a unique homomorphism exists
whose kernel is } also it is one-one.

While the factor group } which is the unique subgroup in . So a unique homomorphism
exists whose kernel is . Hence there are 2 group homomorphism from to .

Example:3
Find all group homomorphisms from to
P a g e | 180

Solution
The normal subgroups of are } .

Factor group }
which is not a subgroup of . Hence there is no one-one homomorphism

Factor group which is also unique subgroup in so the number of homomorphisms


from to with kernel are | |

Factor group unique subgroup in so number of homomorphisms from to with


as kernel are | |

Also there is a trivial-homomorphism so there are total 8 group homomorphisms from to


are , out of which 2 are onto.

Example:4
Find all group homomorphisms from to

Solution
The normal subgroups of are }, and .

The factor group }


which is not a subgroup of so there is no one-one homomorphism.

While which is isomorphic to cyclic subgroups of order . Thus the number of


homomorphisms with as kernel is | |

Also the factor group } which is a unique subgroup in so a trivial homomorphism


exists. Thus there are total homomorphism from to .

Example:5
Find all group homomorphisms from to

Solution
The normal subgroups of are }

The factor group }


which is not a subgroup of So there is no homomorphism whose
kernel is trivial (no one-one homomorphism).

Also the factor group which is isomorphic to subgroups of order in hence there
are group homomorphism with as kernel.

} so there is a unique homorphism whose kernel is (Identity homomorphism).


P a g e | 181

Example:6
Find all group homomorphisms from to .

Solution
For three normal subgroups of we have the following factor groups,

}
which is not a subgroup of so there is no one-one homomorphism.

which is a unique subgroup of so there is a unique homomorphism with as


kernel. Also trivial homomorphism exists so there are total group homomorphisms from to
.

Example:7
Find all group homomorphism from to .

Solution
There are three normal subgroups of so the factor groups are following,

}
which is isomorphic to subgroups of and since | | so there are
group homomorphism from to which are one-one.

while subgroups of are isomorphic to so there are group homomorphisms


with as kernel.

Alongwith trivial homomorphism, there are total group homomorphisms.

Example:8
Find all group homomorphism from to .

Solution
There are seven normal subgroups of which are } where are three in
numbers. So the factor groups are following,

}
which is not a sub group of so there is no one-one homomorphism.

so no such homomorphism exist whose kernel is .

also there are three such subgroups of order in .

Hence number of homomorphism with kernel are

Alongwith identity homomorphism there are total group homomorphisms from to .


P a g e | 182

Information Table for


S:No Partition Type of the Name of the Number Order Even/Odd
element element of
elements
1. 6=1+1+1+1+1+1 1 Identity 1 1 Even
2. 6=1+1+1+1+2 (1 2) Transposition 15 2 Odd
3. 6=1+1+1+3 (1 2 3) 3-cycle 40 3 Even
4. 6=1+1+4 (1 2 3 4) 4-cycle 90 4 Odd
5. 6=1+5 (1 2 3 4 5) 5-cycle 144 5 Even
6. 6=6 (1 2 3 4 5 6) 6-cycle 120 6 Odd
7. 6=1+1+2+2 (1 2)(3 4) Product of 45 2 Even
transpositions
8. 6=2+2+2 (1 2)(3 4)(5 6) Product of three 15 2 Odd
transpositions
9. 6=1+2+3 (1 2)(3 4 5) Product of a 120 6 Odd
transposition &
a 3-cycle
10. 6=3+3 (1 2 3)(4 5 6) Product of two 40 3 Even
3-cycles
11. 6=2+4 (1 2)(3 4 5 6) Product of a 90 4 Even
transposition &
a 4-cycle

Information Table for


S:No Partition Type of the Name of the element Number of Order
element elements

1. 7=1+1+1+1+1+1+1 1 Identity 1 1
2. 7=1+1+1+1+3 (1 2 3) 3-cycle 70 3
3. 7=1+1+5 (1 2 3 4 5) 5-cycle 504 5
4. 7=7 (1 2 3 4 6 7) 7-cycle 720 7
5. 7=1+1+1+2+2 (1 2)(3 4) Product of 105 2
transpositions
6. 7=2+2+3 (1 2)(3 4)(5 6 7) Product of two 210 6
transposition &
a 3-cycle
7. 7=1+3+3 (1 2 3)(3 4 6) Product of two 280 3
3-cycle
8. 7=1+2+4 (1 2)(3 4 5 6) Product of a 630 4
transposition &
a 4-cycle
P a g e | 183

Table of Class Equation of Some Familiar Groups

Order Group Nature Class Equation


1 Abelian 1=1
2 Abelian 2=1+1
3 Abelian 3=1+1+1
4 Abelian 4=1+1+1+1
4 Abelian 4=1+1+1+1
5 Abelian 5=1+1+1+1+1
6 Abelian 6=1+1+1+1+1+1
6 Non Abelian 6=1+2+3
7 Abelian 7=1+1+1+1+1+1+1
8 Abelian 8=1+1+1+1+1+1+1+1
8 Abelian 8=1+1+1+1+1+1+1+1
8 Abelian 8=1+1+1+1+1+1+1+1
8 Non Abelian 8=1+1+2+2+2
8 Non Abelian 8=1+1+2+2+2
9 Abelian 9=1+1+1+1+1+1+1+1+1
9 Abelian 9=1+1+1+1+1+1+1+1+1
10 Abelian 10=1+1+1+1+1+1+1+1+1+1
10 Non abelian 10=1+2+2+5
11 Abelian 11=1+1+1+1+1+1+1+1+1+1+1
12 Non Abelian 12=1+1+3+3+2+2
24 Non Abelian 24=1+6+6+3+4
P a g e | 184

Partial Survey Table of Groups of Order upto 30

Order No. of Name of Abelian Groups No. of Name of Non- Total No.
of the Abelian Non- Abelian Groups of
Group Groups Abelian Groups
Groups

1 1 0 - 1
2 1 0 - 1
3 1 0 - 1
4 2 , 0 - 2
5 1 0 - 1
6 1 1 2
7 1 0 - 1
8 3 , , 2 , 5

9 2 , 0 - 2
10 1 1 2
11 1 0 - 1
12 2 , 3 , 5
13 1 0 - 1
14 1 1 2
15 1 0 - 1
16 5 , 9 , , 14
, , ,…

17 1 0 - 1
18 2 , 3 , ,… 5

19 1 0 - 1
20 2 3 ,… 5
P a g e | 185

Ring Theory

DEFINITION:
A non-empty set , together with two binary operations „‟ ‟‟ and „‟ ‟‟ is said to form a ring if the
following axioms are satisfied:

1. is an abelian group under „‟ ‟‟ following properties hold.


a) is closed under „‟ ‟‟
b)
c) there is an element such that known as identity of
d) For each there is an element such that
e)
2. is a semi-group under „‟ ‟‟ following properties hold.
a) is closed under „‟ ‟‟
b)
3. left and right
distributive properties hold.
NOTE:
In mostly books, binary operations of rings are taken as addition and multiplication
but they may be different from both of these. Later we will discuss some examples.

Examples:
The following are some famous examples of ring.

1) all are rings.


2) is a ring under addition and multiplication modulo
3) here is a non-empty set and for are defined as:
is a ring.
4) For a set of integers [] } under usual addition and multiplication
of complex numbers is a ring known as ring of Gaussian integers.
5) The set of all continuous real-valued functions
is a ring.
6) The set of all matrices under usual addition and multiplication is
a ring where is a ring.
7) The set of all polynomials [ ] with integer coefficients under usual addition and
multiplication is a ring.
P a g e | 186

8) The set ⋀ } is a ring under


component wise addition and multiplication.
9) The set of all matrices is a ring under matrix addition and multiplication
over ring of integers modulo
10) is ring with following binary operations;
11) For a ring of integers is also a ring.

COMMUTATIVE RING:
A ring is said to be commutative if

BOOLEAN RING:
A ring is said to be Boolean ring if

} is a Boolean ring under addition and multiplication modulo

Examples:
1) are commutative rings.
2) is a commutative ring under addition and multiplication modulo
3) Ring of Gaussian integers [ ] is a commutative ring
4) Ring of polynomials [ ] is a commutative ring.
5) The power set of a non-empty set is a commutative ring under following binary
operations:

Non-Examples:
1) Set of all matrices over is a non-commutative ring because for
* + * + * +* + * + * +* + * +
2) Ring of Quaternions } is a not
commutative ring because

RING WITH UNITY:


A ring is said to be a ring with unity if multiplicative identity exists in ring there exists an
element

Generally, unity of ring is denoted by and sometimes known as unit element or multiplicative
identity which is obviously unique.
P a g e | 187

Examples:
1) are all rings with unity
2) Set of all matrices over is a ring with unity * +
3) [ ] [ ] are rings with unity

Non-Examples:
1) Ring of even integers is a ring without unity.
2) Set of all matrices of the form * + over integers under matrix addition and
multiplication is a ring without unity.

RESULT:
In a ring for following results hold:





Moreover if is a ring with unity then following results are valid:


ZERO-DIVISOR:
In a ring a non-zero element is said to be zero divisor if there exists a non-zero element
such that or

Examples:
1) is not prime has zero divisors. } has zero divisor. is
divisor as
2) Set of all matrices over integers has zero divisors. for
* + * + * +* + * + * + is zero divisor of

the matrix * +
3) Set of all matrices over set of integers modulo where is not prime
has zero divisors. * + * + * +* + * + * +

is zero divisor of * +
P a g e | 188

Non-Examples:
1) has no zero divisors.
2) [] [ ] has no zero divisors.

INTEGRAL DOMAIN:
A commutative ring which has no zero divisors is called integral domain.

A commutative ring is said to be an integral domain if for

Examples:
1) are all integral domains.
2) is an integral domain under addition and multiplication modulo
3) [ ] [ ] are integral domains.

Non-Examples:
1) Set of all matrices over integers is not an integral domain.
2) is not an integral domain.
3) under addition and multiplication modulo is not an integral domain when is not
prime.

UNIT/MULTIPLICATIVE INVERSE:
An element is said to be unit or multiplicative inverse if there exists
an element such that

REMEMBER:
Unity/unit element and unit are different terms. A unity is unit but converse does not hold.

DIVISION RING:
A ring is called a division ring or skew-field if } is a group under multiplication.

Examples:
1) are all division rings.
2) , where is prime under modulo is a division ring.
3) Ring of Quaternion is a division ring.
P a g e | 189

Non-Examples:
1) is not division ring as multiplicative inverses of all elements of } does not
exist.
2) [ ] is not an integral domain if is not integral domain.

FIELD:
A commutative division ring is called a field.

Or, A ring is said to be field if the elements of } form a commutative group under
multiplication.

Examples:
1) all are field.
2) is a field under addition and multiplication modulo
3) [] } is a field.

Non-Examples:
1) is not field as non-zero elements of does not form group.
2) Ring of Quaternion is not a field.

CENTER OF RING:
The center of a ring is defined as:

IDEMPOTENT ELEMENT:
An element is called idempotent in a ring if

NILPOTENT ELEMENT:
An element is said to be nilpotent in a ring if for some positive integer

RESULTS:
 Every field is an integral domain.
 Every non-zero finite integral domain is a field.
 is a unit if and only if
 Set of all units in a commutative ring with unity form an abelian group.
 Unit of a ring divides every element of the ring.
P a g e | 190

 In an integral domain cancellation law holds.


 Every non-zero element is a unit or zero-divisor.
 A field has no zero-divisor.
 For two commutative rings an elements is zero-divisor of if and only if
one of or is zero-divisor or exactly one of them or is zero.
 Order of any finite field is is a prime.
 Number of units in
 Number of zero divisors of
 If is a field then [ ] is an integral domain.
 If is a unit in a ring for then is also unit.
 A finite commutative ring without zero divisors has unity.
 For a commutative ring with unity
a) is unit in if and only if is unit in a ring.
b) are units if and only if is unit in ring.
 For a commutative ring with unity if for is unit and then is a unit
of ring
 A ring is commutative if and only if .
 is a commutative if and only if
 In an integral domain if for some integer then
 In an integral domain with unity only idempotent elements are zero and unity.
 A non-zero idempotent element cannot be nilpotent.
 In a commutative ring with unity
a) If is a unit then is not nilpotent.
b) If is nilpotent then is unit.
c) The sum of a unit and nilpotent element is unit.
 In a ring without unity, every idempotent element is a zero-divisor but not nilpotent.
 For a prime every zero-divisor of is also nilpotent element.
 Every non-zero nilpotent element is zero divisor but converse is not true as ̅ is a zero
divisor in but it is not nilpotent.
 Any ring of prime order is commutative.
 A ring of order is prime may not be commutative.
 Smallest non-commutative ring is of order
 A ring with unity of order is prime must be commutative.
 If is a commutative ring then center of ring coincides with ring
 Center of a division ring is a field.
 The only Boolean ring that is integral domain is the set of integers under addition and
multiplication modulo }
 Number of nilpotent elements of a ring
 For two rings an element is nilpotent element of if and only if both
and are nilpotent elements of corresponding rings.
 Number of idempotent elements of is prime divisor of
P a g e | 191

 If is ring with unity. If denotes the set of all zero-divisors and denotes the
set of all units then
 In a ring then is commutative ring.
 In a field with more than two elements sum of all elements is zero.
P a g e | 192

Q:1 Which of the following statement is most accurate?

(a) [ ] is a commutative ring.


(b) A field must have zero divisors.
(c) is not a field under addition and multiplication modulo
(d) has no units.

Explanation:
We know that “ring of polynomials [ ] is commutative ring if is a commutative
ring‟‟.
Since set of complex numbers is a commutative ring. So does [ ]
A field does not have zero divisors because every field is an integral domain and an
integral domain is a commutative ring that does not have zero divisors.
is a field under addition and multiplication modulo because it is an abelian group
under addition and } } is a trivial group under multiplication.
Since has only two zero divisors and units So number of units in are
Option is correct.

Q:2 Point out the set of zero divisors for

(a) } (b) }

(c) } (d) }

Explanation:
We know that “a non-zero element of a ring has zero-divisor if there exists a non-zero
element such that ‟‟.
Since }

}
Option is correct.

Q:3 Number of zero divisors of are

(a) (b) (c) (d)


P a g e | 193

Explanation:
“Number of zero divisors of under addition and multiplication modulo are
‟‟.
By using above formula, we have

Option is correct.

Q:4 The unity of ring } under addition and multiplication modulo is?

(a) (b) (c) (d)

Explanation:
The unity of the ring is the multiplicative identity of the non-zero elements of ring.
Since,

is unity of the given ring.


Option is correct.

Q:5 Which of the following ring has neither unit nor zero-divisor?

(a) (b) (c) (d)

Explanation:
Since are field. So all non-zero elements have multiplicative inverse.
It means have units. As are integral domains.
So they have no zero-divisors.
is an infinite commutative ring having no zero-divisor.
Moreover, multiplicative inverses of non-zero elements in does not exist.
So, has neither units nor zero-divisor.
Option is correct.

Q:6 Which of the following ring has finite number of units?

(a) (b) [ ] (c) (d) All of these


P a g e | 194

Explanation:
Since and are infinite field. They have infinite number of units.
Ring of Gaussian integers has finite number of units.
Because an element of [ ] is unit if there exists an element of [ ] if

Either
Case:
Case:
Similarly, for
Only units of [ ] are
Option is correct.

Q:7 Which of the following statement is not true?

(a) A ring of order must be commutative.


(b) A ring with unity need not to be commutative.
(c) A division ring may be finite or infinite.
(d) Ring of polynomials need not to be field for some ring.

Explanation:
A ring of order may be finite or infinite. is a commutative ring under addition
and multiplication modulo But there also exists a ring of matrices over set of
integers modulo ,* + * + * + * +- is a non-commutative ring of
order
A ring with unity need not to be commutative. As the set of all matrices over set of
integers is non-commutative ring with unity * +
A division ring may be finite or infinite. .
Ring of polynomials [ ] is not field if is not field. [ ]
Option is correct.

Q:8 Which of the following is/are idempotent elements of ?

(a) (b) (c) (d)

Explanation:
“We know that an element of ring is said to be idempotent element if ”.
}, Since
So the only idempotent elements are
Option is correct.
P a g e | 195

Q:9 The center of which of the following ring must be a commutative division ring?

(a) (b) (c) (d)


Explanation:
We know that “center of a division ring is field”.
Moreover, “center of a commutative ring is the ring itself‟‟.
Since all above rings are commutative.
So their center coincides with the rings.
Since, are not division rings. Only division ring is
So its center is a field.
Option is correct.

Q:10 Number of idempotent elements of are __________

(a) (b) (c) (d)

Explanation:
We know that “number of idempotent elements of the ring where is the
number of prime divisors of ”.
Prime divisors of are So, number of idempotent elements are .
are idempotent elements.
Option is correct.

Q:11 Number of nilpotent elements of a ring are _______

(a) (b) (c) (d)

Explanation:
We know that „‟number of nilpotent elements of a ring of the form ‟‟.
Number of nilpotent elements of
Option is correct.

Q:12 For a ring of integers, which of the following statement is accurate?

(a) (b)
(c) (d) }
P a g e | 196

Explanation:
We know that “For a ring with unity , intersection of set of zero divisors of and set of
units of is empty‟‟.
Moreover, we know that an integral domain has no zero-divisors and only unit is
Option is correct.

Q:13 Which of the following statement is absolutely right for a non-empty set ?

(a) is a ring without unity.


(b) is non-commutative ring.
(c) is not a ring.
(d) is a ring with unity.

Explanation:
Power set of a non-empty set under binary operations of symmetric difference of sets
and intersection of sets is a commutative ring with unity because for any
,
Option is absolutely correct.

Q:14 The unity of the ring is ______ where

(a) (b) (c) (d)

Explanation:
We know that “an element of ring is unity if ‟‟.
Clearly, .
So is unity of the ring of integers for given binary operations.
Option is correct.

Q:15 Which of the following is Boolean ring?

(a) (b) (c) (d) All


P a g e | 197

Explanation:
“A ring is Boolean ring if ‟‟.
} is a Boolean ring with two elements as
} is a Boolean ring with four elements as
[ ]
[ ]
[ ]
[ ]
is an infinite Boolean ring.
Option is correct.

Q:16 In which of the following ring every zero-divisor is nilpotent element?

(a) (b) (c) (d)

Explanation:
We know that a field has no zero-divisor.
So, and are trivially satisfied for this statement.
Now, we check the zero-divisor of the remaining rings.
} are its zero-divisors that are not nilpotent.
} are its zero-divisors as well as nilpotent as

Moreover, by using result “Every zero-divisor of a ring of the form is nilpotent


element‟‟.
We can direct choose correct option without finding elements.
Option is correct.

Q:17 Point out the ring without unity which is contained in a field?

(a) (b) (c) (d)

Explanation:
Clearly, are rings with unity.
is only ring without unity which is contained in fields
Option is correct.
P a g e | 198

Q:18 The number of units of and , respectively are

(a) (b) (c) (d)

Explanation:
The number of units in are , where denoted the Euler Phi function.
Here and while

Answers

Q1 A Q2 B Q3 C Q4 D Q5 C Q6 B
Q7 A Q8 B Q9 D Q10 A Q11 B Q12 C
Q13 D Q14 B Q15 D Q16 A Q17 D Q18 C
P a g e | 199

Subrings and Ring Homomorphism

SUBRINGS:
A subset of a ring is said to be subring if itself is a ring under the operation of

SUBRING CRITERIA:
A non-empty subset of a ring is a subring if is closed under subtraction and multiplication.

NOTE:
Just like subgroups in „‟Group Theory‟‟, subrings play similar role in „‟Ring Theory‟‟.

Examples:

1. } is the subring of any ring known as trivial subring.


2. Ring of real numbers is subring of ring of complex numbers.
3. Ring of rational numbers is subring of ring of real numbers.
4. Ring of integers is subring of ring of rational numbers.
5. Ring of integers is subring of ring of real numbers.
6. Ring of integers is subring of ring of complex numbers.
7. Ring of rational numbers is subring of ring of complex numbers.
8. For each integer is a subring of
9. Ring of Gaussian integers [ ] is a subring of ring of complex numbers.
10.The set of all diagonal matrices over set of integers ,* + - is a subring

of a set of all matrices over set of integers ,* + -

CHARACTERISTIC OF A RING:
The characteristic of a ring is the least positive integer if exists such that

where is additive identity.


P a g e | 200

NOTE:
If no such integer exists, then characteristic of ring is zero.

Examples:

1. The characteristic of is as
2. The characteristic of is zero.

RING HOMOMORPHISM:
Let and be two rings. A mapping is said to be ring homomorphism if

mapping is group-homomorphism from to and semi-group homomorphism

from to

RING MONOMORPHISM:
For two rings and a ring homomorphism is called ring
monomorphism if is injective or one-one.

RING EPIMORPHISM:
For two rings and a ring homomorphism is called ring epimorphism if
is surjective or onto.

RING ISOMORPHISM:
For two rings and a ring homomorphism is called an ring
isomorphism if is bijective. is monomorphism and epimorphism.

and

RING ENDOMORPHISM:
A ring homomorphism from a ring to the ring is called ring endomorphism.

RING AUTOMORPHISM:
A ring endomorphism is called ring automorphism if is bijective.
P a g e | 201

Examples:

1. For any two rings and the zero mapping defined by


is ring homomorphism known as zero-homomorphism because

2. For any two rings and the identity mapping defined by


is ring homomorphsim known as identity-homomorphism because

3. For a ring of complex numbers the mapping defined by ̅ is a


ring homomorphism because
For ̅̅̅̅̅̅̅̅̅ ̅ ̅
For ̅̅̅̅̅̅ ̅ ̅
4. For a ring of integers and ring of integers under addition and multiplication modulo the
mapping , is a ring
homomorphism.

KERNEL OF A RING HOMOMORPHISM:


Let be a ring homomorphism then kernel of is defined as

Examples:
1. } is a zero-homomorphism. is defined as

2. } is an identity-homomorphism. is defined
as
}
3. } is ring-homomorphism defined
as
}

RESULTS:
 Center of a ring is a subring of ring
 Sum of two subrings of a ring need not to be a subring of ring
 If is a subring of and is a subring of then is a subring of Transitive
property holds in subrings.
 Intersection of any collection of subrings of a ring is a subring of
P a g e | 202

 Union of two subrings of a ring need not to be subring of


are subrings of but their union } is
not a subring because
 The direct product of two rings is a ring.
 Nilpotent elements of a ring form subring of a ring
 For two subrings of their intersection [ ] is a subring
of
 The characteristic of an integral domain is either zero or a prime number.
 The characteristic of a field is either zero or a prime number.
 Let be a ring with unity. If order of unity is infinite under addition then characteristic
of is and if order of unity is finite order under addition then characteristic of is .
 A field of characteristic zero is infinite.
 Characteristic of Boolean ring is
 Characteristic of an integral domain of order , is a prime number.
 A field of order has characteristic
 If is a ring of characteristic then characteristic of a ring of matrices over is
 If is a ring with elements then characteristic of divides
 If then where is a subring of a ring
 For two rings and ring homomorphism

 For a ring homomorphism , } is a subring of


 Product of two ring homomorphism is again a ring homomorphism.
 Inverse of a ring homomorphism is again a ring homomorphism.
 For a ring homomorphism if is a subring of then is a subring of
 Let be a ring homomorphism then
i. is a subring of
ii. Image of is a subring of
iii. If and then
 A ring homomorphism is injective if and only if }
 For a ring homomorphism the homomorphic image of unity need not to be
unity. homomorphic image of unity of is which is not
unity of
( )
 Number of ring homomorphism from to , where is number
of prime divisors of
 If is a commutative ring of characteristic then is ring
homomorphism.
 If is a ring with unity then the mapping defined by
is a ring homomorphism.
 If is a ring with unity and then there exists a subring of isomorphic
to but if then there exists a subring of isomorphic to
 For any positive integer the mapping defined by is a
ring homomorphism, known as natural ring homomorphism.
P a g e | 203

 For two rings


 For a ring of positive integers and two distinct positive integers
 For two rings and
i. The mapping defined by ( ) is a ring homomorphism.
ii. The mapping defined by ( ) is one-to-one ring
homomorphism.
iii.
 For two commutative rings with unity if is ring homomorphism and
characteristic of is non-zero then divides
 If is a commutative ring then ( ).
 If is a commutative ring then [ ] has a subring isomorphic to
 For two rings if then [ ] [ ]
 The only ring automorphism of ring of real numbers is identity map.
 For two rings if divides and is an idempotent element of then mapping
defined by is ring homomorphism.
 A ring homomorphism carries an idempotent element to an idempotent element.
 For any ring the mapping [ ] defined by ( ) is a ring
homomorphism.
 The number of ring homomorphism from to zero map and identity map.
 The number of ring homomorphism from to
P a g e | 204

Q:1 Choose the correct statement.

(a) A ring with unity must have a subring with unity.


(b) A ring with unity can not have a subring with unity.
(c) A ring without unity can not have a subring with unity.
(d) A ring with unity may have a subring without unity.

Explanation:
A ring with unity may have a subring without unity. in
A ring with unity can have a subring with unity. in
A ring without unity can have a subring with unity. ring of matrices of the
form * + over set of integers is a ring without unity but ring of matrices of the

form * + over set of integers is its subring with unity * +


Option is correct.

Q:2 Which of the following statement is false?

(a) Unity of a subring of a ring may be different from unity of ring


(b) is a subring of
(c) is not subring of
(d) Every subring of field need not to be field.

Explanation:
Unity of a subring may be different from unity of ring.
} is a ring with unity
But } is its subring with unity as
} is a subring of }
Union of two rings and need not to be a ring but their union is ring if one of them is
contained in other. Since,
}
}
Their union is which is a subring of
Every subring of a field need not to be field.
is a field but its subring is not field.
Option is correct.
P a g e | 205

Q:3 Which of the following is a subring of ?

(a) ,* + - (b) ,* + -

(c) ,* + - (d) Both

Explanation:
,* + - is a subring of as:

For * + * + ,* + -

* +* + * +
Because .
Also * + * + * +
Because
,* + - is a subring of as:
For * + * + * +* + * +
Also * + * + * +
,* + - is not subring of as:

For * +

But the product * +* + * +


Option is correct.

Q:4 If 20 1 3 is a subring of then unity of is?

(a) [ ] (b)

[ ]

(c) [ ] (d) does not exist


P a g e | 206

Explanation:

Clearly, [ ] . So [ ] can never be unity of

Now, [ ] can never be unity of because for any [ ] ,

[ ][ ] [ ] [ ][ ]

Again we check is unity of as for any 0 1

[ ]

0 1 0 1 0 1

[ ] [ ]
Option is correct.

Q:5 Which of the following statement is true about characteristic of ring?

(a) Characteristic of an infinite ring may be non-zero.


(b) Characteristic of an infinite ring must be infinite.
(c) Characteristic of an infinite ring must be zero.
(d) Characteristic of a finite ring may be infinite.

Explanation:
By definition “characteristic of a ring is the least positive integer such that
‟‟.
Characteristic of a finite ring must be finite as if is a ring with finite elements say
then its characteristic must divide
So characteristic of a finite ring can never be infinite.
Characteristic of an infinite ring can never be infinite by definition.
Characteristic of an infinite ring may be zero or non-zero depending upon ring.
If is an infinite ring with unity such that order of unity under addition is infinite then

But we have an infinite ring that has non-zero characteristic.


We know that characteristic of a Boolean ring is because
Now the infinite Boolean ring has characteristic as

Option is correct.
P a g e | 207

̅
Q:6 For a ring homomorphism defined by 2 then
̅
?
(a) (b) (c) (d) }

Explanation:
By definition “kernel of a ring homomorphism is defined as:
}‟‟.
Since, according to given mapping
So, }
Option is correct.

Q:7 If } is a ring under addition and multiplication then


?

(a) (b) (c) (d)

Explanation:
We know that a finite ring has finite non-zero characteristic.
Moreover, number of elements of a finite ring must divide characteristic of ring.
Only possibility is
Option is correct.

Q:8 Which of the following is not a subring of ( where : is endomorphism?

(a) }
(b) }
(c) A subset of which is closed under usual addition and multiplication.
(d) (√ ) { √ } under usual addition and multiplication.

Explanation:
is actually the Kernel of ring homomorphism which is always subring.
(√ ) { √ } is a ring and a subset of real numbers so it is the subring
of ring of real numbers.
For let then and
Consider
Similarly so which shows that is subring.
is a subset of which is closed under addition and multiplication but not a subring.
P a g e | 208

Q:9 For ,* + - if defined by (* +) is ring


homomorphism then ?

(a) ,* + - (b) ,* + -

(c) ,* + - (d) ,* + -

Explanation:
By given mapping,
,* + (* +) -

,* + -

,* + -.
Option is correct.

Q:10 Number of ring homomorphism from to are ______

(a) (b) (c) (d)

Explanation:
( )
We know that “number of ring homomorphism from to , where
is number of prime divisors of ‟‟.
( )
Number of ring homomorphism from to .

Answers:

Q1 D Q2 C Q3 D Q4 B Q5 A
Q6 C Q7 B Q8 D Q9 D Q10 B
P a g e | 209

Ideals

IDEALS:
A subring of a ring is said to be an ideal or two-sided ideal of if for every and
both

An ideal of is proper ideal if is proper subset of

NOTE:
Like normal subgroup in „‟Group Theory‟‟, ideal plays the similar role in „‟Ring Theory‟‟.

Clearly, every ideal of ring is a normal subgroup of

IDEAL CRITERIA:
A non-empty subset of ring is ideal of if:

i.
ii.

Examples:

1. In any ring } is trivial ideal of


2. In any ring is an improper ideal of
3. Like in subrings, the ideals of are of the form
4. For the ring of polynomials with real coefficients [ ], the subset of all polynomials with
constant term zero is ideal of [ ].
[ ] }
5. For any ring of matrices over set of integers , ,* + - is a

right ideal of but not left as for * +

* +* + * +

6. For any ring of matrices over set of integers ,* + - is a

left ideal of but not right ideal as for * +


P a g e | 210

* +* + * +

REMEMBER:
Clearly, an ideal is always a subring but a subring need not to be an ideal.

For justification, we consider some examples.

Non-Examples:

1. is not an ideal of as for and √ , √


2. is not an ideal of as for and

SIMPLE RING:
A ring } which has no ideal except } and is known as simple ring.

Examples:

1. are simple rings as their only ideals are } and the ring itself.

Non-Examples:

1. are not simple rings as ideals of are of the form and ideals of are of
the form .

FACTOR RINGS/QUOTIENT RINGS:


Let be a ring and be an ideal of ring then set of all cosets of in is a ring under addition
and product of cosets in known as Factor ring or Quotient ring.

} is a ring under the binary operations as:

NATURAL HOMOMORPHISM:

For an ideal of a ring there exists a homomorphism known as natural or canonical


homomorphism.
P a g e | 211

Examples:

1. For any ring , }


and are always quotient rings.

2. Ideals of are of the form We can construct quotient rings of the form for
some positive integer
For }
} }
As and .
3. Ideals of are of the form We can construct quotient rings of the form for some
positive integer
For }
} }

4. For a ring ,* + - and ,* + - ,* + -


is a quotient ring.
5. For any rings is an ideal for So is always a quotient ring.
Non-Examples:

1. For ring of integers is not quotient ring but quotient group as is normal subgroup
of but not ideal of

PRINCIPAL IDEAL:
An ideal of a commutative ring with unity is said to be principal ideal if it is generated by a
single element in say It is denoted and defined as:

⟨ ⟩ }

each element of principal ideal is multiple of

Clearly, ⟨ ⟩ is the smallest ideal of containing

Examples:

1. For any ring } ⟨ ⟩ and ⟨ ⟩ are always principal ideals.


2. For each integer are principal ideals of ring of integers as for
} } ⟨ ⟩
⟨ ⟩ }
3. For ring of polynomials [ ] the set of all polynomials with constant term zero is the
principal ideal of [ ] ⟨ ⟩ is ideal of [ ]
P a g e | 212

Non- Example:

1. For a ring of polynomials [ ] ⟨ ⟩ is not a principal ideal.

PRINCIPAL IDEAL RING:


An integral domain with unity is said to be principal ideal ring if every ideal of is a
principal ideal.

Example:

1. Ring of integers is a principal ideal ring as its every ideal is principal ideal.
⟨ ⟩

PRIME IDEAL:
Let be a commutative ring. An ideal of a ring is called a prime ideal if for ,

Examples:

1. For a ring of integers are prime ideals if and only if is a prime number.
} is a prime ideal as for and
but
2. } as an ideal of ring of integers is a prime ideal as } or
} or }
Non-Example:

1. is not prime ideal as for but neither nor

MAXIMAL IDEAL:
Let be a commutative ring. An ideal is called maximal ideal of if whenever is ideal
of then either

Examples:

1. For any field that is clearly ring, } is only maximal ideal of


2. For ring of integers are maximal ideals of if and only if is prime number
are prime ideals of for a prime
P a g e | 213

Non-Example:

1. is not maximal ideal of as

RESULTS:
 Number of ideals of ( ) .
 For is a ring } is right ideal of
 For is a ring } is left ideal of
 If every subring of a ring is ideal and in then is a
commutative ring. Converse need not to be true. For example is a commutative ring
but its subring is not ideal of
 For any two ideals and of a ring is an ideal of containing both and
Sum of ideals is again an ideal.
 For any two ideals of a ring
 For two ideals of a commutative ring with unity if then
 For a commutative ring, } is an ideal of
 For a commutative ring with unity if then ⟨ ⟩ }
 Sum of a left and right ideal need not to be an ideal. ,* + - is right

ideal of ring of all matrices over and ,* + - is left ideal but the

sum is not ideal as * +* + * +


 Intersection of a left and right ideal of a ring need not to be an ideal.
,* + -, ,* + -. But is not ideal of

as * +* + * +
 For any two ideals of a ring is principal ideal of generated by
⟨ ⟩

 For a commutative ring and } is ideal of


 If is an ideal with the same unity as the ring with unity then

 Property of being ideal need not to be transitive. {[ ] } is an ideal of

{[ ] } which is ideal of {[ ] } but

is not ideal of as [ ][ ] [ ]
P a g e | 214

REMEMBER:
If is an ideal of and is an ideal of then is an ideal of if is a ring with unity that may
be different from unity of

 For any two ideals of is an ideal of product of two ideals is again an


ideal.
 If is left ideal of and is right ideal of then is an ideal of but need not to
be an ideal. ,* + - is left ideal and ,* + - is right

ideal of ring of all matrices over integers but ,* + - is not ideal of

as * +* + * +
 For two ideals ⟨ ⟩ ⟨ ⟩ of a ring ⟨ ⟩ ⟨ ⟩ ⟨ ⟩, ⟨ ⟩ ⟨ ⟩ ⟨ ⟩
and ⟨ ⟩ ⟨ ⟩ ⟨ ⟩
 A division ring is a simple ring.
 If is a ring with unity and only right ideals of are } then is a division ring.
 Intersection of two ideals is again ideal
 Union of two ideals need not to be an ideal. ideals of but their union is not
ideal as }
 Union of two ideals of a ring is ideal if
 A ring can not be expressed as union of two proper ideals but it can be expressed as a
union of three proper ideals.
 The only ideals of a field are } and
 For any ideal of [ ] } is an ideal of containing
 For a ring homomorphism is an ideal of
 For any ideal of a ring } is a subring of
 For a non-commutative ring with unity, center of ring is a subring of but not an ideal
of
 Intersection of finite number of ideals of a ring is ideal of
 Intersection of finite number of left ideals of a ring is left ideal and intersection of
finite number of right ideals is right ideal of
 For a ring homomorphism if is ideal of then is ideal of
 If is right ideal and is left ideal of such that } then where

 For any ideal of a ring is a ring and homomorphic image of


 For an ideal of a ring there exists an epimorphism
 For any two ideals of if then is an ideal of
 For an epimorphism
P a g e | 215

i. is an ideal of
ii.
REMEMBER:
The above statement is known as “Fundamental theorem of homomorphism of rings.

 For an epimorphiasm ,
i. is an epimorphism.
ii. is an isomorphism.
iii. .
( )
 For two ideals of a ring
( )

REMEMBER:
The above statement is known as “First theorem of Isomorphism for rings‟‟.

 For an ideal of a ring and a subring of a ring


i. is a subring of
ii. .
iii.
REMEMBER :
The above statement is known as “Second Isomorphism theorem of rings‟‟.

 For an epimorphism where is a ring with then is of


 For ring of integers, ⟨ ⟩
 has no non-zero nilpotent element if and only if is square free positive integer ( is
not divisible by square of any prime).
 Relation of Isomorphism in rings is an equivalence relation.
 Any homomorphism of a field is either a monomorphism or takes each element to zero.
 For an ideal of a ring
i. If is commutative then is also commutative.
ii. If has unity then is unity of
 Set of all nilpotent elements of a commutative ring forms an ideal of and has no
non-zero nilpotent elements.
 If is a proper ideal of a commutative ring with unity then is a commutative ring
with unity.
P a g e | 216

 Let be an integral domain with non-zero characteristic. If is proper ideal of then


( )
 For ⟨ ⟩ ⟨ ⟩

 Let be a commutative ring with unity an ideal of is maximal ideal if and only if
is a field.
 Let be a commutative ring. An ideal of is prime ideal if and only if is an integral
domain.
 Let be a commutative ring with unity. If every ideal of is prime ideal then is a
field.
 Let be a commutative ring with unity then every maximal ideal is prime ideal.
 In a Boolean ring , every prime ideal is maximal ideal.
 For a commutative ring if is an ideal of and is prime ideal of then is an ideal
of
 A commutative ring is an integral domain if and only if } is prime ideal of
 In a finite commutative ring with unity, every prime ideal of is maximal ideal.

REMEMBER:
For application of above result, ring must be commutative with unity.

 An ideal of a ring is prime ideal if and only if is closed under


multiplication.
 Let be a ring and be a fixed prime then } is an ideal of
Here, denotes additive order of
 Number of prime/maximal ideals of
 An ideal ⟨ ⟩ of [ ] is maximal ideal if and only if | | is a prime number.
P a g e | 217

Q:1 Number of ideals of are _____

(a) (b) (c) (d)

Explanation:
We know that number of ideals of a ring of integers under addition and multiplication
modulo are where is Totient function defined as:
( )
Here, .

Option is correct.

Q:2 Which of the following is maximal ideal of ring of Gaussian integers [ ]?

(a) ⟨ ⟩ (b) ⟨ ⟩ (c) ⟨ ⟩ (d) ⟨ ⟩

Explanation:
We know that “An ideal ⟨ ⟩ of [ ] is maximal ideal if and only if | | is a
prime number‟‟.
Contrapositive statement of above result is “An ideal ⟨ ⟩ of [ ] is not maximal if
and only if | | is not a prime number‟‟.
Clearly, ⟨ ⟩ ⟨ ⟩ and ⟨ ⟩ are not maximal ideals as
| | | | and | | are not prime numbers.
But ⟨ ⟩ is maximal ideal as | | is a prime number.
Option is correct.

Q:3 Number of prime ideals of are ______

(a) (b) (c) (d)

Explanation:
We know that number of prime or maximal ideals of where denotes
number of prime divisors of
Since, only prime divisors of are .
So,
Option is correct.
P a g e | 218

Q:4 Which of the following is prime ideal as well maximal ideal of ring of integers?

(a) ⟨ ⟩ (b) ⟨ ⟩ (c) ⟨ ⟩ (d) ⟨ ⟩

Explanation:
We know that an ideal ⟨ ⟩ of ring of integers is prime and maximal ideal if and only is a
prime number.
are not prime numbers.
So their corresponding ideals are neither prime nor maximal.
Only maximal ideal is ⟨ ⟩ as is a prime number by primality test.
Prime numbers √ and none of these divide
Option is correct.

Q:5 Which of the following statement is correct?

(a) Every prime ideal of a ring may not be maximal ideal of


(b) A prime ideal of a ring is always maximal ideal of
(c) A prime ideal of a ring can never be maximal ideal of
(d) Every maximal ideal of a ring is always prime ideal of

Explanation:
A maximal ideal of a ring need not to be prime ideal.
} is a ring under addition and multiplication modulo
The ideal ⟨ ⟩ } of is maximal as their does not exist any ideal that
contains ⟨ ⟩
But ⟨ ⟩ is not prime as ⟨ ⟩ but ⟨ ⟩
A prime ideal of a ring may be maximal ideal of
For a prime number ⟨ ⟩ of is prime as well maximal ideal.
A prime ideal of a ring need not to be maximal ideal.
⟨ ⟩ is prime ideal of but not maximal as ⟨ ⟩ ⟨ ⟩ }.
Option is correct.
P a g e | 219

Q:6 Under what condition on a ring every prime ideal is maximal ideal?

(a) must be a commutative ring.


(b) must be a ring without unity.
(c) must be a commutative ring with unity.
(d) must be a finite commutative ring with unity.

Explanation:
There exists a commutative ring in which every prime ideal is not maximal ideal.
is a commutative ring with prime ideal ⟨ ⟩ that is not maximal as ⟨ ⟩ ⟨ ⟩
There exists a commutative ring with unity in which every prime ideal is not maximal
ideal.
There exists a ring of even integers with prime ideal ⟨ ⟩ that is not maximal ⟨ ⟩ ⟨ ⟩
We have a result “Every prime ideal of a finite commutative ring is maximal ideal”.
Option is correct.

Q:7 Which of the following statement is not correct?

(a) Intersection of two prime ideals must be a prime ideal.


(b) Intersection of two maximal ideals must be a maximal ideal.
(c) Intersection of a maximal and prime ideal must be maximal ideal.
(d) All of these.

Explanation:
Intersection of two prime ideals need not to be prime ideal.
Since, For a prime number ⟨ ⟩ is prime ideal of ring of integers.
⟨ ⟩ ⟨ ⟩ } } } ⟨ ⟩
Which is not prime ideal as ⟨ ⟩ but ⟨ ⟩
Intersection of two maximal ideals need not to be a maximal ideal.
⟨ ⟩ ⟨ ⟩ } ⟨ ⟩ ⟨ ⟩
Intersection of a maximal and prime ideal of a ring need not to be maximal ideal.
⟨ ⟩ is prime ideal of and ⟨ ⟩ is maximal ideal of but ⟨ ⟩ ⟨ ⟩ ⟨ ⟩ is not
maximal ideal.
Option is correct.
P a g e | 220

Q:8 Under what condition on a ring every maximal ideal of is prime ideal?

(a) must be a commutative ring.


(b) must be a finite ring.
(c) must be commutative ring with unity.
(d) must be a ring without unity.

Explanation:
There exists a commutative ring without unity in which not every maximal ideal of
is prime ideal.
} is a commutative that is also finite but it has a maximal ideal
} that is not prime ideal as ⟨ ⟩ but ⟨ ⟩
We have a result “In a commutative ring with unity, every maximal ideal is prime ideal‟‟.
In ring of integers every maximal ideal is prime ideal as it satisfies above criteria.
Option is correct.

Q:9 Which of the following statement is not correct?

(a) Quotient ring of an integral domain need not to be an integral domain.


(b) Quotient ring of an integral domain must be an integral domain.
(c) Quotient ring of a commutative ring must be a commutative ring.
(d) Homomorphic image of a commutative ring must be commutative.

Explanation:
Quotient ring of a commutative ring must be commutative.
Homomorphic image of a commutative ring must be commutative.
Quotient ring of an integral domain need not to be an integral domain.
is an integral domain but its quotient ring is not quotient ring as it has
zero divisors.
Option is correct.

Q:10 Which of the following quotient ring has no non-zero nilpotent element?

(a) ⟨ ⟩
(b) ⟨ ⟩
(c) ⟨ ⟩
(d) ⟨ ⟩
P a g e | 221

Explanation:
We have a result “A quotient ring ⟨ ⟩
has no non-zero nilpotent if and only if is square
free element. is not square of some integer.
By using above result, ⟨ ⟩ ⟨ ⟩ ⟨ ⟩ have non-zero nilpotent elements.

⟨ ⟩
has no non-zero nilpotent elements.

⟨ ⟩
} has non-zero nilpotent elements. are nilpotent elements of

⟨ ⟩
} has non-zero nilpotent elements. are nilpotent
elements of
⟨ ⟩
} has non-zero nilpotent elements. are
nilpotent elements of
Option is correct.

Q:11 In which of the following statement, converse does not hold?

(a) For an ideal of a ring if is commutative ring then so is


(b) For an ideal of a ring if has unity then unity of is
(c) Homomorphic image of a commutative ring is commutative ring.
(d) All of these

Explanation:
Take a ring ,* + - and for its ideal ,* + - is

commutative as ,* +- which is commutative but is non-commutative.

Take a ring homomorphism defined by (* +)

where, ,* + -.
Since, is an epimorporphism so by fundamental theorem for ring homomorphism
which is commutative but itself is not commutative.
Take two rings and define a mapping
defined by
Clearly, is an epimorphsim.
By fundamental theorem of homomorphism .
It is obvious that quotient ring is a ring with unity but the original ring is
without unity.
Option is correct.
P a g e | 222

Q:12 Which of the following ring has exactly two maximal ideals?

(a) (b) (c) (d)

Explanation:
A ring for two distinct primes has exactly two maximal ideals ⟨ ⟩ ⟨ ⟩
So, has exactly two maximal ideals. ⟨ ⟩⟨ ⟩
A field has only one maximal ideals.
Since, are fields they have only one maximal ideal.
Only maximal ideal of is ⟨ ⟩
Option is correct.

Q:13 Which of the following quotient ring is an integral domain?

[] []
(a) (b) (c) (d) All of these
[] []

Explanation:
is not an integral domain as it has zero-divisors.
[] []
is an integral domain but is not an integral domain by using the following
[] []
result.
[]
“ [ ] is an integral domain if and only is a prime number of the form ‟‟.
Option is correct.

Q:14 Number of ideals of are _____

(a) (b) (c) (d)

Explanation:
} } } } .
So, ideals of are } } } }
Option is correct.
P a g e | 223

Q:15 In which of the following ring, every prime ideal is maximal ideal?

(a) Commutative ring (b) Unity ring


(c) Integral domain (d) Boolean ring

Explanation:
In a commutative ring, every prime ideal need not to be maximal ideal.
For example in the ring of integers, } is a prime ideal as for any integers

} } but } is not maximal as } ⟨ ⟩


In an integral domain and unity ring/ring with unity, every prime ideal is not maximal.
For example ring of integers which is an integral domain.
We have a result “In a Boolean ring, every prime ideal is maximal”.
Option is correct.

Q:16 For a ring of integers, ⟨ ⟩ ⟨ ⟩

(a) } (b) }
(c) Both (d) }

Explanation:
We have a result, “For a ring of integers ⟨ ⟩ ⟨ ⟩
.
One more result, we will use here
“For two rings ‟‟
So, ⟨ ⟩ ⟨ ⟩ } }
Clearly, } is not a ring as it is not closed under addition.
Option is correct.

Q:17 Which of the following statement is appropriate?

(a) Sum of a subring and an ideal of a ring must be an ideal.


(b) Sum of an ideal and a subring of a ring must be a subring.
(c) Sum of an ideal and a subring of a ring may not be a subring.
(d) Only
P a g e | 224

Explanation:
Sum of an ideal and a subring of a ring need not to be an ideal.

{[ ] } {[ ] } is an ideal

of and {[ ] } is a subring of

But, {[ ] } is not an ideal of ,

[ ][ ] [ ]

as for some and


According to second Isomorphism theorem “For an ideal and a subring of a ring
must be a subring of ”.
Option is correct.

Q:18 In the ring of integers, possible value of for ⟨ ⟩ ⟨ ⟩ ⟨ ⟩⟨ ⟩ ⟨ ⟩ ⟨ ⟩?

(a) (b) (c) (d)

Explanation:
We have a result
“In the ring of integers, for ⟨ ⟩⟨ ⟩ ⟨ ⟩⟨ ⟩ ⟨ ⟩ ⟨ ⟩”
⟨ ⟩⟨ ⟩ ⟨ ⟩ ⟨ ⟩ ⟨ ⟩ ⟨ ⟩
Option is correct.

Q:19 Which of the following must be an ideal of a commutative ring ?

(a) (b) (c) (d)

NOTE:
Here, denote center, set of nilpotent elements and set of idempotent elements of
ring respectively.
P a g e | 225

Explanation:
“For a commutative ring which is trivially ideal of ‟‟.
“For a commutative ring is an ideal of ‟‟.
For a commutative ring need not to be an ideal of
For example } but } is not an ideal of as
for
Option is correct

Q:20 Which of the following two rings have same characteristic?

(a) (b) (c) (d) None

Explanation:
We know that “Characteristic of is ‟‟.
Since, , ( ) .
( ) .
Since, ( )
Option is correct.

Answers:

Q1 D Q2 D Q3 C Q4 B Q5 A
Q6 D Q7 D Q8 C Q9 B Q10 D
Q11 D Q12 C Q13 A Q14 A Q15 D
Q16 C Q17 B Q18 A Q19 D Q20 C
P a g e | 226

Vector Space
Let be any field and be any non-empty set then is a vector space over denoted by ( )
if the following axioms hold:
is an abelian group.

( i.e. There exist a map “.” Where . : Called scalar multiplication)


the following conditions satisfied.

where is the multiplicative identity of


Remember that elements of vector space are called vectors and that of field are scalars.

Examples:
Let be any field then is a vector space.
√ √ , are vector spaces.
( is a vector space called Euclidean space.
is a vector space where is set of all matrices over
Let [ ] denotes set of all polynomials with entries from is a vector space over
Let [ ] denotes set of all polynomials of degree less than or equal to with entries from is
a vector space over
Let and consider } then is a vector space over under the addition
and scalar multiplication defined by

Non-Examples:

is not a vector space under the scalar multiplication defined by


where .
is not a vector space as
Let be the first quadrant in the that is, set
,* + - then is not a vector space over as for

* +
P a g e | 227

Let be the union of first and third quadrant in the that is , set
,* + - then is not a vector space over as for

* + * + * +

Theorem:
If is vector space over then

Subspace:
Let be a vector space and then we say is a subspace of if is a
vector space.

Trivial Subspaces:
Zero subspace } is called Trivial subspaces of any vector space

Non-Trivial Subspaces:
All other subspaces of are called non-trivial subspaces of

Examples:
is a non-trivial subspace of .
is a non-trivial subspace of (√ ) .
√ is subspace of .
[ ] is a subspace of [ ].

Subspace Criteria:
Let be a vector space and then we say is a subspace of iff
.
Remember that first of all you will check whether additive identity of is in If it is not in
then don‟t need to apply subspace criteria.

Example:1

Consider 20 1 | 3 then is clearly a subspace of


P a g e | 228

Example:2
A plane in not through the origin is not a subspace of similarly a line in not through
the origin is not a subspace of as both did not contain additive identity.

Example:3
Let } then is not a subspace of as .

Example:4
Let } then is a subspace of as
.

Example:5
Let be the set of points inside and on a unit circle in the that is the set
,* + } then is not a subspace of as for

Example:6
The only subspaces of are } lines through origin and .

Example:7
The only subspaces of are } lines through origin, planes through origin and .

Example:8
Let be vector space of all real matrices then set of all diagonal , upper triangular,
lower triangular, symmetric, skew symmetric matrices are subspaces of

Union of Two subspaces:


Let be subspaces of a vector space then need not to be subspace of
Let and consider } and } then is not a
subspace of as then must contain and is not
closed under addition as
is subspace of iff either or .

Intersection of subspaces:
Intersection of two subspaces is a subspace of In fact, intersection of any collection of
subspaces is again a subspace.
P a g e | 229

Sum of two subspaces:


Let be subspaces of a vector space then
} is also a subspace of .
and both are subspaces of

Result:
If are three subspaces of a vector space Such that
then

Example:
Above result is not true if . For this consider ,
| } | } | }
Then } and but

Direct Sum of subspaces:


A vector space is the direct sum of its subspaces and denoted by if

}
Example:
For any field consider the vector space then is the direct sum of its two
subspaces and where
| } and | }

Example:
Let be vector space of all real valued functions then is the direct sum of
| } and | }

Example:
Let be vector space of all real matrices. Let be set of all symmetric matrices in and
be set of all skew symmetric matrices in then
P a g e | 230

Q:1 Which of the following is not a vector space?

(a) ( ) (b) ( ) (c) ( ) (d) ( )

Explanation:
( ) , ( ) , ( ) are vector spaces while is not a vector space under the scalar
multiplication defined by
where .

Q:2 Which is following is not subspace of

(a) } (b) }
(c) } (d) }

Explanation:
Clearly in option (c) does not belong to the given set don‟t need to apply subspace
criteria and } is not a subspace of
For the sake of our convenience we check option (d)
Let }

Consider

if

and then
Using
we can see that
Hence is a subspace of
In a similar fashion we can check
} and } are subspaces of

Q: Q:3 Let then which is a subspace of

(a) | | } (b) }
(c) for some } (d) All of these
P a g e | 231

Explanation:
Since sum of two singular matrices need not to be singular as for
* + and * +

But * + which is not in so is not closed under addition.


For two idempotent matrices
* + and * + * + is not idempotent.
For option (c) we may proceed as take then and
Consider
so is a
subspace.

Q:4 Which of the following is false?

(a) is a vector space. (b) A sub space is also a vector space


(c) for some elements of (d) A vector is an element in vector space

Explanation:
Option (a) , (b) , (d) are correct statements while statement holds good for all
elements of a vector space

Q:5 Let and be the subspaces of than which is not a subspace of

(a) (b) (c) (d) None of These.

Explanation:
As sum and Intersection of two subspaces is a subspace and union of two subspaces need
not to be subspace of vector space (Example already discussed).

Q:6 Let then which is a subspace of ?

(a) (b) (c) (d)


P a g e | 232

Explanation:
and are not vector spaces due to failure of scalar multiplication axiom while
is not a subset of so cannot be a subspace of
is trivial subspace of .

Q:7 Let } } then is


(a) plane (b) (c) (d) plane

Explanation:
Since then and
then elements of are of the form ( ) and hence sum has elements of the form
( ).

Rewriting in matrix form [ ]

Now applying row operations we get 0 1 so

Q:8 Let } } be subspaces of then

(a) (b) (c) (d) None

Explanation:
We can observe that } and we can show that .
Thus the space is direct sum of its subspaces and

Q:9 Let be vector space of matrices over field of real numbers and and be
subspaces of symmetric and skew-symmetric matrices respectively then

(a) (b) } (c) (d)


P a g e | 233

Explanation:
Since, every real matrix can be written as
where and and Null matrix is the only matrix
which is symmetric and Skew-symmetric.
Hence is the direct sum of and .

Q:10 Consider the following subsets of


} }
} }
Which of the following pair have the property of being a subspace of

(a) (b) (c) (d)

Explanation:
For we proceed as and then
. For consider and but .
and are subspaces of (Using Subspace criteria).

Q:11 Consider the following subsets of


} }
} }
Which of the following pair is subspace of

(a) (b) (c) (d)

Explanation:
Since and are in but
Also and are in but
and are subspaces of
Verify, using subspace criteria.
P a g e | 234

Q:12 Let be set of all complex Hermition matrices then is vector space over

(a) (b) but not (c) Both and (d) but not

Explanation:
is vector space over but not over as for ,* + but

* + * +

Q:13 Which of the following is not a vector space over the field of real numbers?

(a) }
(b) , | [ ] ( ) -
(c) | [ ] }
(d) , | [ ] ( ) -

Explanation:
is a vector space over as .
For we will check that is subspace of all real valued functions by using subspace
criteria.
Let and be in then ( ) ( )
Consider ( ) ( ) ( )
Now for any ( ) ( )
So is subspace of vector space of all real valued functions.
In a similar fashion you can check .
For take and consider ( ) ( ) ( ) .
Hence is not closed under addition.

Q:14 The set } is not a vector subspace of because

(a) zero element does not exist. (b) is not closed under scalar multiplication.
(c) is not closed under vector addition (d) multiplicative inverses does not exist.
P a g e | 235

Explanation:
For given set zero element exist.
is closed under scalar multiplication as if we take
(Here, ) then
Inverse of each element exist.
But is not closed under vector addition as if we take
then

Q:15 Let be a vector space of all real-valued functions then pick up the set which is not a
subspace of ?

(a) } (b) }
(c) } (d) }

Explanation:
For option (a),
Take
Then ,

Also for
Option (a) is a subspace of
For option (b),
Take then
But vector
addition does not hold.
For option (c), take and
Then must exist as both of
these limits exist separately.
Similarly, for option (d).

Answers:
Q:1 d Q:2 c Q:3 c Q:4 c Q:5 c

Q:6 d Q:7 c Q:8 c Q:9 c Q:10 d

Q:11 d Q:12 b Q:13 d Q:14 c Q:15 b


P a g e | 236

Linear Combination:
Let ( ) be a vector space and and ,
where
Then is the linear combination of

Examples:
Let take any then can be written as linear combination of three
vectors , then
.We say is a linear combination of
, and .

Spanning Set:
Let then set consisting of all linear combinations of the elements of is called
Span( and denoted by
is a subspace of and it is the smallest subspace containing
is said to be spanned (generated) by and is called spanning set for

Example:
} then and is a subspace of

As [ ] [ ] [ ] where [ ], [ ]

Example:

Let be set of all vectors of the form 0 1 then =span{ } where thus is a

subspace of

Example:

Let be set of all vectors of the form [ ] where are arbitrary. Then

} and hence is a subspace of


P a g e | 237

Example:
The polynomials span the vector space of all polynomials of degree less than
or equal to So span }

Theorems:

Linear Span of
If is subspace of then and conversely.
If } and } are two set of vectors in any vector space , then
span span if and only if each vector in is a linear combination of those in and each
vector in is a linear combination of those in .

Null Space of a Matrix:


Consider the following system of homogeneous equations:

In the Matrix form where * + the set of all solutions of is the


null space of matrix .
The null space of an matrix , written as Nul( , is the set of all solutions of the
homogeneous equation ,
Null( is in and } or we can say Nul( is the set of all in that are
mapped into zero vector of via the linear transformation

Example:

Let =* + and let [ ] then is in Nul( as

Example:

Let =* + and let [ ] then is not in Nul( as * +.

Theorem:
The null space of an matrix is a subspace of

Example:
Find a spanning set for the null space of the matrix
P a g e | 238

[ ]

Solution:

First of all we will find the general solution of in terms of free variables.
After reducing the matrix in reduced row echelon form we have

[ ]

The general solution is , with and are free.


Now decompose the vector giving the general solution into a linear combination of the vectors
where the weights are the free variables.

[ ] [ ] [ ] [ ] [ ]
Every linear combination of and is an element of Nul( Thus { } is a spanning
set for Nul( .

Column Space of a Matrix:


The Column space of an matrix , written as Col( , is the set of all linear combinations
of the columns of If } then
Span }
Or
Col( for some in }

Example:
Find a Matrix such that

{[ ] }

Since

{ [ ] [ ] } {[ ] [ ] }.

Let =[ ] then
P a g e | 239

Theorems:
 The Column space of an matrix is a subspace of
 The Column space of an matrix is all of iff the equation has a
solution for each

Contrast Between Null( ) and Col( ) for an matrix .


 Null( is implicitly defined. i.e we have only a condition which is satisfied by
the vectors in Nul( while Col( is explicitly defined. i.e we are aware of the method
to build vectors in Col(
 Row operations on are required to find vectors in Nul , on the other hand to find
vectors in Col( is quite easy as the columns of are given and others are formed from
them.
 There is no obvious relation between Null( and the entries in , while there is an
obvious relationship between Col( and entries in
 A vector in Nul( has the property that whereas a vector in Col( has the
property that the equation is consistent.
 Null( } iff the equation has only the trivial solution while Col(
iff the equation has a solution for each in .
 Null( } iff the linear transformation is one-to-one,
Whereas Col( iff the linear transformation maps onto .

Linear Transformation:
A Linear transformation from a vector space into a vector space is a rule that assigns to
each vector in a unique vector in such that
for all , in
for all in and all scalars
Or and
The Kernel (Null Space) of such a is the set of all in such that and it is clearly a
subspace of .
The Range of is the set of all vectors in of the form for some in , subspace of .
If is matrix transformation say then Kernel and the range of are just Null Space
and Column space of any matrix .

Examples:

1. The identity map such that and zero map defined by


are linear transformations.
2. Let be vector space of all real-valued functions defined on [ ] with the property that
is differentiable and is continuous on [ ], take [ ] be vector space of all
continuous functions and let be the transformation that changes in to its
derivative in then is linear transformations as
and .
P a g e | 240

Kernel of is the set of constant functions defined on [ ].


3. defined by then is clearly a linear transformation
Whereas }
4. defined by then is not linear transformation as
and but
Also since a linear transformation is always a group homomorphism so identity should map to
identity, but here
5. let } whereas is a linear transformation from vector space to ,
then is clearly a vector space over (just like group of automorphism).

Notations:
}
For any linear transformation and

Result:
Let be linear transformation then Ker } iff is one-one.

Linearly independent set:


An indexed set of vectors } is said to be linearly independent over iff
has only trivial solution. i.e.
If any one of is non-zero then given set is linearly dependent.

Examples:
Zero vector is linearly dependent.
A finite set containing zero vector is linearly dependent.
Any singleton set } is linearly independent iff
Any set consisting of two vectors is linearly dependent iff one of the vectors is multiple of other.
A set with two or more vectors is linearly dependent iff at least one of the vector in is
expressible as a linear combination of other vectors in .
Let , , then set } is linearly dependent
as
The set } is linearly independent in [ ] as there is no scalar c exists such that
[ ]
The set } is linearly dependent in [ ] as [ ]

Geometrical interpretation of linear independence in


 In , a set of two vectors is linearly independent if and only if the vectors do not lie
on the same line when they are placed with their initial points at origin.
This result follows from the fact that two vectors are linearly independent iff neither vector is a
scalar multiple of the other,
P a g e | 241

 In , a set of three vectors is linearly independent iff vectors do not lie on the same
plane when they are placed with their initial points at origin.
This results follows from the fact that three vectors are linearly independent iff none of the
vectors is linear combination of other two.

Theorems:
 An indexed set { } of two or more vectors with is linearly independent iff
some (with is a linear combination of the preceding vectors ,
 If is linearly independent set then every subset of is linearly independent.
 If is linearly dependent set then every superset of is linearly dependent.
 If } is a linearly independent subset of a vector space ,
then the set } is also linearly independent.

Linear Dependence of Functions:


If are 2 times differentiable functions on then

| | called Wronskian. We can extend this determinant for

functions.

Theorem:
If the functions have continuous derivatives on
and if Wronskian of these functions is not identically zero for all in then these
functions form a linearly independent set of vectors in
Remark: The converse of above theorem is false. If the Wronskian of
is identically zero on then no conclusion can be drawn
about linear independence of } This set of vectors may be linearly dependent or
independent.

Convention:
represents vector space of all functions with continuous derivative on
.

Example:
Linear independent set in
The functions and form a linearly independent set of vectors in
As | | which is not zero for all in .
P a g e | 242

Example:
Linear independent set in
The functions , and form a linearly independent set of vectors in

As | | which is non-zero for any in

Basis and Dimension:


Let be any vector space and be any non-empty subset of then is a basis of iff
(ii) is linearly independent
Whereas the dimension of any vector space is the number of elements in its basis set.

Examples:
1. , }
Take any arbitrary vector then thus
Clearly given set is linearly independent , hence is a basis for and
The set is called standard basis for . Note that every linearly independent set of 2
elements is basis set for
2. , }
then is standard basis for and . On generalizing we say
3. over , },
Then is basis set for and
4. ,* + - is a vector space over and ,* + * + * +- is
basis.
5. then and basis elements are
* +,* +, * + * +.
On generalizing for
6. Let be vector space of all symmetric matrices of order over then

7. Let be vector space of all skew symmetric matrices of order over


then
8. √ is a 2 dimensional vector space with basis set √ }
9. is 1- dimensional vector spaces.
10. is a 2-dimensional vector space while basis set is }.
11. Let ( ) and then [ ]
P a g e | 243

Theorems:
 Let { } be a set in and let
If one of the vectors in say is a linear combination of the remaining vectors in ,then
the set formed from by removing still spans .
 If } , some subset of is a basis for .
(above theorem is known as spanning set theorem)
 If a vector space has basis } then any set in containing more than
vectors is linearly dependent.
 If a vector space has basis of vectors, then any basis of consists of exactly vectors.
 A one-to-one linear transformation preserves basis and dimension.
 A linear transformation maps a linearly independent set to a linearly independent set.
 If is linearly independent then can be extended to form basis of
 If } spans a vector space then any subset of is a basis for .

Infinite Dimensional Vector Spaces:


If basis of any vector space is infinite then is called an infinite dimensional vector space.

Examples:
, vector space of all real valued functions ,

An application of Spanning Set Theorem:

{ | } where is a subspace of .

Solution:
Any vector where and

[ ] [ ] [ ] [ ] then set } spans .

But so is linearly dependent and the set } is independent hence

Basis for Null and Col :


Find a basis for for Col where

[ ] where [ ]
P a g e | 244

Solution:
As and so by spanning set theorem we may discard and and
} will still span .

Let } {[ ] [ ] [ ]} since each column is non-zero and none of them is

linear combination of others so is linearly independent and is basis for

Results on Dimension of a vector space:

 Let be a subspace of a finite-dimensional vector space . Any linearly independent set


in can be expanded if necessary to a basis for . Also is finite dimensional and

 Let be a dimensional vector space , . Any linearly independent set of exactly


elements in is automatically a basis for .
 Any finite dimensional vector space contains a basis.
 Let be an -dimensioan vector space, A set of vectors } is a basis for
iff each vector in is uniquely expressed as a linear combination of
 If and are finite-dimensional subspaces of a vector space then

 Let and be subspaces of a vector space , then is a subspace of and ,


Also .

 Let be an subspace of an vector space ,


then
 Let and be subspaces of a vector space if then
}
 Let be a finite-dimensional vector space and let } be any basis
If a set has more than vectors then it is linearly dependent.
If a set has fewer than vectors then it does not span .
 If is an vector space over then is isomorphic to as a vector
space.

The Row Space:


If is an matrix , each row of has entries then each row is a vector in
The set of all linear combination of the row vectors is called the row space of and is denoted
by Row As rows of are the columns of Thus Col is row space of
Before an example to understand row space, we state a theorem.
If two matrices and are row equivalent, then their row space are the same. If is in echelon
form, the non-zero rows of form a basis for the row space of as well as for that of .
P a g e | 245

Example:

[ ].

Solution: to find basis for the row space and column space , row reduce to echelon form

As [ ] by above theorem the first three rows of form a basis for

the row space of Thus basis for row space of A is


}
For the column space, observe from that the pivots are in column 1,2,4 thus columns 1,2,4 of
form a basis for For Null space of we need reduced echelon form. So reduced

echelon form is [ ] so and are free variables.

Thus basis for

{[ ] [ ]}
(Keep in mind that row operations may change linear dependence relations among rows of a
matrix).

Rank Theorem:
The rank of is the dimension of the column space of
Since row is same as Col( ) so the dimension of row space of is the rank of .
The Rank theorem or Rank-Nullity theorem is stated as
If is a matrix then ( )
The pivot columns of a matrix form a basis for
( ) number of pivot columns = number of non-zero rows is row
reduced echelon form of matrix
( ) Number of free variables. (Number of deleted rows).
If is a matrix then ( )
Or
Number of pivot columns + Number of free variables=
Now, we will see an application of Rank Theorem:
P a g e | 246

Example:

[ ]

Solution:
First we will convert given matrix in reduced echelon form

[ ]

after applying row operations , , we get

[ ] so , , while is arbitrary. Thus

[ ] [ ] [ ] = Thus every linear combination of is an element of

And } spans thus by Rank Theorem


Number of pivot columns = 3 , dim 1,

Invertible Matrix Theorem:


Let be an matrix then the following statements are each equivalent to the statement that
is an invertible matrix.
The columns of form a basis of .
Col
Dim(Col
Rank
Null }
Dim(Null

Quotient Spaces:

Let be subspace of a vector space then Quotient space consists of all cosets of the
form is a vector space over under the addition and scalar multiplication defined
as;
P a g e | 247

Theorems:

 Let be a linear transformation then


(Fundamental theorem of Homomorphism for vector spaces)
 If is onto then
 If and be two subspaces of a vector space then
 Let be a subspace of a finite dimensional vector space then
dim dim dim
P a g e | 248

Q:1 The set of all solutions of is

(a) Null space (b) Column space (c) Row space (d) Rank

Explanation:
The set of all solutions of homogeneous system is called Null space.

Q:2 Dimension of when and

(a) (b) (c) (d)

Explanation:
Since ( ) and ( ) so dimension of quotient space is

Q:3 Which of the following is the linear transformation?

(a) defined by
(b) and
(c) where
(d) defined by | |

Explanation:
Consider then but
so is not linear
Also but
So is not a linear transformation.
For take such that and
so is not linear transformation.
is linear transformation. Check

Q:4 If } is a basis of then

(a) (b) (c) (d)


P a g e | 249

Explanation:
Since the set } is a basis of so for any, ,
We have such that .
By comparing real and imaginary parts we get, , .
Converting in matrix form
* +* + * + since given set is basis so * + is linearly independent and

Q:5 Let be space of all solutions of linear homogeneous differential equation


then

(a) (b) (c) (d)

Explanation:
The set } spans the solution space of given D.E
and being the linearly independent set of solutions is basis.
Hence

Q:6 Let } then

(a) 2 (b) 3 (c) 4 (d) 1

Explanation:

Since, [ ] [ ] [ ] [ ]

So the set {[ ] [ ]} spans and being linearly independent set is basis.

Hence
P a g e | 250

Q:7 The dimension of } subspace of is

(a) 2 (b) 3 (c) 4 (d) 1

Explanation:
As and

[ ] [ ] [ ] [ ] [ ] [ ]

Thus the set is basis of and

{[ ] [ ] [ ] [ ]}

Q:8 Let be linear such that then

(a) (b) (c) (d)

Explanation:

but is correct.

Q:9 Let { } then


_____

(a) (b) (c) (d)

Explanation:
Given that
Now,

Since, there are 5 dependent variables.


So,
P a g e | 251

Q:10 Let be linear transformation such that


then ( ) and ( ) are

(a) (b) (c) (d)

Explanation:
As is generated by
So will be generated by
. Now we will check that whether the set } is linearly
independent or not.
As
So the set is linearly dependent and after removing from we get
} as linearly independent set.
Thus } spans and being linearly independent set forms a basis of
hence ( ) .
For ( ) consider then
on comparing we get
, ,

So [ ] [ ] [ ] Thus the vector generates and being

linearly independent set is basis of ,.


So ( )

Q:11 Pick up the basis set for a subspace ,* + - of the


vector space of all real matrices.

(a) ,* + * + * + * +- (b) ,* + * +-

(c) ,* +- (d) ,* + * +-

Explanation:
Using relation,
So, ,* + - , * + -
P a g e | 252

Q:12 The dimension of null space of the matrix [ ] is?

(a) (b) (c) (d)

Explanation:
We know that
For this purpose, we find rank of given matrix.

[ ] [ ] [ ] [ ]

{
Thus,

Q:13 Let defined by a linear


transformation then ?

(a) (b) (c) (d)

Explanation:

Matrix of linear transformation is [ ]

[ ] {

[ ] {

Thus,
P a g e | 253

Q:14 A linear transformation defined by then


( ) ?

(a) (b) (c) (d)

Explanation:
We know that ( )
Matrix of linear transformation is * + which is already in echelon form.
Thus, ( )

Q:15 Let defined by be a


linear transformation then ?

(a) (b) (c) (d)

Explanation:
We know that }

On solving above system of equations, we have


Thus, } }

In matrix notation, [ ] [ ] {

Thus,

Q:16 The order of matrix of linear transformation is?

(a) (b) (c) (d)

Explanation:
Order of matrix of linear transformation
Thus, order of given matrix of linear transformation is
P a g e | 254

Q:17 Let be a linear transformation,


and be the matrix of linear transformation with respect to standard basis then eigenvalues
of are?

(a) (b) (c) (d)

Explanation:
The matrix of linear transformation with respect to standard basis

is [ ] now characteristics equation of matrix is


is one of its root.
On solving remaining roots are

Q:18 Let be a linear transformation defined by


where are standard basis of then pick up the
correct one.

(a) (b) forms a non-invertible matrix of linear transformation.


(c) (d) is a non-diagonalizable matrix.

Explanation:

The matrix of linear transformation is [ ] having

Clearly, is diagonalizable as well as an invertible matrix.

Q:19 Which of the following triplets of functions is linearly independent?

(a) (b) (c) (d) All of these


P a g e | 255

Explanation:
There differentiable functions defined on some closed interval are linearly
independent if and only if

| |

| |

| |

Q:20 Which of the following must be basis of ?

(a) (b)
(c) (d)

Explanation:
A basis of consists of exactly three elements. Option (d) discarded.
Three vectors will form basis for if and only if they are linearly independent.
For option (a) matrix form is given as

[ ] [ ] [ ] {

Since, there exists zero row in echelon form so vectors are not linearly independent.
Similarly, are not linearly independents (verify it, using matrix
echelon form). Now, we check for (c).

[ ] [ ] [ ] {
P a g e | 256

Answers:

Q:1 a Q:2 c Q:3 c Q:4 d Q:5 d


Q:6 a Q:7 c Q:8 c Q:9 c Q:10 a
Q:11 c Q:12 b Q:13 b Q:14 c Q:15 b
Q:16 c Q:17 b Q:18 c Q:19 a Q:20 c
P a g e | 257

Definition: (Inner Product)


Let be a vector space over the field of real or complex numbers.
A mapping is said to be inner product on
If the following conditions are satisfied:
and
̅̅̅̅̅̅̅̅̅̅̅̅̅
Where ̅̅̅̅̅̅̅̅̅̅̅̅̅ denotes the complex conjugate of .

The pair is called inner product space.


is an inner product space where the inner product is dot product that is for
and
is an inner product space where the inner product is usual multiplication of real numbers.
is an inner product space where ̅ for
Let be vector space of all matrices over . Then is an inner product space under the
inner product defined as
where [ ] and
[ ]
is an inner product space with inner product for defined by
where
Let be vector space of all real-valued continuous functions on the interval [ ]
Then for ∫ is an inner product on .
Let be vector space of all matrices over .
Then is an inner product space under the inner product defined as
for .

Definition:
Let be an inner product space and then √ ‖ ‖ is called the norm or length of
vector

Cauchy Schwarz Inequality:


let and be elements of an inner product space over Then | | ‖ ‖‖ ‖
Theorem: The norm in an inner product space satisfies the following axioms for all .

‖ ‖ and ‖ ‖
‖ ‖ | |‖ ‖
‖ ‖ ‖ ‖ ‖ ‖
Definition:

Let be an inner product space and then ‖ ‖‖ ‖


is the angle between
two vectors and where .
Two vectors and are said to orthogonal iff so and are orthogonal iff .
P a g e | 258

Definition:
A set } vectors in an inner product space over is said to be an
orthogonal system if its distinct vectors are orthogonal. i.e.
If
is said to be an orthonormal system if {

Definition:
A square matrix over for which is called an orthogonal matrix.

Theorems:
 Every orthonormal system } is linearly independent.
 The following conditions for a square matrix are equivalent:
 is orthogonal
 The rows of form an orthonormal set.
 The columns of form an orthonormal set.
 If is orthogonal matrix then | |
 Set of orthogonal matrices form a group under multiplication

Eigen Values and Eigen Vector:


If is an matrix over then a scalar is called an eigen value of if there exist a
non-zero column vector such that where is eigenvector of
Now where is identity matrix of order .
since and system has a non-trivial solution iff is singular. i.e
| |

Remarks:
For matrix the equation | | is polynomial of degree and so has
roots, where some of the roots can be repeated.
The polynomial can be written as
[ ]
For | |
Trace(
The equation | | is called the Characteristic equation of .

Example:

Check whether * + and * + are eigen vectors of * +


Since * +* + * + * + so is an eigen vector of .
P a g e | 259

Also * +* + * + * + so is not an eigen vector of

Example:

Find Eigen values and Eigen vector of * +

Solution:
The characteristic equation is | | then and so equation
are the eigenvalues of equation
Now we will find eigenvector corresponding to
Let * + be eigenvector then implies that * +* + * +

on solving we get so * + 0 1 0 1

So eigenvector corresponding to is 0 1. Using the same approach we get * + as an


eigenvector corresponding to

Example:

Find eigenvalues and Eigen vector of [ ]

The characteristic equation for is and roots of the equation are


The eigenvalue is of multiplicity

For eigenvector corresponding to the where 0 1

[ ]0 1 [ ] applying row operations on coefficient matrix we get

[ ]0 1 [ ]

Let then then 0 1 [ ] [ ] hence [ ]

Scalar multiples of [ ] are all eigenvectors corresponding to and forms a subspace


of generated by [ ] which is the Eigen space of corresponding to
For eigenvector corresponding to consider then

[ ]0 1 [ ] let and then


P a g e | 260

And 0 1 0 1 [ ] [ ] so there are two linearly independent eigen vectors

[ ] [ ] corresponding to .

Any linear combination of [ ] [ ] is also an eigenvector corresponding to . Thus

{[ ] [ ]} is basis of Eigen space of corresponding to

Cayley-Hamilton Theorem:
Every matrix is a root of its characteristics polynomial satisfies its characteristics
equation.

Example:

Let us consider a matrix * + We find its characteristics polynomial as:


| | | |
We verify Cayley-Hamilton Theorem that is he root of its characteristics polynomial.
* + * + * + * + * + * + * +

Remember!
For a matrix, characteristics equation is | |
For a matrix, characteristics equation is ∑ | |
For a triangular matrix characteristics polynomial is

Theorems:
 Non-zero eigenvectors of a matrix corresponding to distinct eigenvalues are linearly
independent.
 If is an eigenvalue of an orthogonal matrix, then | |
 Any two eigenvectors corresponding to two distinct eigenvalues of an orthogonal matrix
are orthogonal.
 Eigen values of a diagonal matrix are its diagonal elements and eigenvectors are the
standard basis vectors.
 A matrix and have same eigenvalues.
 An eigenvector of a square matrix cannot correspond to two distinct eigenvalues.
 If is an eigenvalue of a non-singular matrix then is an eigenvalue of .
 If and are square matrices then and have same eigenvalues.
P a g e | 261

 If are eigenvalues of a square matrix of order then


where is scalar, are eigen values of
 Suppose is an eigenvector of matrices and , then is also an eigenvector of
, where are any scalars.

Definition:
If and are two matrices over then is said to be Similar to if there exists a
nonsingular matrix such that .

Theorems:
 Similarity of matrices is an equivalence relation on the set of all matrices.
 Similar matrices have same eigenvalues.
 An matrix has linearly independent eigenvectors if and only if is similar to a
diagonal matrix.
 The Eigen values of symmetric matrix are all real.
 Eigenvectors of a symmetric matrix corresponding to distinct eigenvalues are
orthogonal.

Definition:
An matrix is said to be diagonalizable if it is similar to a diagonal matrix. i.e. is
diagonalizable if there exists an invertible matrix such that is a diagonal matrix. The
matrix is said to diagonalizable .
If is an orthogonal matrix and is a diagonal matrix then is called
orthogonally diagonalizable and is said to orthogonally diagonalizable .
P a g e | 262

Q:1 Let be a matrix with eigen values Then which can be the eigen value of
.

1 (a) (b) (c) (d)

Explanation:
If is an eigen value of matrix then is an eigenvalue of
So must be eigenvalue of

Q:2 If * + is an eigenvector of * + then ___

(a) (b) (c) (d) None of these

Explanation:
* +* + * + on solving we get

Q:3 The minimum and maximum eigenvalue of [ ] are and Then other

eigenvalue is

(a) (b) (c) (d)

Explanation:
Let be required eigen value and since sum of all eigenvalues of matrix is
so
P a g e | 263

Q:4 Let [ ] and be one of its eigenvalue then which of the following

must be another eigenvalue of ?

(a) (b) (c) (d)

Explanation:
Let be eigenvalues of
We know that
Recall that if a matrix has property “Sum of all entries in each row(column) is zero then one
of its eigenvalue must be zero‟‟.

Q:5 Let [ ] then

(a) (b) (d) (d)

Explanation:
We know that if is an eigenvalue of a matrix then eigenvalue of
is
Also we know that eigenvalues of a triangular matrix are its main diagonal entries.
Using this concept,

1+1+

Q:6 Pick up the equation satisfied by * +?

(a) (b)
(c) (d) None of these

Explanation:
For any matrix, characteristic equation is
| |
Now, | |
Equation becomes as
According to Cayley-Hamilton Theorem, must satisfy equation
P a g e | 264

Q:7 Let [ ] then choose the equation whose root is ?

(a) (b)
(c) (d)

Explanation:
According to Cayley-Hamilton Theorem, every matrix is a root of its characteristics
polynomial. Also we know that, characteristics polynomial of a triangular matrix is
| |

Characteristics equation is which is satisfied by

Q:8 Consider a matrix [ ] then which of the following must be eigenvalue of

(a) (b) (c) (d)

Explanation:
We know that

Then, at least one of the eigenvalue must be zero.

Q:9 Let * + then ?

(a) (b) (c) (d)

Explanation:
According to Cayley-Hamilton Theorem,

Since, so
P a g e | 265

Q:10 Let and be eigenvectors of a matrix with two eigenvalues


and then ?

(a) (b) (c) (d)

Explanation:
Since, is matrix. So, it has total eigenvalues
Now, are linearly independent (verify it).
So, eigenvalue has multiplicity at least
Since,
Now,

Q:11 In an inner product space then


choose appropriate value for ?

(a) (b) – (c) (d)

Explanation:
Given that,
̅ ̅ ̅̅̅

Q:12 Let be a invertible matrix with real entries such that then choose
the best option.

(a) All eigenvalues of are non-zero (b) At least one non-zero eigenvalue of
(c) All eigenvalues of are zero. (d) All eigenvalues of are same.

Explanation:
Given that

[ ]
Eigenvalues of a zero-matrix are all zero.
P a g e | 266

Q:13 Let [ ] and then ?

(a) (b) (c) (d)

Explanation:
According to Cayley-Hamilton Theorem, must satisfy characteristic equation of

To find

| |

Q:14 Two eigenvalues of a matrix * + have ratio for What is another value
of for which eigenvalues have same ratio?

(a) (b) (c) ⁄ (d) ⁄

Explanation:
Let be eigenvalues such that
Also,
Also,
From

Q:15 For a matrix * +, if * + is an eigenvector then corresponding eigenvalue is?

(a) (b) (c) (d)

Explanation:
Let * + and be its corresponding eigenvalue then

* +* + * +
P a g e | 267

Q:16 Pick up the best option.

(a) zero can never be eigenvalue of an invertible matrix.


(b) All eigenvalues of a non-invertible matrix are zero.
(c) There exists at most one zero eigenvalue of a non-invertible matrix.
(d) A non-invertible matrix has at least one zero eigenvalue.

Explanation:
It‟s not necessary that an invertible matrix does not have zero eigenvalue.
For example the matrix * + is invertible but zero is its eigenvalue.
All eigenvalues of a non-invertible matrix need not to be zero.
For example the matrix * + is non-invertible but its eigenvalues are
Null matrix is a zero matrix but its all eigenvalues are zero.
Option (d) is best one.

Q:17 The number of linearly independent eigenvectors of the matrix * + is/are

(a) (b) (c) (d) None

Explanation:
We know that corresponding to distinct eigenvalues, eigenvectors are linearly independent.
So, we find the eigenvalues of given matrix.
Characteristics equation is

Q:18 Let be eigenvalues corresponding to eigenvectors * + * + of a matrix What


will be matrix ?

(a) * + (b) * + (c) * + (d) * +


P a g e | 268

Explanation:
Let be required matrix.
Using, * +* + * +

* + * + ,

Again, * +* + * +

* + * + ,
On solving we get
Thus matrix * +

Q:19 Consider the following statements for a matrix [ ]

(i) is an invertible matrix. (ii) has eigenvalue of multiplicity


(iii) must satisfy (iv) is diagonalizable.
Choose the correct statements.

(a) only (i) and (iv) are correct. (b) only (i), (iii) and (iv) are false.
(c) only (i), (iii) and (iv) are correct. (d) only (iii) and (iv) are correct.

Explanation:
Given matrix is non-invertible as
Characteristics equation of is ∑

According to Cayley-Hamilton Theorem must satisfy equation


On solving
We know that a matrix of order is diagonalizable if number of linearly
independent vectors is equal to
Corresponding to distinct eigenvalues, there exist linearly independent eigenvectors.
Thus matrix is diagonalizable.
P a g e | 269

Q:20 Consider a matrix * + then ?

(a) (b) (c) (d)

Explanation:
Characteristics equation of is
According to Cayley-Hamilton Theorem,

Q:21 Which of the following matrix is orthogonal?

(a) Null matrix (b) Diagonal matrix (c) Symmetric matrix (d) None

Explanation:
A null matrix can never be orthogonal as its inverse does not exist.
A diagonal matrix need not to be orthogonal.
For example, * + is not orthogonal as * +* + * + A
symmetric matrix need not to be orthogonal as above example.

Q:22 Consider a matrix [ ] where Then are two real and distinct
eigenvalues of if

(a) (b) (c) (d)

Explanation:
The characteristics equation of is

So,
Now, we check for
Take then
Now,
P a g e | 270

Q:23 Let be two similar matrices of order such that are eigenvalues of
then ?

(a) (b) (c) (d)

Explanation:
Similar matrices have same eigenvalues.
Also,

Q:24 Let [ ] if then ?

(a) (b) (c) (d) None of these

Explanation:
We find the characteristics equation of using Cayley-Hamilton Theorem and compare it
with given relation to find value of

compare it with
We have,

Q:25 Let be a matrix with an eigenvalue Also, and


then eigenvalues of are?

(a) (b)
(c) (d)

Explanation:
Let be eigenvalues of and
Then

Now, eigenvalues of are


P a g e | 271

Q:26 Let [ ] be a real matrix with eigenvalues and If eigenvectors

corresponding to eigenvalues and are [ ] and [ ] then value of is?

(a) (b) (c) (d)

Explanation:
We know that

Using, [ ][ ] [ ] {

Using, [ ][ ] [ ] {

From equation
Putting
Putting
On solving

So,

Q:27 Which of the following matrix is diagonalizable?

(a) * + (b) * + (c) * + (d) * +

Explanation:
We know that a matrix is diagonalizable if
From above options we see that only fulfill condition (It‟s an exercise verify yourself).
P a g e | 272

Q:28 Eigenvectors of a matrix ______ are orthogonal.

(a) * + (b) * + (c) * + (d) All of these

Explanation:
Eigenvectors of a symmetric matrix corresponding to distinct eigenvalues are orthogonal.
All above are symmetric matrices with distinct eigenvalues.

Q:29 Let be a linear transformation as


then eigenvalues of matrix of are?

(a) (b) (c) (d)

Explanation:

Matrix of linear transformation is [ ]

Now, characteristics equation of above matrix is


eigenvalues are

Q:30 Choose the correct one.

(a) All eigenvalues of a triangular matrices are real.


(b) An eigenvector of a square matrix cannot correspond to two distinct eigenvalues.
(c) Eigenvalues of Eigenvalues of
(d) Eigenvalues of scalar matrices are distinct.

Explanation:
Eigenvalues of scalar matrices (which is also diagonal) are not distinct that is the diagonal
elements.
For two square matrices eigenvalues of both are same.
All eigenvalues of a triangular matrices need not to be real. For example * +
An eigenvector of a square matrix cannot correspond to two distinct eigenvalues.
P a g e | 273

Answers:

Q:1 a Q:2 d Q:3 b Q:4 d Q:5 b

Q:6 a Q:7 c Q:8 a Q:9 a Q:10 c

Q:11 a Q:12 c Q:13 c Q:14 c Q:15 a

Q:16 d Q:17 c Q:18 b Q:19 d Q:20 a

Q:21 d Q:22 d Q:23 c Q:24 b Q:25 c

Q26 b Q:27 c Q:28 d Q:29 b Q:30 b


P a g e | 274

References:
 Contemporary Abstract Algebra by Joseph A Gallian
 Abstract Algebra by David S. Dummit , Richard M. Futtee
 Abstract Algebra Theory and Applications by Thomas W. Judson , Stephen F. Austine
 Elements of Abstract Algebra by Allen Clark
 Elements of Abstract and Linear Algebra by E.H. Connel
 A First Course in Abstract Algebra by John B Fraleigh
 Topics in Algebra by I. N. Herstien
 Abstract Algebra by I. N. Herstien
 A Course in Group Theory by J F Humphreys
 The Theory of Groups by Ian D. Macdonald
 The Theory of Groups by Hall Marshal
 An Introduction to ideas and Methods of the Theory of Groups by Antonio Machi
 A Book of Abstract Algebra by Charles C Pinter
 A Course in Group Theory by John S Rose
 A First Course in Abstract Algebra by Joseph J. Rotman
 A Course on Finite Groups by Harvey E Rose
 Introduction to the Theory of Groups by Walter Ledermann
 Abstract Algebra, A First Course by Dan Saracino
 A Course in Group Theory by J R Robinson
 Schaum‟s outlines of Group Theory
 Marlow Anderson A first course in Abstract Algebra
 Modern Algebra by Linda Gilbert
 Algebra by Micheal Artin
 A Course in Ring and Ideals by David M Burton
 Elementary Linear Algebra by Howard Anton
 Linear Algebra and its Applications by David C Lay

You might also like